You are on page 1of 101

Naskah Soal

SIMAKUI 2010
SELEKSI MASUK UNIVERSITAS INDONESIA

(Matematika Dasar, Bahasa Indonesia, Bahasa Inggris)


Distributed by :

Kemampuan Dasar
Pak Anang

SELEKSI MASUK UNIVERSITAS INDONESIA

SIMAK UI
KEMAMPUAN DASAR
Matematika Dasar Bahasa Indonesia Bahasa Inggris

203
Universitas Indonesia 2010

PETUNJUK UMUM
1. Sebelum mengerjakan ujian, periksalah terlebih dulu, jumlah soal dan nomor halaman yang terdapat pada naskah soal. Naskah soal ini terdiri dari 13 halaman. 2. Tulislah nomor peserta Anda pada lembar jawaban di tempat yang disediakan. 3. Tulislah kode naskah soal ini, pada lembar jawaban di tempat yang disediakan. Kode naskah soal ini: 7. Tulislah jawaban Anda pada lembar jawaban ujian yang disediakan. 8. Untuk keperluan coret-mencoret, harap menggunakan tempat yang kosong pada naskah soal ini dan jangan pernah menggunakan lembar jawaban karena akan mengakibatkan jawaban Anda tidak dapat terbaca. 9. Selama ujian, Anda tidak diperkenankan bertanya atau meminta penjelasan mengenai soal-soal yang diujikan kepada siapapun, termasuk kepada pengawas ujian. 10. Setelah ujian selesai, Anda diharapkan tetap duduk di tempat Anda sampai pengawas ujian datang ke tempat Anda untuk mengumpulkan lembar jawaban. 11. Perhatikan agar lembar jawaban ujian tidak kotor, tidak basah, tidak terlipat, dan tidak sobek.

203

4. Bacalah dengan cermat setiap petunjuk yang menjelaskan cara menjawab soal. 5. Pikirkanlah sebaik-baiknya sebelum menjawab tiap soal, karena setiap jawaban yang salah akan mengakibatkan pengurangan nilai (penilaian: benar +4, kosong 0, salah -1). 6. Jawablah lebih dulu soal-soal yang menurut Anda mudah, kemudian lanjutkan dengan menjawab soal-soal yang lebih sukar sehingga semua soal terjawab.

PETUNJUK KHUSUS
PETUNJUK A:
Pilih satu jawaban yang paling tepat.

PETUNJUK B:
Soal terdiri dari 3 bagian, yaitu PERNYATAAN, kata SEBAB, dan ALASAN yang disusun berurutan. Pilihlah: (A) Jika pernyataan benar, alasan benar, dan keduanya menunjukkan hubungan sebab dan akibat (B) Jika pernyataan benar, alasan benar, tetapi keduanya tidak menunjukkan hubungan sebab dan akibat (C) Jika pernyataan benar dan alasan salah (D) Jika pernyataan salah dan alasan benar (E) Jika pernyataan dan alasan keduanya salah

PETUNJUK C:
Pilihlah: (A) Jika (1), (2), dan (3) yang benar (B) Jika (1) dan (3) yang benar (C) Jika (2) dan (4) yang benar (D) Jika hanya (4) yang benar (E) Jika semuanya benar

Kode Naskah Soal:


MATA UJIAN TANGGAL UJIAN WAKTU JUMLAH SOAL Keterangan : : : : : Matematika Dasar, Bahasa Indonesia, dan Bahasa Inggris 11 APRIL 2010 120 MENIT 60 Mata Ujian MATEMATIKA DASAR nomor 1 sampai nomor 20 Mata Ujian BAHASA INDONESIA nomor 21 sampai nomor 40 Mata Ujian BAHASA INGGRIS nomor 41 sampai nomor 60

203

MATEMATIKA DASAR
Gunakan Petunjuk A dalam menjawab soal nomor 1 sampai nomor 16. 1. x1 dan x2 adalah bilangan bulat yang merupakan akar-akar persamaan kuadrat x2 (2p + 4)x + (3p + 4) = 0, di mana p adalah suatu konstanta. Jika x1 , p, x2 merupakan tiga suku pertama dari suatu deret geometri, maka suku ke-12 dari deret geometri tersebut adalah .... 4. Jika x + y + 2z = k , x + 2y + z = k dan 2x + y + z = k , k = 0, maka x2 + y 2 + z 2 jika dinyatakan dalam k adalah ....

(A) (B) (C) (D) (E)

(A) 1 (B) 1 (C) 6 + 2 5 (D) 6 2 5 (E) 4


2. lim ( 64x2 + ax + 7 8x + b) =
x

k2 16 3k 2 16 4k 2 17 3k 2 8 2k 2 3

3 . Jika a and b 2 bilangan bulat positif, maka nilai a + b adalah ....

(A) 5 (B) 9 (C) 12

(D) 16 (E) 24

5. Jika (p, q ) merupakan penyelesaian dari sistem berikut: 3 log x + 2 log y = 4 3 log (x2 ) 4 log (4y 2 ) = 1, maka nilai p q = ....

3. Ketinggian roket setelah t menit diluncurkan vertikal ke atas dari permukaan tanah memenuhi hubungan h = 65t 5t2 , h dalam km dan t dalam menit. Roket tersebut mencapai ketinggian tidak kurang dari 150 km selama ... menit.

(A) 2 (B) 4 (C) 5

(D) 9 (E) 13

(A) 3 (B) 5 (C) 7

(D) 10 (E) 13

c Universitas Indonesia

Downloaded from http://pak-anang.blogspot.com

Halaman 1 dari 13 halaman

Kode Naskah Soal:


6.
2

203

10. Nilai

log 5. 6 log 5 + 3 log 5. 6 log 5 = .... 2 log 5. 3 log 5

(A) 0 (B) 1 (C) 2

(D) 5 (E) 6

Nilai minimum fungsi f (x, y ) = 500x + 1000y pada daerah yang diarsir adalah ....

11. Dalam suatu penerbangan, penumpangnya terdiri atas 9 anak laki-laki, 5 anak Indonesia, 9 orang laki-laki dewasa, 7 anak laki-laki warga negara asing, 14 warga Indonesia, 6 laki-laki Amerika dewasa, dan 7 perempuan warga negara asing. Jumlah penumpang penerbangan tersebut adalah ....

(A) 8.000 (B) 6.000 (C) 5.750


7.

(D) 5.000 (E) 4.500

(A) 39 (B) 34 (C) 33

(D) 29 (E) 26

12. Persamaan garis l yang menyinggung lingkaran x2 + y 2 = 8 pada titik x = 2 dan memiliki gradien positif adalah ....

(A) y = x 4 (B) y = x + 4 (C) y = 2x + 4


Luas segitiga pada gambar adalah .... cm2 (A) 4(1 3) (B) 4( 3 1) (C) 4( 3 + 1) (D) 2( 3 + 1) (E) 2(1 3) 8. Bilangan bulat terkecil yang memenuhi pertidaksamaan .... 1 32
2x

(D) y = x 8 (E) y = x + 8

13. Fungsi f : dan g : didenisikan sebagai f (x) = 23x1 dan g (x) = 4(x + 2)3 . Jika f 1 adalah invers dari f , maka (f 1 g )(x) = .... (A) 2 log 3 2x

(B) (C) (D) (E)


1 adalah 8

2 2 2 2

log (2x)3 log (2x + 4) log 2x log (2x + 2)

<

2 2x5

14. Untuk 0 < x <

, nilai 2 2 sin x + sin 2x cos x + sin 2x cos3 x + .... = ....

(A) 9 (B) 8 (C) 7

(D) 6 (E) 7

(A) sin x (B) cos x (C) 2 sin x

(D) 2 sec x (E) 2 csc x

9. Diketahui sistem persamaan berikut: 2x + y = 3 (3x 2y 1)(x + y 6) = 0. Jika (x1 , y1 ) dan (x2 , y2 ) adalah penyelesaian dari sistem persamaan tersebut, maka nilai dari x1 + x2 + y1 + y2 = ....

(A) 6 (B) 5 (C) 4


c Universitas Indonesia

(D) 5 (E) 6

Downloaded from http://pak-anang.blogspot.com

Halaman 2 dari 13 halaman

Kode Naskah Soal:


15. Persamaan kuadrat x2 (a2 + 7)x + 4 = 0 mempunyai akar-akar x1 dan x2 . Jika nilai dari x1 x2 + x2 x1 = 8, maka hasil kali dari nilai-nilai a yang memenuhi adalah ....

203

(A) 5 (B) 5 (C) 5 (D) 4 (E) 5


16. lim 1 1 3x 1 x x2 ( ) +( ) = ... 2 2

19. 10 orang yang mengunjungi restoran akan menempati 2 meja bundar. Meja bundar A berukuran besar untuk 6 orang dan meja bundar B untuk 4 orang. Banyaknya cara mereka menggunakan kedua meja tersebut adalah .... (1) 5!.3!
10 (2) C4 .5!.3! 10 (3) C2 10 (4) C6 .5!.3!

(A) 4 (B) 2 (C) 1

(D) 2 (E) 3

20. Diberikan sepasang persamaan 2x 3y = 13 dan 3x + 2y = b dengan 1 b 100, dan b bilangan bulat. Misalkan n2 = x + y , dengan x dan y adalah solusi dari persamaan di atas, yang berupa bilangan bulat, maka nilai n yang memenuhi adalah .... (1) 4 (2) 3 (3) 1 (4) 2

Gunakan Petunjuk C dalam menjawab soal nomor 17 sampai nomor 20. 17. Panitia Perayaan Hari Kemerdekaan RI 17 Agustus yang terdiri dari 4 orang akan dipilih dari 4 pasang suami istri. Banyaknya cara pemilihan panitia tersebut jika .... (1) semua orang dapat dipilih = 70 cara (2) terdiri dari 2 pria dan 2 wanita = 36 cara (3) terdiri dari 3 pria dan 1 wanita = 16 cara (4) semua panitia harus pria = 1 cara 18. Jika 2 sin2 x 7 sin x + 3 < 0 dengan <x< 6 2 1 (2) 3 < tan x < 3 1 (3) 0 < cos x < 3 2 1 (4) < sin x < 3 2 (1) <x< , 2 2 maka pernyataan berikut ini yang benar adalah ....

c Universitas Indonesia

Downloaded from http://pak-anang.blogspot.com

Halaman 3 dari 13 halaman

Kode Naskah Soal:

203

BAHASA INDONESIA
Gunakan Petunjuk A dalam menjawab soal nomor 21 sampai nomor 40. 21. Penggunaan ejaan pada kalimat berikut salah, KECUALI ... 23. Karat sering muncul pada sepeda kesayangan kita manakala kita kurang rajin merawatnya. Terlebih pada bagian pelek yang berlapis krom. Alhasil, penampilan sepeda jadi kurang sip. Bila sudah melekat di sepeda kita, karat akan sulit dihilangkan. Jika terus menyikat atau mengampelasnya, lapisan asli besi bisa rusak. Tanpa sengaja saya memperoleh kiat mengusir karat tanpa merusak lapisan besinya asalkan lapisan asli besi belum mengelupas. Jadi, ketika karat sepeda masih tipis dan baru, langsung gunakan kiat ini. Jangan karena malas, sepeda disimpan di gudang. Karat justru akan senang. Pesan yang ingin disampaikan melalui tulisan di atas adalah ...

(A) Pengacara terdakwa, Parlindungan Simanungkalit, SH, akan mengajukan eksepsi pada sidang minggu depan. (B) Mungkin dr. Brajadenta, MA, SH berkepentingan dengan perkara yang menghebohkan dunia kedokteran dan hukum itu. (C) Prof. Dr. Basa Situmeang memang seorang Profesor dan dia menamatkan pendidikan S3-nya di Bandung setelah terlebih dahulu menekuni bahasa Jerman selama enam bulan di Berlin. (D) Saya katakan bahwa Joni Agus Setyarto, S.H. sengaja datang ke sini untuk melamar anak saya, Ayu Minarti, S.Hum., yang baru saja diwisuda kemarin. (E) Ungkapan "Mangan ora mangan kumpul" perlu ditinjau lagi relevansinya untuk jaman sekarang, jaman Reformasi yang penuh dengan kebebasan itu.
22. Secara sosial bahasa mempunyai makna yang dapat menimbulkan berbagai penafsiran. Membelokkan makna simbolis ke arah pengertian tertentu justru bisa berdampak negatif yang berjangka panjang dan turun-temurun. Pembelokan makna simbolis dari agama, budaya, dan tradisi melalui bahasa, misalnya, akan bisa mendatangkan malapetaka turun-temurun yang menyandera generasi. Bisa dibayangkan dampaknya jika penafsiran yang salah disebarluaskan ke masyarakat. Gagasan utama teks di atas adalah ...

(A) Sepeda akan rusak jika sudah terkena karat. (B) Karat akan mengganggu penampilan sepeda kita. (C) Jangan menganggap remeh karat sepeda kita meskipun masih tipis dan baru. (D) Bila karat sudah di sepeda kita, karat akan sulit dihilangkan. (E) Tanpa sengaja seseorang memperoleh kiat mengusir karat tanpa merusak lapisan besinya asalkan lapisan asli besi belum mengelupas.
24. Para maniak sepak bola dibuat lupa daratan oleh permainan bola ajaib yang telah menjadi salah satu ikon terbesar budaya olahraga tontonan manusia abad XXXXI. Makna ungkapan lupa daratan dalam kalimat di atas adalah ...

(A) lupa diri. (B) tidak peduli pada hal yang lain. (C) bertindak sesuai dengan harga diri. (D) terlalu asyik. (E) bersorak-sorai.

(A) Dampak penafsiran makna bahasa yang salah dapat dibayangkan. (B) Membelokkan makna simbolis berdampak negatif. (C) Makna bahasa dapat menimbulkan banyak penafsiran. (D) Pembelokan makna simbolis mendatangkan malapetaka. (E) Secara sosial bahasa dapat menimbulkan penafsiran.

c Universitas Indonesia

Downloaded from http://pak-anang.blogspot.com

Halaman 4 dari 13 halaman

Kode Naskah Soal:


25. Cady dan para koleganya meneliti 6.997 pasien kanker payudara. Sebagian dari mereka rutin menjalani mamogra pada periode 19901999 dan sebagian lainnya tidak. Perkembangan para pasien tersebut lalu dicatat hingga tahun 2007. Setelah sekitar 12,5 tahun periode follow-up, 461 dari mereka meninggal karena kanker payudara dan hampir 75% dari mereka tidak menjalani pemeriksaan mamogra secara teratur. Simpulan yang dapat diambil dari paragraf di atas adalah ...

203

27. Penulisan gabungan kata dalam kalimat berikut benar, KECUALI ...

(A) Pemeriksaan mamogra yang teratur dapat menghindari risiko kematian pada pasien kanker payudara. (B) Mamogra adalah metode pengobatan kanker payudara bagi perempuan guna menghindari kematian. (C) Perempuan pada umumnya mendapat gangguan kanker payudara karena tidak menjalani pengobatan secara dini. (D) Pada tahun 19901999 Cady dan rekan-rekan mengadakan penelitian terhadap perempuan yang terkena kanker payudara. (E) Tidak semua pasien perempuan terkena penyakit kanker payudara karena rutin menjalani mamogra.
26. Banyak orang yang berniat menambah asupan sayurnya dengan menyantap salad. Namun, jangan percaya bahwa semua salad itu menyehatkan. Ada banyak jenis lemak yang dikandung oleh berbagai sajian salad. Anda harus tahu mana yang memberi manfaat gizi bagi Anda dan mana yang tidak. Tukar beberapa sayuran yang disajikan dengan jenis sayur yang lebih sehat. Maksud kutipan di atas terdapat pada kalimat berikut, KECUALI ...

(A) Saya tidak memberi tahu siapa pun soal kedatangannya. (B) Buku karya Dewi Lestari dipromosikan secara getoktular. (C) Astronot sedang berlatih beradaptasi di wilayah antigravitasi. (D) Orang utan dapat kalian lihat di kebun binatang. (E) "Sebar luaskan informasi penting ini ke segenap karyawan!" perintah manajer itu.
28. Kalimat yang tidak efektif adalah ...

(A) Tindak kekejaman, kekerasan, dan menindas orang kecil merupakan perbuatan tidak terpuji. (B) Ketika saya datang, mereka sudah berkumpul di halaman sekolah untuk menanti inspektur upacara. (C) Tata tertib ini tidak boleh diubah sampai ada tata tertib baru yang disahkan oleh pimpinan. (D) Sebagai mahasiswa, Anda diharapkan dapat memberi keteladanan yang baik. (E) Diperlukan orang yang sanggup berpikir kritis dan tidak ekstrem.
29. Cara penulisan kata berikut sesuai dengan EYD, KECUALI ...

(A) khazanah. (B) konkret. (C) karier.

(D) jadwal. (E) sejarahwan.

(A) Makanan salad ternyata tidak selamanya aman untuk kesehatan. (B) Makanan salad jadi berbahaya jika disajikan dengan varian yang berlemak. (C) Makanan salad baik untuk kesehatan jika sayuran yang disajikan bergizi. (D) Makanan salad menjadi sehat jika tidak diberi mayonais. (E) Makanan salad tidak baik bagi kesehatan jika pengelolaannya tidak tepat.

c Universitas Indonesia

Downloaded from http://pak-anang.blogspot.com

Halaman 5 dari 13 halaman

Kode Naskah Soal:


30. (1) Pembayaran dengan kartu kredit sangat praktis. (2) Kartu kredit mempunyai kelebihan dibandingkan dengan uang tunai. (3) Kartu kredit lebih praktis daripada uang tunai karena dapat dibawa ke mana-mana, sedangkan uang tunai dalam jumlah yang besar sulit atau tidak dapat dibawa ke mana-mana. (4) Kartu kredit juga dapat digunakan untuk pembayaran jarak jauh, sedangkan uang tunai sulit atau tidak dapat digunakan untuk pembayaran jarak jauh. (5) Oleh karena itu, kita perlu memiliki kartu kredit. Kalimat yang mengganggu kepaduan paragraf di atas adalah ...

203

32. Pada saat ini masyarakat semakin dipermudah untuk memiliki kendaraan pribadi. Adanya kemudahan ini tidak lepas dari persaingan di antara perusahaan pembayaran yang sering mengadakan perang harga. Ide kedua kalimat di atas dapat digabung menjadi satu. Kalimat yang paling tepat sebagai penggabungan ide kedua kalimat itu adalah ...

(A) kalimat (1). (B) kalimat (2). (C) kalimat (2) dan (5). (D) kalimat (5). (E) kalimat (1) dan (5).
31. Kata berimbuhan yang dipakai pada konteks yang tidak tepat ditemukan dalam kalimat ...

(A) Burung gereja beterbangan mendengar suara lonceng berdentang. (B) Anak-anak kejar-mengejar di halaman sekolah. (C) Para penonton berlari-larian turun ke tengah lapangan tatkala melihat api berkobar. (D) Amran dan Rio selalu surat-menyurat melalui e-mail. (E) Pemandangan indah terhalang karena lembah berselimutkan kabut.

(A) Pada saat ini masyarakat semakin dipermudah untuk memiliki kendaraan pribadi dan kemudahan ini tidak lepas dari persaingan di antara perusahaan pembayaran yang sering mengadakan perang harga. (B) Pada saat ini masyarakat semakin dipermudah untuk memiliki kendaraan pribadi karena adanya persaingan di antara perusahaan pembayaran yang sering mengadakan perang harga. (C) Pada saat ini persaingan di antara perusahaan pembayaran dalam bentuk perang harga mempermudah masyarakat untuk memiliki kendaraan pribadi. (D) Makin dipermudahnya masyarakat untuk memiliki kendaraan pribadi pada saat ini tidak lepas dari persaingan di antara perusahaan pembayaran yang sering mengadakan perang harga. (E) Perang harga di antara perusahaan pembayaran yang memudahkan masyarakat untuk semakin memiliki kendaraan pribadi sendiri tidak lepas dari persaingan pada saat ini.
33. Banyak perempuan menguatkan diri, lalu meninggalkan keluarga dan kampung halaman untuk bekerja ke luar negeri dengan tawaran gaji yang lumayan besar bagi mereka yang berpendidikan rendah. Kalimat di atas kurang padu karena penggunaan kata tugas yang tidak tepat. Kata tugas yang tidak tepat dalam kalimat di atas adalah ...

(A) dan. (B) yang. (C) dengan.

(D) untuk. (E) lalu.

c Universitas Indonesia

Downloaded from http://pak-anang.blogspot.com

Halaman 6 dari 13 halaman

Kode Naskah Soal:


34. Usaha pembangunan berencana yang pada umumnya dilakukan oleh hampir semua negara yang sedang berkembang itu memang bertujuan untuk meningkatkan kemakmuran serta mengejar kemajuan teknologi dan ilmu pengetahuan. Gagasan utama kalimat di atas adalah ...

203

(A) Usaha pembangunan berencana dilakukan oleh hampir semua negara. (B) Usaha pembangunan berencana memang bertujuan. (C) Usaha pembangunan berencana meningkatkan kemakmuran. (D) Usaha pembangunan berencana mengejar kemajuan teknologi dan ilmu pengetahuan. (E) Usaha pembangunan berencana di hampir semua negara yang sedang berkembang.
35. Kalimat-kalimat berikut tidak efektif, KECUALI ...

36. Bayi yang dilahirkan tidak dengan berat badan rendah pada umumnya mempunyai status gizi saat lahir yang kurang lebih sama dengan status gizi bayi di Amerika. Akan tetapi, seiring dengan bertambahnya umur ditambah faktor-faktor lainnya, sebagian besar bayi tersebut terus mengalami penurunan status gizi. Puncak penurunan terjadi pada umur kurang lebih 1824 bulan. Pada kelompok umur inilah prevalensi balita kurus (wasting) dan balita pendek (stunting) mencapai titik tertinggi (Hadi, 2001). Setelah melewati umur 24 bulan, status gizi balita umumnya mengalami perbaikan meskipun tidak sempurna. Topik yang dibicarakan dalam bacaan tersebut adalah ...

(A) status gizi balita. (B) perbandingan gizi balita di Indonesia dan Amerika. (C) kasus gizi buruk pada balita. (D) prevalensi balita kurus dan balita pendek. (E) perbaikan status gizi balita.
37. Formosa adalah nama lain Taiwan dalam bahasa Portugis yang berarti pulau cantik. Nama itu disematkan bukan tanpa alasan. Kecantikan alamnya menggaung ke berbagai penjuru dunia. Salah satunya adalah Alishan yang membuat seseorang berada di atas awan ketika datang ke sana. Selain itu, ada pula Danau Sun Moon. Danau itu merupakan danau terbesar di Taiwan. Kalimat yang dapat melanjutkan paragraf tersebut adalah ...

(A) Tulisan ini membahas tentang peran dan fungsi kepala desa dalam memimpin masyarakat. (B) Dalam tulisan ini dikaji peran dan fungsi kepala desa dalam memimpin desa. (C) Supermal Karawaci beroperasi setiap hari Senin hingga Minggu dari pukul 09.00-22.00. (D) Makalah ini akan membuktikan bagaimana sosietas memengaruhi kepribadian seseorang. (E) Sejarah Kota Yogyakarta sampai berakhirnya kekuasaan Belanda dan Jepang merupakan daerah yang menganut sistem feodal.

(A) Tidak ada orang di Taiwan yang tidak mengetahui danau yang terkenal indah itu. (B) Banyak wisatawan mendatangi daerah perkotaan Taiwan. (C) Keindahan alam Taiwan memang mengundang banyak wisatawan. (D) Keindahan Taiwan dapat pula dijumpai pula di daerah perkotaan, misalnya di Taipei. (E) Pemerintah Taiwan berusaha meningkatkan pendapatan dari sektor pariwisata.

c Universitas Indonesia

Downloaded from http://pak-anang.blogspot.com

Halaman 7 dari 13 halaman

Kode Naskah Soal:


38. Bagi mereka yang terjun langsung di lapangan perlu juga diingatkan, jangan menambah penderitaan para korban. Bantuan yang ada hendaknya disalurkan dengan tepat sasaran. Bantuan untuk bencana ini harus dikelola dengan jujur dan transparan. Derita para korban jangan ditambah lagi karena keinginan memuaskan nafsu serakah. Melalui bencana ini, marilah kita mengikat persaudaraan kita dengan suatu rasa kebersamaan, kearifan, dan kecerdasan kita dalam merajutnya. Tragedi yang terjadi di Sumatra Barat ini adalah derita kita bersama. Derita anak bangsa. Karena itu, segala daya, upaya, dan pengorbanan harus dimaksimalkan demi meringankan penderitaan para korban. Ini adalah derita kita bersama, derita yang harus merangsang rasa kemanusiaan kita untuk berbicara. Marilah kita bantu dengan ikut meringankan beban mereka. Pernyataan yang sesuai dengan isi teks di atas adalah ...

203

39. (1) O, ibunya sayang, setua itu ia masih rajin pergi ke pasar seperti dulu. (2) Ia tidak berkata lagi, melangkah meninggalkan adiknya sendirian di dapur. (3) Tanah masih basah bekas hujan semalam, namun cuaca bersih sekali, langit biru merata dan puncak Gunung Kelud jauh di timur tersaput merah. (4) Pelan sekali seperti merayapi puncaknya, matahari naik juga akhirnya. (5) Hari masih pagi. (6) Burung jalak berlompatan di tanah samping perigi, kutilang ramai beterbangan dari dahan ke dahan. (7) Kandang sapi ... (Dikutip dari novel Pulang karya Toha Mohtar. Jakarta: Pustaka Jaya, 1994 [cet. ke-5], hlm. 22). Makna kata tersaput merah pada kalimat (3) dalam teks di atas adalah ...

(A) Pengelolaan bantuan untuk bencana Sumatra Barat yang terjun langsung di lapangan diharapkan tepat sasaran dan transparan. (B) Diharapkan kita semua membantu korban bencana Sumatra Barat dengan jujur dan transparan untuk meringankan beban mereka. (C) Dengan adanya bencana gempa bumi yang melanda Provinsi Sumatra Barat diharapkan kita bahu-membahu membantu mereka. (D) Kita bangsa Indonesia harus turut berduka atas terjadinya musibah gempa bumi di Sumatra Barat dengan memberikan bantuan. (E) Marilah kita membantu korban bencana gempa bumi di Sumatra Barat dengan rasa kemanusiaan, baik moral maupun material.

(A) terhiasi/dihiasi warna merah dari sinar matahari pagi. (B) ternodai, terkotori/dinodai warna merah dari sinar matahari pagi. (C) tertutup/terlapisi warna merah dari sinar matahari pagi. (D) terdominasi warna merah dari sinar matahari pagi. (E) tersapu/tertelan warna merah dari sinar matahari pagi.
40. "Peta bahasa ini yang merupakan peta yang menunjukkan data inventarisasi bahasa daerah di Indonesia serta wilayah persebarannya," kata Kepala Pusat Bahasa Depdiknas di sela-sela gladi bersih pameran pendidikan di Sasana Budaya Ganesha Institut Teknologi Bandung. Kalimat tersebut tidak efektif. Kalimat tersebut dapat menjadi efektif jika diperbaiki dengan cara ...

(A) mengubah kata inventarisasi dengan inventaris. (B) menambah -lah pada kata ini. (C) mengubah kata persebarannya dengan penyebarannya. (D) mengganti kata gladi bersih dengan gladi resik. (E) mengganti kata tengah-tengah.
di sela-sela dengan di

c Universitas Indonesia

Downloaded from http://pak-anang.blogspot.com

Halaman 8 dari 13 halaman

Kode Naskah Soal:

203

BAHASA INGGRIS
(1) ____________________________ (2) Depending on what it resembles, a doll may seem to the little girl who plays with it to be a baby, a child, or an older person. (3) The baby doll presumably appeals to the girls developing sense of her own nature as a female, and by "mothering" the doll she strengthens her role identity. (4) Some dolls, on the other hand, can be bought in the same size and "age" as the child, up to maxima usually of about 40 inches and 6 years of age; these dolls can actually wear the same clothes the little girl herself wears. (5) Collecting dolls is a very popular hobby. (6) Their appeal therefore seems to lie in a sort of companionship they provide in the girls imagination. (7) Finally, the dolls that represent older people presumably provide a focus for the childs ego ideal, appealing to his or her sense of a desired future identity. (8) In Japan, for instance, where dolls have been very important for millennia, both boys and girls celebrate annual festivals during which they are presented with dolls that represent men and women outstanding in Japanese history; during the festivals manly and womanly virtues are praised for the childrens edication. Gunakan Petunjuk A dalam menjawab soal nomor 41 sampai nomor 42. 41. The paragraph should begin with _______.

(A) Baby dolls, child dolls, older dolls are common examples of dolls (B) There seems to be three ways in which dolls are valued (C) Dolls attract children regardless their age and sex (D) There are several kinds of dolls that girls prefer to buy (E) The annual doll festivals are very important for Japanese children
42. The sentence which is irrelevant to the topic of the text is sentence number _______.

(A) three (B) four (C) ve

(D) six (E) seven

c Universitas Indonesia

Downloaded from http://pak-anang.blogspot.com

Halaman 9 dari 13 halaman

Kode Naskah Soal:

203

Reality television is a genre of television programming which, it is claimed, presents unscripted dramatic or humorous situations, documents actual events, and features ordinary people rather than professional actors. It could be described as a form of articial or "heightened" documentary. Although the genre has existed in some form or another since the early years of television, the current explosion of popularity dates from around 2000. Reality television covers a wide range of television programming formats, from game or quiz shows which resemble the frantic, often demeaning programmes produced in Japan in the 1980s and 1990s (a modern example is Gaki no tsukai), to surveillance- or voyeurism- focused productions such as Big Brother . Critics say that the term "reality television" is somewhat of a misnomer and that such shows frequently portray a modied and highly inuenced form of reality, with participants put in exotic locations or abnormal situations, sometimes coached to act in certain ways by o-screen handlers, and with events on screen manipulated through editing and other post-production techniques. Part of reality televisions appeal is due to its ability to place ordinary people in extraordinary situations. For example, on the ABC show, The Bachelor , an eligible male dates a dozen women simultaneously, travelling on extraordinary dates to scenic locales. Reality television also has the potential to turn its participants into national celebrities, in talent and performance programs such as Pop Idol, though frequently Survivor and Big Brother participants also reach some degree of celebrity. Some commentators have said that the name "reality television" is an inaccurate description for several styles of program included in the genre. In competition-based programs such as Big Brother and Survivor, and other special-living-environment shows like The Real World, the producers design the format of the show and control the day-to-day activities and the environment, creating a completely fabricated world in which the competition plays out. Producers specically select the participants, and use carefully designed scenarios, challenges, events, and settings to encourage particular behaviours and conicts. Mark Burnett, creator of Survivor and other reality shows, has agreed with this assessment, and avoids the word "reality" to describe his shows; he has said, "I tell good stories. It really is not reality TV. It really is unscripted drama." Gunakan Petunjuk A dalam menjawab soal nomor 43 sampai nomor 47. 43. The topic of this text is _______. 45. The words "heightened documentary" in paragraph 1 means that _______.

(A) critics on reality television programs (B) what reality television programs really are (C) the popularity of reality television programs (D) manipulation in reality television programs (E) the appeal of reality television programs
44. In the rst line, the writer says "it is claimed" to show that _______ in the rst sentence.

(A) the story in the reality television program is made up (B) the story in the reality television program is of a real event (C) the reality television is able to locate people in an extraordinary situation (D) the reality television focuses on games and quizzes (E) the production of the reality television programs is well-managed
46. Which statement is NOT true about Survivor?

(A) the writer agrees with the statement (B) everyone agrees with the statement (C) no one agrees with the statement (D) the writer gives his approval to the statement (E) the writer somewhat disagrees with the statement

(A) It is a popular reality TV show. (B) Its creator does not think it shows reality. (C) Its participants can become celebrities like those of pop idol. (D) It involves an important element of competition. (E) It is considered to be in the same category of shows as The Real World.

c Universitas Indonesia

Downloaded from http://pak-anang.blogspot.com

Halaman 10 dari 13 halaman

Kode Naskah Soal:


47. The purpose of the writer is to show _______.

203

(A) that the term "reality television program" is not precise (B) some criticisms on reality television programs (C) various formats that the reality television programs have (D) the maneuvers used in the reality television programs (E) screen manipulation in the reality television programs

c Universitas Indonesia

Downloaded from http://pak-anang.blogspot.com

Halaman 11 dari 13 halaman

Kode Naskah Soal:

203

Gastronomy is the study of the relationship between culture and food. It is often thought ____(48)____ that the term gastronomy refers exclusively to the art of cooking, but this is only a small part of this discipline; it cannot always be said that a cook is also gourmet. Gastronomy studies various ____(49)____ components with food as its central axis. Thus, it is related to the Fine Arts and Social Sciences, and even to the Natural Sciences in terms of the digestive system of the human body. A gourmets principal activities involve discovering, tasting, experiencing, researching, understanding and writing about foods. Gastronomy is, ____(50)____, an interdisciplinary activity. Good observation will ____(51)____ that around the food, there exist dance, dramatic arts, painting, sculpture, literature, architecture, and music; in other words, the Fine Arts. ____(52)____, it also involves physics, mathematics, chemistry, biology, geology, agronomy, and also anthropology, history, philosophy, psychology, and sociology. The application of scientic knowledge to cooking and gastronomy has become known as molecular gastronomy. Gunakan Petunjuk A dalam menjawab soal nomor 48 sampai nomor 52. 48. ....

(A) accurately (B) erroneously (C) surprisingly


49. ....

(D) strangely (E) seriously

(A) culture (B) cultural (C) culturally


50. ....

(D) culturalist (E) culturing

(A) in addition (B) in contrast (C) therefore


51. ....

(D) nevertheless (E) furthermore

(A) convince (B) propose (C) dierentiate (D) refuse (E) reveal
52. ....

(A) However (B) Consequently (C) Hence

(D) Thus (E) Therefore

c Universitas Indonesia

Downloaded from http://pak-anang.blogspot.com

Halaman 12 dari 13 halaman

Kode Naskah Soal:


Gunakan Petunjuk A dalam menjawab soal nomor 53 sampai nomor 60. 53. "Im afraid you cant get away with this kind of work." "_______"

203

57. "Wheres the report, Ben? You told me it would be ready by now. I need it for the board meeting this afternoon." "Dont worry, Jim. _______ before the board meeting."

(A) I guess youre right. I didnt realize it was that bad. (B) Dont worry, Im not going away. (C) Im sorry, I didnt mean to frighten you. (D) Thats the only way I can do anyway. (E) Dont be afraid. Im going to be away.
54. "The earthquake happening at about 3 oclock yesterday caused many people to panic." "Including me, as I _______ on the third-oor of my oce at that time."

(A) I have it nished (B) I have had it nished (C) I will have it nished (D) The report will nish (E) The report has nished
58. "This rock concert is boring and too loud for me." "I agree. _______ leave?"

(A) worked (B) have worked (C) have been working (D) was working (E) had worked
55. "Did you receive our inquiry? When will we receive your conrmation?" "My apology. It seems that _______. Could you possibly resend it?"

(A) Will we (B) Shall we (C) Must we

(D) Would we (E) Dont we

59. _______ as the author of Faust, Goethe, made important contributions to every branch of writing, and from the 1770s until his death all major developments in German literature reected his inuence.

(A) Knowing most widely (B) To be most widely known (C) He was most widely known (D) Having known most widely (E) Most widely known
60. "I wouldnt have bought a new laptop if I had not needed one." This means that I _______ a new laptop.

(A) we mislay your letter (B) your letter is mislaid (C) we had mislaid your letter (D) your letter has been mislaid (E) your letter was mislaid
56. In the treatment of people _______, the diet must include more calories than the body needs for energy so that the excess calories can be stored in the body as fat.

(A) did not need (B) would buy (C) would need

(D) bought (E) didnt buy

(A) whose underweight (B) which are underweight (C) that are to be underweight (D) who are underweight (E) whom are underweight

c Universitas Indonesia

Downloaded from http://pak-anang.blogspot.com

Halaman 13 dari 13 halaman

SELEKSI MASUK UNIVERSITAS INDONESIA

SIMAK UI
KEMAMPUAN DASAR
Matematika Dasar Bahasa Indonesia Bahasa Inggris

204
Universitas Indonesia 2010

PETUNJUK UMUM
1. Sebelum mengerjakan ujian, periksalah terlebih dulu, jumlah soal dan nomor halaman yang terdapat pada naskah soal. Naskah soal ini terdiri dari 13 halaman. 2. Tulislah nomor peserta Anda pada lembar jawaban di tempat yang disediakan. 3. Tulislah kode naskah soal ini, pada lembar jawaban di tempat yang disediakan. Kode naskah soal ini: 7. Tulislah jawaban Anda pada lembar jawaban ujian yang disediakan. 8. Untuk keperluan coret-mencoret, harap menggunakan tempat yang kosong pada naskah soal ini dan jangan pernah menggunakan lembar jawaban karena akan mengakibatkan jawaban Anda tidak dapat terbaca. 9. Selama ujian, Anda tidak diperkenankan bertanya atau meminta penjelasan mengenai soal-soal yang diujikan kepada siapapun, termasuk kepada pengawas ujian. 10. Setelah ujian selesai, Anda diharapkan tetap duduk di tempat Anda sampai pengawas ujian datang ke tempat Anda untuk mengumpulkan lembar jawaban. 11. Perhatikan agar lembar jawaban ujian tidak kotor, tidak basah, tidak terlipat, dan tidak sobek.

204

4. Bacalah dengan cermat setiap petunjuk yang menjelaskan cara menjawab soal. 5. Pikirkanlah sebaik-baiknya sebelum menjawab tiap soal, karena setiap jawaban yang salah akan mengakibatkan pengurangan nilai (penilaian: benar +4, kosong 0, salah -1). 6. Jawablah lebih dulu soal-soal yang menurut Anda mudah, kemudian lanjutkan dengan menjawab soal-soal yang lebih sukar sehingga semua soal terjawab.

PETUNJUK KHUSUS
PETUNJUK A:
Pilih satu jawaban yang paling tepat.

PETUNJUK B:
Soal terdiri dari 3 bagian, yaitu PERNYATAAN, kata SEBAB, dan ALASAN yang disusun berurutan. Pilihlah: (A) Jika pernyataan benar, alasan benar, dan keduanya menunjukkan hubungan sebab dan akibat (B) Jika pernyataan benar, alasan benar, tetapi keduanya tidak menunjukkan hubungan sebab dan akibat (C) Jika pernyataan benar dan alasan salah (D) Jika pernyataan salah dan alasan benar (E) Jika pernyataan dan alasan keduanya salah

PETUNJUK C:
Pilihlah: (A) Jika (1), (2), dan (3) yang benar (B) Jika (1) dan (3) yang benar (C) Jika (2) dan (4) yang benar (D) Jika hanya (4) yang benar (E) Jika semuanya benar

Kode Naskah Soal:


MATA UJIAN TANGGAL UJIAN WAKTU JUMLAH SOAL Keterangan : : : : : Matematika Dasar, Bahasa Indonesia, dan Bahasa Inggris 11 APRIL 2010 120 MENIT 60 Mata Ujian MATEMATIKA DASAR nomor 1 sampai nomor 20 Mata Ujian BAHASA INDONESIA nomor 21 sampai nomor 40 Mata Ujian BAHASA INGGRIS nomor 41 sampai nomor 60

204

MATEMATIKA DASAR
Gunakan Petunjuk A dalam menjawab soal nomor 1 sampai nomor 18. x 1. Jika f (x) = , x = 2 maka f 1 (x) adalah 24 x .... 4. Jika 3 1 3c 3 4 3 3 = 2 1 6 21 2b 3 maka nilai dari a + b + c + d adalah .... 4a d ,

(A) f (x) = (B) f (x) = (C) f (x) = (D) f (x) = (E) f (x) =

2x x2 + 2 2x , x = 1 x2 1 2x , x2 + 1 2x , x= 2 2 x 2 x , x = 1 2 x 1

(A) 47 (B) 37 (C) 27

(D) 17 (E) 7

5. Agar pertidaksamaan 2x2 + 4x + a2 > 6 dipenuhi oleh semua bilangan riil x, maka ....

(A) a > 2 atau a < 2 (B) 2 < a < 2 (C) 2 2 < a < 2 2 (D) a < 2 2 atau a > 2 2 (E) a < 3 atau a > 3
6. Garis y = mx + 5 memotong parabola y = x2 4mx + 4n di titik P dan Q. Jika P = (1,6), maka koordinat Q adalah .... 3 13 (A) ( , ) 2 2 5 + 21 15 + 21 (B) ( , ) 2 2 5 21 15 21 (C) ( , ) 2 2 9 29 (D) ( , ) 4 4 (E) (4,9) 7. Jika f (x) = f 1
5 5

2. Jika A dan B adalah dua kejadian dengan 1 1 11 P (A) = dan P (B ) = serta P (A B ) = , 8 2 16 maka kejadian A dan B adalah ....

(A) saling bebas (B) saling lepas (C) tidak saling bebas (D) saling lepas dan tidak bebas (E) tidak dapat ditentukan hubungannya
3. x1 dan x2 adalah bilangan bulat yang merupakan akar-akar persamaan kuadrat x2 (2p + 4)x + (3p + 4) = 0, di mana p adalah suatu konstanta. Jika x1 , p, x2 merupakan tiga suku pertama dari suatu deret geometri, maka suku ke-12 dari deret geometri tersebut adalah ....

log (x + 1) +

log

1 , maka x2

log 2 = ....

(A) 1 (B) 1
(C) 6 + 2 5 (D) 6 2 5

(A) 3 (B) 4 (C) 5

(D) 6 (E) 7

(E) 4
c Universitas Indonesia
Downloaded from http://pak-anang.blogspot.com

Halaman 1 dari 13 halaman

Kode Naskah Soal:


8. Jika x + y + 2z = k , x + 2y + z = k dan 2x + y + z = k , k = 0, maka x2 + y 2 + z 2 jika dinyatakan dalam k adalah ....

204

(A) (B) (C) (D) (E)


9.

k2 16 3k 2 16 4k 2 17 3k 2 8 2k 2 3

11. Sebuah menara dan gedung masing-masing mempunyai tinggi 50 m dan 62 m. Pada saat sudut elevasi matahari mencapai 60o , selisih bayangan menara dan gedung adalah ....

(A) (B) (C)

3 2 3 3 3

(D) 4 3 (E) 8 3

12. Koesien suku tengah dari (3 2x)6 adalah ....

(A) 4320 (B) 2160 (C) 160


13.

(D) 2160 (E) 4320

Luas segitiga pada gambar adalah .... cm2 (A) 4(1 3) (B) 4( 3 1) (C) 4( 3 + 1) (D) 2( 3 + 1) (E) 2(1 3) 10. Sigma membeli 5 kg jeruk impor berlabel diskon 10%, dan 7 kg jeruk lokal berlabel diskon 5%. Sigma membayar dengan pecahan Rp100.000,00 dan menerima uang kembali Rp26.350,00. Kasir menyatakan bahwa jumlah potongan harga sesuai dengan label diskon adalah Rp5.850,00. Jika pada waktu dan di toko yang sama Prima membeli 2 kg jeruk impor dan 3 kg jeruk lokal sejenis dengan yang dibeli Sigma, maka Prima harus membayar sebesar ....

Sebuah tempat air terbuat dari plat baja yang berbentuk separuh tabung (sesuai gambar). Bagian atas terbuka dan kapasitasnya 125 liter. Agar bahan pembuatannya sehemat mungkin, nilai h = ... meter.

14. Persamaan 3 cos x sin x = 2 p dapat dicari penyelesaiannya apabila p memenuhi ....

(A) 1 (B) 5 (C) 10

(D) 50 (E) 100

(A) 4 p 0 (B) 0 p 4 (C) 4 p 2 (D) p 2 atau p 2 (E) 2 p 2

(A) Rp30.600,00 (B) Rp31.650,00 (C) Rp33.000,00

(D) Rp34.500,00 (E) Rp35.150,00

c Universitas Indonesia

Downloaded from http://pak-anang.blogspot.com

Halaman 2 dari 13 halaman

Kode Naskah Soal:


15. Segitiga OAB adalah segitiga sama kaki (OA = OB ). Titik O merupakan titik asal, dan B terletak di sumbu X positif. Jika koordinat titik A(3, 4), maka koordinat titik berat segitiga OAB adalah .... 18.

204

(A) (B) (C) (D) (E)

4 8 , 3 3 4 2 , 3 3 2 8 , 3 3 8 4 , 3 3 8 2 , 3 3 Jarak dari titik R ke garis horizontal adalah .... (A) 3 + 3 cm (B) 3 3 cm (C) 3 + 2 3 cm (D) 6 + 2 3 cm (E) 6 2 3 cm

16. Jika 4 log(2 log x) + 2 log(4 log x) = 2, maka 5 log x + x + 5 = ....

(A) 1 (B) 2 (C) 4

(D) 5 (E) 16

17. Jumlah nilai x dan y yang merupakan bilangan bulat dari sistem persamaan berikut: 2x + 3 y 1 = 0 x2 xy 2y 2 x 4y 2 = 0, adalah ....

Gunakan Petunjuk C dalam menjawab soal nomor 19 sampai nomor 20. 3 3 sin x 1 19. Jika A = untuk < x < 2 sin x cos x 2 2 dan det ( A ) = 1, maka x mempunyai nilai .... 2 5 (2) 6 3 (3) 2 7 (4) 6 (1) 20. Diberikan sepasang persamaan 2x 3y = 13 dan 3x + 2y = b dengan 1 b 100, dan b bilangan bulat. Misalkan n2 = x + y , dengan x dan y adalah solusi dari persamaan di atas, yang berupa bilangan bulat, maka nilai n yang memenuhi adalah .... (1) 4 (2) 3 (3) 1 (4) 2

(A) 7 (B) 1 (C) 1

(D) 3 (E) 7

c Universitas Indonesia

Downloaded from http://pak-anang.blogspot.com

Halaman 3 dari 13 halaman

Kode Naskah Soal:

204

BAHASA INDONESIA
Gunakan Petunjuk A dalam menjawab soal nomor 21 sampai nomor 40. 21. Jika data perusahaan Anda diakses lebih dari satu orang, dari setiap orang dapat diberi hak akses data yang berbeda-beda, password ini akan sangat berguna. Kalimat inti dari kalimat luas tersebut adalah ... 23. Sejumlah badan usaha setelah bencana gempa menimpa Padang, Sumatera Barat, telah melakukan PHK (Pemutusan Hubungan Kerja). Hotel Ambacang melakukan PHK terhadap 120 karyawan dengan pesangon gaji 1 bulan gaji. Hotel Rocky melakukan PHK terhadap 130 karyawan dengan pesangon 3 bulan gaji. Selain itu, Hotel Bima Minang juga akan mem-PHK 200 karyawannya. Sebaliknya, 200 karyawan RS BMC diminta pihak manajemen untuk membuat surat pengunduran diri dengan alasan RS tersebut tidak mampu menggaji karyawan karena bangunan dan peralatan RS rusak berat. Gagasan utama paragraf tersebut adalah ...

(A) Jika data perusahaan Anda diakses lebih dari satu orang. (B) Setiap orang dapat diberi hak akses data yang berbeda-beda. (C) Password ini sangat berguna. (D) Data perusahaan Anda diakses. (E) Setiap orang dapat diberi hak akses data.
22. Untuk menanggulangi pencemaran tanah akibat penumpukan sampah itu dapat dilakukan melalui berbagai cara seperti melalui program 3 R, yaitu reduce, reuse, dan recycle. Perbaikan kalimat yang tepat untuk kalimat tersebut adalah ...

(A) Pascagempa di Padang banyak hotel yang mem-PHK-kan karyawannya. (B) Pengusaha dengan terpaksa harus mem-PHK karyawan. (C) Sejumlah badan usaha melakukan PHK. (D) PHK membuat para pekerja tidak memiliki pekerjaan. (E) PHK terjadi karena perusahaan tersebut bangkrut.
24. Terlepas dari kontroversi mengenai autisme, para peneliti tidak menjumpai hubungan yang jelas antara autisme dan vaksinasi. Beberapa kasus memang memperlihatkan bahwa gejala autisme muncul setelah si anak mendapatkan suntikan vaksin tertentu. Vaksin yang sering dicurigai dapat memicu autisme ialah MMR. Namun, para peneliti yakin ini hanya sebuah kebetulan. Gagasan pokok paragraf di atas adalah ...

(A) Penanggulangan pencemaran tanah akibat penumpukan sampah itu dapat dilakukan melalui program 3 R, yaitu reduce, reuse, dan recycle. (B) Menanggulangi pencemaran tanah akibat penumpukan sampah itu dapat dilakukan melalui berbagai cara. (C) Menanggulangi pencemaran tanah akibat penumpukan sampah itu dapat dilakukan melalui program 3 R, yaitu reduce, reuse, dan recycle. (D) Untuk menanggulangi pencemaran tanah akibat penumpukan sampah itu melalui berbagai cara dapat dilakukan seperti program 3 R, yaitu reduce, reuse, dan recycle. (E) Penanggulangan pencemaran tanah akibat penumpukan sampah itu berbagai cara dapat dilakukan seperti program 3 R, yaitu reduce, reuse, dan recycle.

(A) Para peneliti tidak menjumpai hubungan yang jelas antara autisme dan vaksinasi. (B) Beberapa kasus memperlihatkan bahwa gejala autisme muncul setelah si anak mendapatkan suntikan vaksin tertentu. (C) Vaksin yang sering dicurigai dapat memicu autisme ialah MMR. (D) Para peneliti yakin ini hanya sebuah kebetulan. (E) Gejala autisme yang muncul setelah si anak mendapatkan suntikan vaksin tertentu merupakan sebuah kebetulan.

c Universitas Indonesia

Downloaded from http://pak-anang.blogspot.com

Halaman 4 dari 13 halaman

Kode Naskah Soal:


25. Di antara lima kalimat berikut, kalimat yang penulisannya benar dari segi kaidah adalah ...

204

(A) "Siapa dia, Mas?" tanyanya sambil menunjuk perempuan molek berambut panjang yang dari tadi berdiri mematung di dekat kios Pak Udin. (B) Makna kata eksibel adalah "luwes". (C) "Sewaktu pintu kelas kubuka, kudengar suara serentak dari murid-muridku, "Selamat pagi, Bu Guru," kisah Bu Kiki. (D) Kata depan "di", "ke", dan "dari" dituliskan terpisah dari kata yang mengikutinya, misalnya di kamar, ke atas, dan dari belakang. (E) Harga kaos ini tidak lebih dari Rp. 30.000,- .
26. Kalimat-kalimat berikut tidak efektif, KECUALI ...

(A) Topeng adalah benda dengan bentuk tertentu yang dapat dikenakan seseorang sehingga pada saat seseorang mengenakan topeng tersebut, ia dapat menarik sifat dari topeng yang dikenakannya. (B) Dengan memiliki sepasang mata dan alisnya, dua daun telinga yang dihiasi anting, satu hidung dengan kumis bapang, jambang dan jenggot yang lebat, serta mulut yang terbuka sehingga terlihat gigi yang besar dengan taringnya yang tajam. (C) Sedangkan menurut Kardji, topeng merupakan kata lainnya dari tapel sebagai penutup muka. (D) Tidak dapat diketahui dengan pasti apakah sejak dulu Ondel-ondel digunakan sepasang (lelaki-perempuan), bahkan sekarang diikutsertakan pula anak-anaknya yang lebih kecil. (E) Bila merujuk pada pendapat orang bahwa Ondel-ondel merupakan penjelmaan danyang (setan), maka bentuk yang seram lebih representatif dibanding yang cantik.
27. Di antara kalimat-kalimat berikut, kalimat yang bukan kalimat baku adalah ...

28. Perlu kita sadari bahwa tidak semua tayangan televisi dapat berpengaruh negatif terhadap pembentukan perilaku anak. Ada tayangan yang justru mengajarkan anak berperilaku baik, menghargai pendapat teman, dan hormat terhadap orangtua. Sayangnya, tayangan televisi yang seperti itu sangat sedikit jumlahnya. Hasil penelitian Yayasan Kesejahteraan Anak Indonesia menyebutkan persentase acara televisi khusus anak-anak hanya sekitar 2,7%4,5% dari total tayangan. Yang lebih mengkhawatirkan, materi yang disajikan sangat mengganggu perkembangan perilaku anak-anak karena kebanyakan bertemakan cerita kurang bermutu. ... , diperlukan perhatian yang cukup serius untuk mengantisipasi pengaruh negatif tayangan televisi bagi anak karena anak adalah generasi penerus bangsa dan melalui tangan merekalah pembangunan bangsa ini akan diwariskan. Jadi, sangat tidak mungkin apabila bangsa yang besar ini dibangun oleh generasi muda yang memiliki perilaku buruk. Kata sambung antarparagraf yang harus digunakan pada awal paragraf kedua adalah ...

(A) akan tetapi. (B) meskipun demikian. (C) karena. (D) oleh karena itu. (E) padahal.
29. Di antara lima kalimat berikut, kalimat yang penulisannya benar dari segi kaidah penulisan angka adalah ...

(A) Permintaan para langganan telkomsel segera ditanggapi dengan cepat. (B) Pengendara mobil dilarang melewati jalan itu, kecuali pada hari libur. (C) Sehubungan dengan itu, dikemukakannya saran-saran yang positif. (D) Kita memerlukan pemikiran untuk memecahkan masalah itu. (E) Satu cara terbaik hidup sehat adalah mengatur diet yang seimbang.
c Universitas Indonesia

(A) "Kalau ingin memahami lebih lanjut topik yang sedang kita bahas, Saudara dapat membaca Bab III, Pasal 8, halaman 67," katanya. (B) 15 orang dinyatakan hilang dalam gempa tahun lalu. (C) Dari 33 peserta, delapan belas di antaranya perempuan dan sisanya laki-laki. (D) Saya telah menyerahkan 7 karung beras kepada fakir miskin. (E) Dia dilahirkan tahun 50-an.

Downloaded from http://pak-anang.blogspot.com

Halaman 5 dari 13 halaman

Kode Naskah Soal:


30. Peningkatan jumlah penumpang kereta rel listrik Jabotabek pada Oktober 2007 tercatat sebanyak 182.528 orang dan meningkat menjadi 195.588 orang pada November 2007. Jika kita memperbaiki kalimat di atas, kata yang harus dibuang adalah ...

204

(A) sebanyak. (B) meningkat menjadi. (C) peningkatan. (D) tercatat. (E) jumlah.
31. Baterai nano memiliki kelebihan bisa "tidur panjang" alias tahan lama meski tidak dipakai selama 15 tahun. Dengan begitu, baterai dapat digunakan sebagai power untuk alat sensor monitor radioaktif atau sensor monitor bahan kimiawi beracun lain. Baterai nano ini bisa langsung "hidup" dan dengan waktu singkat memberi energi tinggi saat dibutuhkan. Baterai nano juga baterai pertama yang mampu membersihkan dirinya sendiri setelah digunakan. Ia akan menetralkan cairan kimia beracun yang timbul di dalamnya. Yang dinyatakan mengenai baterai nano dalam teks di atas adalah ...

32. (1) Investasi yang baik adalah investasi emas dalam bentuk batangan ... investasi dalam bentuk tanah. (2) Emas batangan mudah dijual kembali. (3) ..., emas batangan tidak memerlukan ongkos pembuatan seperti halnya emas perhiasan. (4) ..., tidak ada salahnya jika Anda mempertimbangkan investasi emas dalam bentuk batangan. Kata hubung yang paling sesuai untuk mengisi rumpang di atas adalah ...

(A) dibandingkan, lagi pula, dengan demikian (B) daripada, selain itu, jadi (C) dari, sebagai tambahan, jadi (D) dibandingkan dengan, selain itu, dengan demikian (E) daripada, sementara itu, jadi
33. Banyak atlet yang dulunya berprestasi, tetapi sekarang hidupnya susah. Hal ini disebabkan oleh kecenderungan untuk beralih profesi bagi seorang atlet masih sulit. Selain minimnya keterampilan, atlet segan menanggalkan titelnya sebagai "sang juara". Banyak atlet yang masih gengsi untuk menjadi pedagang atau pegawai kantoran. Jadi, mereka memilih menjadi pelatih. Ketika mereka pensiun dari pelatih, mereka tidak mempunyai penghasilan. Akhirnya, hidup mereka menjadi susah. Situasi ini sebenarnya dapat dihindari kalau saja mereka mau berusaha meninggalkan ego mereka sebagai "sang juara". Pesan yang paling tepat yang ingin disampaikan melalui teks di atas adalah ...

(A) kelebihan dan kekurangan baterai nano. (B) kelebihan dan kekurangan baterai nano dibandingkan dengan baterai lain. (C) penggunaan baterai nano sensor monitor bahan kimiawi beracun. (D) kelebihan baterai nano sebagai alat sensor monitor radioaktif. (E) kelebihan-kelebihan baterai nano.

(A) Alih profesi bagi seorang atlet masih sulit. (B) Atlet ingin tetap menjadi sang juara. (C) Atlet harus meninggalkan ego "sang juara". (D) Atlet masih gengsi untuk menjadi pedagang atau pegawai kantoran. (E) Atlet lebih senang menjadi pelatih.

c Universitas Indonesia

Downloaded from http://pak-anang.blogspot.com

Halaman 6 dari 13 halaman

Kode Naskah Soal:


34. Saat ini, sebagian burung di Indonesia berada di tepi jurang kepunahan. Dalam hal ini, tercatat 71 jenis burung endemik berstatus kritis, genting, dan rentan. Di antaranya adalah burung yang merupakan lambang nasional Indonesia, elang jawa. Kalimat yang dapat melanjutkan kutipan di atas adalah ...

204

(A) Burung endemik lain yang berstatus kritis adalah cenderawasih. (B) Burung paok hujan memiliki panjang 18 cm. (C) Peneliti Indonesia tidak memiliki data komplet mengenai migrasi burung daratan. (D) Jenis burung ini patut dilindungi. (E) Habitat burung-burung di Indonesia itu rusak.
35. Kalimat yang tidak efektif adalah ...

37. ... Batik kraton adalah batik yang pada awalnya berkembang di dalam kraton. Batik ini dulu hanya boleh dipakai oleh keturunan bangsawan yang tinggal di lingkungan Kraton Solo atau Puro Mangkunegaran, tetapi sekarang boleh dipakai siapa saja. Batik nonkraton adalah batik yang berkembang di luar kraton, yaitu di kalangan para saudagar. Batik ini sejak dulu boleh dipakai siapa saja. Kalimat yang tepat mengawali paragraf di atas adalah ...

(A) Pemakai batik kraton dan batik nonkraton. (B) Pemakaian batik kraton terbatas, sedangkan batik nonkraton tidak terbatas. (C) Batik kraton dan batik nonkraton adalah batik yang digemari orang. (D) Ada dua jenis batik solo, yaitu batik kraton dan batik nonkraton. (E) Pada awalnya, pemakaian batik kraton dan batik nonkraton berbeda.
38. Kata berimbuhan yang dipakai pada konteks yang tidak tepat ditemukan dalam kalimat ...

(A) Tindak kekejaman, kekerasan, dan menindas orang kecil merupakan perbuatan tidak terpuji. (B) Ketika saya datang, mereka sudah berkumpul di halaman sekolah untuk menanti inspektur upacara. (C) Tata tertib ini tidak boleh diubah sampai ada tata tertib baru yang disahkan oleh pimpinan. (D) Sebagai mahasiswa, Anda diharapkan dapat memberi keteladanan yang baik. (E) Diperlukan orang yang sanggup berpikir kritis dan tidak ekstrem.
36. Ulama itu mempertemukan Hasan dan Azizah. Proses pembentukan kata mempertemukan dalam kalimat di atas sejalan dengan proses pembentukan kata berimbuhan memper-kan dalam kalimat ...

(A) Burung gereja beterbangan mendengar suara lonceng berdentang. (B) Anak-anak kejar-mengejar di halaman sekolah. (C) Para penonton berlari-larian turun ke tengah lapangan tatkala melihat api berkobar. (D) Amran dan Rio selalu surat-menyurat melalui e-mail. (E) Pemandangan indah terhalang karena lembah berselimutkan kabut.

(A) Patih Gadjah Mada berupaya mempersatukan kerajaan-kerajaan Nusantara. (B) Atasan Anda tidak mempersalahkan Anda atas peristiwa yang terjadi kemarin. (C) Mengapa Andi selalu mempertanyakan nilai-nilai kuliahnya? (D) Saya mohon Anda tidak mempersamakan saya dengan dia. (E) Wartawan memperjuangkan orang yang tertindas dengan kata-kata.

c Universitas Indonesia

Downloaded from http://pak-anang.blogspot.com

Halaman 7 dari 13 halaman

Kode Naskah Soal:


39. (1) O, ibunya sayang, setua itu ia masih rajin pergi ke pasar seperti dulu. (2) Ia tidak berkata lagi, melangkah meninggalkan adiknya sendirian di dapur. (3) Tanah masih basah bekas hujan semalam, namun cuaca bersih sekali, langit biru merata dan puncak Gunung Kelud jauh di timur tersaput merah. (4) Pelan sekali seperti merayapi puncaknya, matahari naik juga akhirnya. (5) Hari masih pagi. (6) Burung jalak berlompatan di tanah samping perigi, kutilang ramai beterbangan dari dahan ke dahan. (7) Kandang sapi .... (Dikutip dari novel Pulang karya Toha Mohtar. Jakarta: Pustaka Jaya, 1994 (cet. ke-5), hlm. 22). Makna kata perigi pada kalimat 6 dalam teks di atas adalah ...

204

(A) balai-balai. (B) gudang tua. (C) empang. (D) tanggul. (E) sumur.
40. Perlindungan terhadap kesehatan dan keselamatan kerja sesuai dengan Undang-undang Tenaga Kerja diberlakukan terhadap semua orang di tempat kerja. Anjuran kesehatan dan keselamatan kerja merupakan tugas penting manajemen yang baik di semua wilayah. Perusahaan, bagaimanapun, berkewajiban melindungi semua karyawan di seluruh tempat kerja. Karyawan harus menerima salinan tertulis undang-undang tersebut dari perusahaan agar mereka mengetahui hak mereka. Dari kalimat berikut ini, kalimat yang sesuai dengan kutipan di atas adalah ...

(A) Tidak ada peraturan kesehatan dan keselamatan kerja sebelum sebelum adanya Undang-Undang Tenaga Kerja. (B) Undang-undang kesehatan dan keselamatan kerja hanyalah tanggung jawab karyawan. (C) Karyawan perlu mengetahui isi Undang-Undang Tenaga Kerja karena berkaitan dengan hak kesehatan dan keselamatan mereka selama bekerja. (D) Undang-undang menyatakan bahwa semua karyawan harus menerima salinan tertulis. (E) Perlindungan terhadap kesehatan dan keselamatan kerja berlaku untuk karyawan tetap.

c Universitas Indonesia

Downloaded from http://pak-anang.blogspot.com

Halaman 8 dari 13 halaman

Kode Naskah Soal:

204

BAHASA INGGRIS
European society in the 18th century was a broad pyramid, with the few of the nobility at the top and the masses of the peasantry at the bottom. In western Europe there was an increasing split between the wealthy nobility, who spent much of their time practicing exquisite etiquette at court, and those lower nobles who stayed in the countryside, hunting and running their estates with little concern for either social niceties or abstract ideas. Only a few at the social peak lived the life portrayed in 20th century historical novelsamid bright chandeliers, powdered wigs, and beautiful womenbut it was in this milieu that the century earned its reputation for licentiousness and decadence. Every monarch had his mistresses, and the ways of Versailles were mimicked across Europe, most absurdly in the courts of the tiny German principalities. Gunakan Petunjuk A dalam menjawab soal nomor 41 sampai nomor 42. 41. The topic of this paragraph is _______.

(A) the wealthy and the poor (B) the European pyramid (C) the wealthy nobility in Europe (D) the European luxurious life in the 18th century (E) social life in the 18th century European society
42. The paragraph which precedes this one most likely discusses _______.

(A) European lower nobles (B) wealthy people in Germany (C) family disputes in Europe (D) social class in European society (E) social problems

c Universitas Indonesia

Downloaded from http://pak-anang.blogspot.com

Halaman 9 dari 13 halaman

Kode Naskah Soal:

204

A monkey that has acquired the sole power to hand out apples is generously rewarded with grooming sessions by the other monkeys in its group. However, as soon as another monkey can hand out apples as well, the market value of the rst monkey is halved. The monkeys therefore unerringly obey the law of supply and demand. In the experiment researchers placed food containers with highly-desired pieces of apple in two groups of South African vervet monkeys. For the monkeys there was just one problem: only one in each group could open the food container. This monkey had a low position in the rank order and was therefore scarcely groomed. However, as soon as she acquired the power to hand out apples she was valued more and was groomed a lot by the rest of the group. Yet she could only enjoy that privilege briey; the researchers placed a second food container that could be opened by another low-ranking female. From that moment onwards the market value of the rst monkey was halved, and she was therefore groomed half as often. The experiments revealed that the female monkeys that could open the food containers were groomed more than when they exerted no power over the food production. The females concerned also did not have to groom the other monkeys as long. They were therefore paid for their services as food suppliers. Biological market theory predicts that the market value of these female monkeys should vary according to the law of supply and demand. The fact that the grooming time of the rst monkey was halved as soon as the second monkey gained the power to distribute apples, conrms this idea; the price of goods - in this case the female monkeys who could open the containers - was instantaneously adjusted to the market. Immediately after the opening of the food containers, the researchers registered how long the females were groomed for. The next occasion on which the females could open a container was, however, several days later. The fact that the females were still groomed more indicates that the vervet monkeys apply a strategy that works in the long term. The choice of partners is also inuenced by long-term attitudes; the monkeys can value one monkey relatively more than the others. A change in price grooming for less long if there is another monkey that supplies apples is only possible if a negotiation process takes place. Many economists assume that such negotiations can only take place if they are concluded with a contract. However, the vervet monkeys do not have the possibility to conclude such binding contracts and yet they still succeed in agreeing to a change in price for a service. Gunakan Petunjuk A dalam menjawab soal nomor 43 sampai nomor 47. 43. The most appropriate title for this passage is _______. 45. According to the text, the grooming time of the female monkey that can open a food container _______.

(A) Vervet Monkeys Grooming Habits (B) Why Monkeys Groom (C) Monkeys Reward for Distributing Food (D) Economic Rules of Supply and Demand for Monkeys (E) The Application of Economic Principles
44. The word "unerringly" in paragraph 1 is closest in meaning to _______.

(A) is decided through negotiations with other monkeys (B) is not aected when another female can supply apples (C) is reduced if there is another monkey that supplies apples (D) is determined by the number of apples they can provide (E) is always the same for every monkey
46. Which of the following statements is NOT TRUE, according to the text?

(A) equally (B) precisely (C) progressively (D) abruptly (E) unexpectedly

(A) Vervet monkeys only apply a short term strategy. (B) Female monkeys supplying apples received rewards from other groups. (C) There were two food containers involved in the experiment. (D) Vervet monkeys behave according to the law of supply and demand. (E) Vervet monkeys could have negotiations without binding contracts.

c Universitas Indonesia

Downloaded from http://pak-anang.blogspot.com

Halaman 10 dari 13 halaman

Kode Naskah Soal:


47. This passage would probably be an assigned reading in which of the following course?

204

(A) Economics (B) Science (C) Sociology

(D) Statistics (E) Psychology

c Universitas Indonesia

Downloaded from http://pak-anang.blogspot.com

Halaman 11 dari 13 halaman

Kode Naskah Soal:

204

Major developments have taken place in the eld of agriculture during the last century, one of the most important of which has been the introduction and extensive use of machinery. This has had great eects on the environment and on the lives of millions of people around the world. ____(48)____ , consideration of some of the eorts of agricultural mechanization , both positive and negative, is ____(49)____ for any country currently experiencing an increase in the use of such machines. The ____(50)____ increase in output that has been made possible by more use of mechanization is probably the most important positive eect of this process. The speed of planting crops, spreading fertilizers and pesticides, and harvesting, is phenomenal. All three of these processes contribute to equally enormous increases in production. ____(51)____, mechanization has improved food production during this century and has helped to feed the larger world ____(52)____. Gunakan Petunjuk A dalam menjawab soal nomor 48 sampai nomor 52. 48. ....

(A) Besides (B) Therefore (C) Otherwise


49. ....

(D) In addition (E) However

(A) serious (B) thorough (C) ecient


50. ....

(D) sensitive (E) essential

(A) whole (B) absolute (C) vast


51. ....

(D) urgent (E) gradual

(A) However (B) In addition (C) Besides


52. ....

(D) Thus (E) Moreover

(A) population (B) populated (C) populous

(D) populate (E) populist

c Universitas Indonesia

Downloaded from http://pak-anang.blogspot.com

Halaman 12 dari 13 halaman

Kode Naskah Soal:


Gunakan Petunjuk A dalam menjawab soal nomor 53 sampai nomor 60. 53. "Did you complete your term paper last night?" "_______, but I was too tired"

204

57. "Did you receive our inquiry? When will we receive your conrmation?" "My apology. It seems that _______. Could you possibly resend it?"

(A) I was lucky (B) I should have done it (C) Im terribly sorry (D) I couldnt make it (E) I managed to
54. "The earthquake happening at about 3 oclock yesterday caused many people to panic." "Including me, as I _______ on the third-oor of my oce at that time."

(A) we mislay your letter (B) your letter is mislaid (C) we had mislaid your letter (D) your letter has been mislaid (E) your letter was mislaid
58. Ive just received an email from the bookstore _______ that the newest Harry Potter novel will be available next week.

(A) worked (B) have worked (C) have been working (D) was working (E) had worked
55. "Wheres the report, Ben? You told me it would be ready by now. I need it for the board meeting this afternoon." "Dont worry, Jim. _______ before the board meeting."

(A) to say (B) saying (C) which said

(D) to be saying (E) said

59. "Your hair looks great." "Thanks. I ______________ at that new hairdressers."

(A) cut my hair (B) have cut my hair (C) had my hair cut (D) had to cut my hair (E) was cutting my hair
60. "Ive called him several times to no avail. He must have forgotten to bring his mobile phone." This means that he _______.

(A) I have it nished (B) I have had it nished (C) I will have it nished (D) The report will nish (E) The report has nished
56. _______ his parents had wanted him to be a government administrator when he graduated from college, he ended up to be the most wanted terrorist of the country.

(A) didnt have a mobile phone (B) didnt remember to bring his mobile phone (C) remembered bringing his mobile phone (D) could not remember his mobile phone (E) didnt forget to bring his mobile phone

(A) When (B) While (C) Until

(D) Since (E) Unless

c Universitas Indonesia

Downloaded from http://pak-anang.blogspot.com

Halaman 13 dari 13 halaman

SELEKSI MASUK UNIVERSITAS INDONESIA

SIMAK UI
KEMAMPUAN DASAR
Matematika Dasar Bahasa Indonesia Bahasa Inggris

205
Universitas Indonesia 2010

PETUNJUK UMUM
1. Sebelum mengerjakan ujian, periksalah terlebih dulu, jumlah soal dan nomor halaman yang terdapat pada naskah soal. Naskah soal ini terdiri dari 11 halaman. 2. Tulislah nomor peserta Anda pada lembar jawaban di tempat yang disediakan. 3. Tulislah kode naskah soal ini, pada lembar jawaban di tempat yang disediakan. Kode naskah soal ini: 7. Tulislah jawaban Anda pada lembar jawaban ujian yang disediakan. 8. Untuk keperluan coret-mencoret, harap menggunakan tempat yang kosong pada naskah soal ini dan jangan pernah menggunakan lembar jawaban karena akan mengakibatkan jawaban Anda tidak dapat terbaca. 9. Selama ujian, Anda tidak diperkenankan bertanya atau meminta penjelasan mengenai soal-soal yang diujikan kepada siapapun, termasuk kepada pengawas ujian. 10. Setelah ujian selesai, Anda diharapkan tetap duduk di tempat Anda sampai pengawas ujian datang ke tempat Anda untuk mengumpulkan lembar jawaban. 11. Perhatikan agar lembar jawaban ujian tidak kotor, tidak basah, tidak terlipat, dan tidak sobek.

205

4. Bacalah dengan cermat setiap petunjuk yang menjelaskan cara menjawab soal. 5. Pikirkanlah sebaik-baiknya sebelum menjawab tiap soal, karena setiap jawaban yang salah akan mengakibatkan pengurangan nilai (penilaian: benar +4, kosong 0, salah -1). 6. Jawablah lebih dulu soal-soal yang menurut Anda mudah, kemudian lanjutkan dengan menjawab soal-soal yang lebih sukar sehingga semua soal terjawab.

PETUNJUK KHUSUS
PETUNJUK A:
Pilih satu jawaban yang paling tepat.

PETUNJUK B:
Soal terdiri dari 3 bagian, yaitu PERNYATAAN, kata SEBAB, dan ALASAN yang disusun berurutan. Pilihlah: (A) Jika pernyataan benar, alasan benar, dan keduanya menunjukkan hubungan sebab dan akibat (B) Jika pernyataan benar, alasan benar, tetapi keduanya tidak menunjukkan hubungan sebab dan akibat (C) Jika pernyataan benar dan alasan salah (D) Jika pernyataan salah dan alasan benar (E) Jika pernyataan dan alasan keduanya salah

PETUNJUK C:
Pilihlah: (A) Jika (1), (2), dan (3) yang benar (B) Jika (1) dan (3) yang benar (C) Jika (2) dan (4) yang benar (D) Jika hanya (4) yang benar (E) Jika semuanya benar

Kode Naskah Soal:


MATA UJIAN TANGGAL UJIAN WAKTU JUMLAH SOAL Keterangan : : : : : Matematika Dasar, Bahasa Indonesia, dan Bahasa Inggris 11 APRIL 2010 120 MENIT 60 Mata Ujian MATEMATIKA DASAR nomor 1 sampai nomor 20 Mata Ujian BAHASA INDONESIA nomor 21 sampai nomor 40 Mata Ujian BAHASA INGGRIS nomor 41 sampai nomor 60

205

MATEMATIKA DASAR
Gunakan Petunjuk A dalam menjawab soal nomor 1 sampai nomor 17. 1. Jika f (x) = x x1 , g (x) = dengan x+1 x 1 x = 1, x = 0 maka (f g ) (x) adalah .... 1x 1 , x= 1 2x 2 1 , x=0 x 1 2x , x=1 1x 1 , x=0 x 1+x 1 , x= 1 + 2x 2 3. Diketahui AX = B, BC = D. Jika 1 2 3 2 A= ,C = ,D= 3 5 1 1 maka X adalah .... 7 5 2 1

(A) (B) (C) (D) (E)

2 1 41 19 33 19 54 31

(A) (B) (C) (D) (E)

33 19 54 31 33 54 19 31 41 2 19 1

2. x1 dan x2 adalah bilangan bulat yang merupakan akar-akar persamaan kuadrat x2 (2p + 4)x + (3p + 4) = 0, di mana p adalah suatu konstanta. Jika x1 , p, x2 merupakan tiga suku pertama dari suatu deret geometri, maka suku ke-12 dari deret geometri tersebut adalah ....

4. Jika x + y + 2z = k , x + 2y + z = k dan 2x + y + z = k , k = 0, maka x2 + y 2 + z 2 jika dinyatakan dalam k adalah ....

(A) (B) (C) (D) (E)

(A) 1 (B) 1
(C) 6 + 2 5 (D) 6 2 5

(E) 4

k2 16 3k 2 16 4k 2 17 3k 2 8 2k 2 3

c Universitas Indonesia

Downloaded from http://pak-anang.blogspot.com

Halaman 1 dari 11 halaman

Kode Naskah Soal:


5.

205

Luas segitiga pada gambar adalah .... cm2 (A) 4(1 3) (B) 4( 3 1) (C) 4( 3 + 1) (D) 2( 3 + 1) (E) 2(1 3) 6. Jika y = 3x + 1 , maka nilai y untuk x yang memenuhi x2 2x + 1 2 |x + 1| 0 adalah ....

9. Segitiga ABC mempunyai sisi-sisi a, b dan c, dengan perbandingan a : b : c = 2 : 3 : 4, maka tan(A + B ) = .... (A) 15 15 (B) 4 2 (C) 3 15 (D) 4 (E) 15 10. Grak fungsi f (x) = 3 cos x , terletak di bawah grak g (x) = 2 sin x pada interval ....

(A) 75 < x < 285 (B) 75 < x < 345 (C) x < 75 atau x > 285 (D) x < 75 atau x > 345 (E) x < 15 atau x > 75
11. Jika x dan y adalah bilangan bulat yang memenuhi sistem berikut: 3x + 4y = 32 y>x 3 y < x, 2 maka x + y = ....

(A) y 8 atau y 2 (B) 8 y 0 (C) y 10 atau y 0 (D) 10 y 0 (E) y 8 atau y 0


7. Kota A memiliki 500 pemilih yang semuanya menentukan dua hal pada suatu referendum. Hal pertama menunjukkan bahwa terdapat 375 yang setuju, sementara hal kedua menunjukkan 275 yang setuju. Jika ternyata tepat ada 40 pemilih yang tidak setuju dengan kedua hal tersebut, maka banyaknya pemilih yang setuju tehadap kedua hal tersebut adalah ....

(A) 5 (B) 7 (C) 8

(D) 9 (E) 12

(A) 190 (B) 210 (C) 310

(D) 350 (E) 460


2

12. Jika 1 dan 2 memenuhi persamaan 10 cos 2x + sin x 9 = 0 dengan 1 > 2 , maka csc2 1 csc2 2 = ....

8. Jika x memenuhi persamaan x 4 log x2 = ....

log x

= 16, maka

(A) 41 (B) 9 (C) 1

(D) 9 (E) 41

(A) 2 atau 2 1 (B) 4 atau 4 (C) 1 atau 1 (D) 4 atau 4 1 (E) 2 atau 2

13. Jika diketahui f (x) = |tan(x)| , maka laju perubahan f (x) pada saat x = k , di mana < k < akan sama dengan .... 2

(A) sin(k ) (B) cos(k ) (C) sec2 (k )

(D) sec2 (k ) (E) cot(k )

c Universitas Indonesia

Downloaded from http://pak-anang.blogspot.com

Halaman 2 dari 11 halaman

Kode Naskah Soal:


14. Diketahui f (x) = 3 sin 2x cos 2a 3 cos 2x sin 2a , sin(x a) nilai maksimum dari fungsi tersebut adalah ....

205

Gunakan Petunjuk C dalam menjawab soal nomor 18 sampai nomor 20. 18. Diberikan sepasang persamaan 2x 3y = 13 dan 3x + 2y = b dengan 1 b 100, dan b bilangan bulat. Misalkan n2 = x + y , dengan x dan y adalah solusi dari persamaan di atas, yang berupa bilangan bulat, maka nilai n yang memenuhi adalah .... (1) 4 (2) 3 (3) 1 (4) 2 19. Daerah hasil dari fungsi f (x) = adalah .... (1) 1 (2) 0 (3) 1 (4) (, ) 20. Nilai x yang memenuhi persamaan x2 px + 20 = 0 dan x2 20x + p = 0 adalah .... (1) 10 4 5 (2) 1 (4) 1 (3) 10 + 4 5 x , x=0 |x| 0, x = 0

(A) 2 (B) 3 (C)

(D) 5 (E) 6

15. Zakiya membeli x tangkai bunga seharga y rupiah, dengan x dan y adalah bilangan bulat, y dalam ribuan (misalnya 2 adalah Rp2.000,00). Saat hendak meninggalkan toko, pramuniaganya berkata, "Jika Anda membeli lagi 18 tangkai bunga, saya akan menjualnya dengan harga 6 sehingga Anda hemat 0,6 per lusin tangkai bunga". Nilai x dan y yang memenuhi kondisi ini adalah ....

(A) x = 10, y = 4 (B) x= 12, y = 3 (C) x = 12, y = 4 (D) x = 10, y = 3 (E) x = 15, y = 5
16. Jika a dan b adalah bilangan bulat positif dan b bukan kuadrat dari suatu bilangan bulat, relasi dari a dan b sehingga jumlah dari a + b dan kebalikannya merupakan bilangan bulat adalah ....

(A) a2 = b 1 (B) a2 = b + 1 (C) a = b + 1


2

(D) a2 = b2 + 1 (E) a = b + 1
17. Jika (x1 , y1 ) dan (x2 , y2 ) adalah penyelesaian dari sistem persamaan berikut: 1 5 ( log x) + 3 log y = 4 2 x log 25 y log 9 = 1 maka 5 log x1 x2 3 log y1 y2 = ....

(A) 4 (B) 6 (C) 8

(D) 12 (E) 16

c Universitas Indonesia

Downloaded from http://pak-anang.blogspot.com

Halaman 3 dari 11 halaman

Kode Naskah Soal:

205

BAHASA INDONESIA
Gunakan Petunjuk A dalam menjawab soal nomor 21 sampai nomor 40. 21. Para pegawai negeri menerima gaji setiap awal bulan dan dibelanjakan sebagian untuk keperluan sehari-hari. Kalimat tersebut dapat menjadi efektif jika diperbaiki dengan cara ... 24. Kalimat berikut yang efektif adalah ...

(A) menghilangkan kata para. (B) menambahkan kata pada sebelum kata setiap. (C) mengganti kata dibelanjakan dengan membelanjakannya. (D) menambahkan nya pada kata keperluan. (E) menghilangkan kata awal.
22. Pernahkah mengalami pada saat bangun tidur, mata Anda tampak sembab dan kulit terlihat kusam? Itu artinya Anda kurang tidur. Perlu diketahui, selain asupan nutrisi dari dalam dan luar tubuh, kualitas tidur juga bisa memengaruhi kondisi kesehatan dan keindahan kulit kita. Pernyataan berikut tepat sebagai jawaban untuk pertanyaan dalam kutipan di atas, KECUALI ...

(A) Elizabeth Pahl, pemandu kami, bercerita, udara pagi itu memang sangat luar biasa. (B) Selain itu, karena bentuk topogranya, banyak ditemui jurang-jurang, lidah glestser serta celah-celah es yang berbahaya. (C) Dengan menumpang Aeroot SU-787, dari Moskow kami tiba tepat pukul 19.50 di Miralnye Vody. (D) Meskipun Rusia sudah berubah dan membuka diri ala reformasi, namun sisa-sisa ciri negara sosialis dari negeri tirai besi ini masih terlihat. (E) Senin pagi, 17 Agustus 2009, di tengah kebekuan udara yang mencapai suhu 25o C, kami sudah bangun dan mempersiapkan diri untuk mendaki puncak Gunung Elbrus.
25. Pada malam Minggu diselenggarakan pertunjukan musik di panggung utama. Kata pertunjukan mengalami pola pembentukan yang sama dengan kata berimbuhan per-an dalam kalimat berikut, KECUALI ...

(A) Jam tidur yang tidak cukup dapat mengakibatkan mata sembab dan kulit kusam. (B) Pola makan yang tidak benar akan berdampak bagi kesehatan kulit. (C) Kualitas tidur yang tidak wajar dapat memengaruhi mata sembab dan kulit kusam. (D) Penggunaan krim malam sesaat sebelum tidur akan membuat kulit cerah. (E) Ruang tidur yang mewah dapat membuat tidur nyaman sehingga mata tidak sembab.
23. Kalimat yang tidak efektif adalah ...

(A) Tindak kekejaman, kekerasan, dan menindas orang kecil merupakan perbuatan tidak terpuji. (B) Ketika saya datang, mereka sudah berkumpul di halaman sekolah untuk menanti inspektur upacara. (C) Tata tertib ini tidak boleh diubah sampai ada tata tertib baru yang disahkan oleh pimpinan. (D) Sebagai mahasiswa, Anda diharapkan dapat memberi keteladanan yang baik. (E) Diperlukan orang yang sanggup berpikir kritis dan tidak ekstrem.

(A) Peragaan busana kali ini menampilkan busana pengantin. (B) Saya selalu berupaya memberikan perhatian kepada orang tua. (C) Ani mengagumi perlakuan Ria kepada sahabatnya yang cacat. (D) Persamaan hak kaum perempuan dengan pria terus diperjuangkan. (E) Perbuatan Rio terhadap temannya sulit dimaafkan. 26. (1) Tempe dan tahu buatan Mampang tidak menggunakan formalin. (2) Agar tetap awet, kuncinya adalah masa fermentasi yang sempurna. (3) Biasanya setelah diberi ragi, tempe dibiarkan selama tiga hari hingga benar-benar siap dijual. (4) Tempe dengan proses pembuatan seperti itu memiliki rasa yang enak. (5) Kedelai yang dipakai biasanya dimpor dari Amerika Serikat. Kalimat yang tidak mendukung gagasan utama adalah ... (A) kalimat 2. (B) kalimat 3 dan 4. (C) kalimat 4. (D) kalimat 4 dan 5. (E) kalimat 5.

c Universitas Indonesia

Downloaded from http://pak-anang.blogspot.com

Halaman 4 dari 11 halaman

Kode Naskah Soal:


27. (1) Penularan DBD terjadi melalui gigitan nyamuk Aedes aegypti/Aedes albopictus betina. (2) Nyamuk ini sebelumnya telah membawa virus dalam tubuhnya dari penderita demam berdarah lain. (3) Nyamuk tersebut berasal dari Brazil dan Ethiopia dan sering menggigit manusia pada waktu pagi dan siang. (4) Orang yang berisiko terkena demam berdarah adalah anak-anak yang berusia di bawah 15 tahun. (5) Orang yang tinggal di lingkungan lembap serta daerah pinggiran kumuh juga berpotensi terkena demam berdarah. (6) Penyakit DBD sering terjadi di daerah tropis dan muncul pada musim penghujan. (7) Virus ini kemungkinan muncul akibat pengaruh musim/alam serta perilaku manusia. Perbaikan yang tepat untuk kalimat (3) adalah ...

205

29. Mikhael Novak (The Joy of Sports, 1976) mengemukakan bahwa peristiwa olahraga termasuk sepak bola, hadir seperti "liturgi" atau upacara religius lainnya. Di sana ada tata cara yang nyaris sakral dan mesti dihormati. Fungsinya pun menyerupai fungsi liturgi: memberi manusia perasaan kesatuan utuh dengan hal-hal di luar dirinya, mengajari manusia untuk menghormati "nasib" (kalah/menang), menampilkan model kepahlawanan atau kepengecutan, dan akhirnya merayakan kebersamaan itu sendiri. Maksud pernyataan Mikhael Novak dalam teks di atas adalah ...

(A) Nyamuk tersebut berasal dari Brazil dan Ethiopia. Nyamuk itu senang menggigit manusia pada waktu pagi dan siang. (B) Nyamuk tersebut berasal dari Brazil dan Ethiopia, lalu sering menggigit manusia pada waktu pagi dan siang. (C) Nyamuk tersebut berasal dari Brazil dan Ethiopia. Biasanya, nyamuk itu menggigit manusia pada waktu pagi dan siang. (D) Nyamuk tersebut berasal dari Brazil dan Ethiopia. (E) Nyamuk tersebut sering menggigit manusia pada waktu pagi dan siang.
28. Kata berimbuhan yang dipakai pada konteks yang tidak tepat ditemukan dalam kalimat ...

(A) Olahraga mempunyai nilai sebagaimana layaknya agama. (B) Olahraga dan agama itu sama.

spiritual

(C) Kemiripan antara olahraga dan agama terletak pada liturgi. (D) Nilai sakral dalam olahraga harus dihormati. (E) Nilai spiritual dalam merayakan kebersamaan.
olahraga adalah

30. Asap rokok yang baru mati di asbak mengandung tiga kali lipat bahan pemicu kanker. Kalimat di atas bukan kalimat yang efektif. Agar menjadi efektif, kalimat tersebut dapat diperbaiki menjadi ...

(A) Burung gereja beterbangan mendengar suara lonceng berdentang. (B) Anak-anak kejar-mengejar di halaman sekolah. (C) Para penonton berlari-larian turun ke tengah lapangan tatkala melihat api berkobar. (D) Amran dan Rio selalu surat-menyurat melalui e-mail. (E) Pemandangan indah terhalang karena lembah berselimutkan kabut.

(A) Asap rokok yang baru mati terkandung tiga kali lipat bahan pemicu kanker. (B) Asap rokok yang baru mati di asbak mengandung tiga kali lipat bahan yang memicu kanker. (C) Asap rokok yang baru dimatikan di asbak mempunyai tiga kali lipat bahan pemicu kanker. (D) Asap rokok yang baru mati di asbak memiliki tiga kali lipat bahan pemicu kanker. (E) Asap dari rokok yang baru dimatikan di asbak mengandung tiga kali lipat bahan pemicu kanker. 31. Enam belas orang utan sumatra akan dilepasliarkan di habitat aslinya di Taman nasional Bukit Tigapuluh, Jambi. Makna dilepasliarkan dalam kalimat tersebut adalah ... (A) dilepas lalu dibiarkan hidup bebas. (B) dijadikan binatang liar. (C) dipelihara di alam liar. (D) diberikan kandang di hutan. (E) dilatih untuk hidup di alam bebas.

c Universitas Indonesia

Downloaded from http://pak-anang.blogspot.com

Halaman 5 dari 11 halaman

Kode Naskah Soal:


32. Kalimat berikut yang bebas dari kesalahan penulisan kata adalah ...

205

(A) Prihatmoro ingin mengunjungi bapaknya di Medan pasca nikah bulan ini. (B) Semua yang tinggal di blok itu ikut berbela sungkawa atas meninggalnya Pak Wir yang dikenal sangat dekat dengan mahasiswa. (C) Tuhan Yang Mahakuasa akan melindungi siapa pun yang suka menolong orang lain tanpa pamrih. (D) Pesan saya hanya satu: sebarluaskan informasi penting ini ke segenap masyarakat yang tinggal di daerah kumuh itu. (E) Jarang yang menyadari bahwa kita berangkat dari sudut-pandang yang berbeda dalam menyikapi rencana kedatangan Miyabi ke Indonesia.
33. Pulau Bali tersohor di seantero dunia karena keindahan pemandangannya dan lebih lagi karena kekayaan tradisi kesenian dan religinya. Kita dapat mengira bahwa reputasi yang terpuji itu sepenuhnya disebabkan oleh daya sorgawi yang kini melekat pada Pulau Bali lebih dari sekadar sarana pemasaran usang brosur-brosur pariwisata. Dalam teks tersebut kata yang penulisannya tidak sesuai dengan kaidah EYD adalah ...

36. Burung elang sangat dekat dengan kehidupan masyarakat Arab. Penyair Arab sering membuat puisi yang mengidentikkan burung elang dan kelas masyarakat di sana. ... Di berbagai tempat umum di Abu Dhabi, banyak dijumpai gambar diri sheikh dengan burung elang perkasa bertengger di lengan. Elang bagi masyarakat badawi Arab merupakan satwa yang membantu kelangsungan hidup mereka selama mengembara di gurun pasir. Kalimat yang dapat mengisi bagian yang kosong adalah ...

(A) Elang menjadi simbol Jazirah Arab lebih dari kawasan mana pun. (B) Elang dapat ditemukan di mana pun di Jazirah Arab. (C) Elang adalah lambang keperkasaan. (D) Selain itu, orang Arab juga suka memelihara elang. (E) Elang merupakan lambang kekuasaan bagi mereka.
37. Adanya politisasi terlihat pada masalah investasi riil dari luar negeri. Pihak asing digambarkan "menguasai" perekonomian nasional. Hal ini terlihat pada kasus penjualan Indosat ke Temasek Holdings dan peningkatan peran perusahaan minyak asing. ..., kecuali pada saham perbankan dan transaksi di bursa, nilai investasi asing di Indonesia sangat kecil dibandingkan dengan negara lain. Konsentrasi kepemilikan asing, justru ada di wilayah "panas"(sektor nansial, khususnya bursa) sehingga uang mereka gampang menguap. ..., ketika terjadi krisis nansial, Indonesia tetap terkena. Nilai tukar rupiah jatuh hingga lebih dari Rp12.000,00 per dolar AS. Kata sambung yang tepat untuk menghubungkan kalimat pada paragraf tersebut adalah ...

(A) seantero. (B) sekadar. (C) sorgawi.

(D) religi. (E) tersohor.

34. Penulisan kata dalam kalimat-kalimat berikut ini tidak mengikuti aturan EYD, KECUALI ...

(A) Ketidakberpihakan Saudara pada semua partai sangat kami hargai. (B) Kami akan menindak-lanjuti laporan yang disampaikan Ketua Tim Empat. (C) Para ahli bahasa itu berusaha meng-Indonesia-kan sejumlah kata asing. (D) Kami tidak bertanggungjawab atas kecelakaan bus antarprovinsi itu. (E) Gerakan anti pemerintah itu berusaha menyebarluaskan ideologi mereka.
35. Cara penulisan kata berikut sesuai dengan EYD, KECUALI ...

(A) walaupun demikian; oleh karena itu. (B) sebaliknya; oleh karena itu. (C) oleh karena itu; walaupun demikian. (D) padahal; akibatnya. (E) dengan demikian; di samping itu.

(A) khazanah. (B) konkret. (C) karier.

(D) jadwal. (E) sejarahwan.

c Universitas Indonesia

Downloaded from http://pak-anang.blogspot.com

Halaman 6 dari 11 halaman

Kode Naskah Soal:


38. Gambang semarang sebenarnya merupakan turunan dari gambang kromong yang populer di Jakarta. Irama gambang semarang rancak, mengentak, dan terkesan humoristik. Menurut dugaan, gambang semarang merupakan versi sederhana dari gambang kromong. Adapun gambang kromong adalah ekspresi rindu dendam cina peranakan pada kesenian leluhurnya. Itu adalah musik hibrida: akulturasi dari musik Cina, Betawi, Melayu, dan Deli. Saat ini, gambang kromong memiliki nasib seperti nasib musik tradisional lainnya, yaitu jarang berpentas. Demikian pula dengan nasib gambang semarang. Topik pembicaraan bacaan tersebut adalah ...

205

(A) gambang semarang dan gambang kromong. (B) gambang kromong. (C) gambang semarang. (D) nasib gambang kromong. (E) nasib musik tradisional.
39. Di samping itu, alat-alat kelengkapan yang dimiliki DPR seperti badan kehormatan, komisi-komisi, serta fraksi-fraksi yang ada, harus difungsikan dengan baik. Kepentingan rakyat harus menjadi prioritas utama dibandingkan kepentingan partai maupun kepentingan-kepentingan pihak tertentu. Di pihak lain, untuk anggota DPD, memperjuangkan kepentingan daerah di tingkat pusat merupakan tugas pokok. Peran anggota DPD sangat dibutuhkan mengingat banyaknya aspirasi daerah, khususnya daerah di luar Pulau Jawa, yang kerap tidak terperhatikan oleh pemerintah pusat. Pesan yang terdapat dalam teks tersebut adalah ...

40. Situ Gunung adalah salah satu lokasi wisata tertua di Jawa Barat. Pada zaman Belanda, para noni dan sinyo kerap berlibur di lokasi ini. Pada era keemasan kegiatan pecinta alam 1970-an hingga awal 1990, Situ Gunung merupakan salah satu tempat favorit berkemping. Letaknya yang tersembunyi di lembah di antara bukit-bukit itulah yang menarik perhatian orang. Situ atau danau ini berada pada ketinggian 1.023 meter dari permukaan laut. Danau yang terbentuk dari letusan Gunung Pangrango ini masih menyimpan potensi ora dan fauna yang cukup kaya. Di pohon-pohon sekitar danau, siamang kerap bergelayut dari pohon ke pohon. Hal ini menambah daya tarik Situ Gunung. Pernyataan yang tidak sesuai dengan bacaan tersebut adalah ...

(A) Lokasi wisata Situ Gunung sudah dikenal sejak zaman Belanda. (B) Situ Gunung termasuk dalam kawasan Gunung Gede Pangrango. (C) Pada masa lalu, remaja Belanda sering berwisata ke Situ Gunung. (D) Situ Gunung memiliki kekayaan ora dan fauna. (E) Noni dan sinyo memilih Situ Gunung sebagai tempat berkemping.

(A) Alat-alat kelengkapan yang dimiliki DPR hendaknya difungsikan secara optimal sehingga benar-benar dapat mewakili rakyat. (B) Peran anggota DPR dan DPD hendaknya dapat menyuarakan hati rakyat karena mereka dipilih oleh rakyat. (C) Alat kelengkapan DPR seperti badan kehormatan, komisi-komisi, serta fraksi-fraksi hendaknya jangan melupakan kepentingan partai. (D) DPR sebagai wakil rakyat hendaknya memperhatikan aspirasi rakyat yang berada di daerah, khususnya yang berada di luar Pulau Jawa. (E) Kepentingan rakyat hendaknya diperjuangkan oleh DPR, sementara kepentingan daerah di tingkat pusat merupakan tugas pokok bagi DPD.
c Universitas Indonesia
Downloaded from http://pak-anang.blogspot.com

Halaman 7 dari 11 halaman

Kode Naskah Soal:

205

BAHASA INGGRIS
Humans have been using wind power for at least 5,500 years to propel sailboats, sailing ships, operating windmills, and most recently, generating electricity, though its uses have been limited by the winds speed and strength instability. However, a new study by scientists at the Carnegie Institution and California State University compiled the rst-ever global survey of wind energy available at high altitudes in the atmosphere. The researchers assessed potential for wind power in terms of wind power density, which takes into account both wind speed and air density at dierent altitudes in the worlds ve largest cities: Tokyo, New York, Sao Paulo, Seoul, and Mexico City. They found that there is a huge amount of energy available in high altitude winds, especially near the jet streams. Jet streams are meandering belts of fast winds at altitudes between 20 and 50,000 feet that shift seasonally, but otherwise are persistent features in the atmosphere. Jet stream winds are generally steadier and 10 times faster than winds near the ground, making them a potentially vast and dependable source of energy. Several technological schemes have been proposed to harvest this energy, including tethered, kite-like wind turbines that would be lofted to the altitude of the jet streams. ____________________________ Gunakan Petunjuk A dalam menjawab soal nomor 41 sampai nomor 42. 41. The topic of the paragraph is _______.

(A) future uses of wind power (B) best locations to exploit wind energy source (C) wind power: a new form of renewable energy (D) the potential of wind power at high altitude (E) wind power turbines and generators
42. The sentence that best ends the paragraph is _______

(A) Up to 40 megawatts of electricity could be generated by current designs and transmitted to the ground via the tether. (B) So, while high-altitude wind may ultimately prove to be a major energy source, it requires substantial infrastructure. (C) Good selection of a wind turbine site is critical to economic development of wind power. (D) Meteorologists also use the jet streams as an aid to forecast weather. (E) Will then wind power tapped by high-ying kites light up our homes?

c Universitas Indonesia

Downloaded from http://pak-anang.blogspot.com

Halaman 8 dari 11 halaman

Kode Naskah Soal:

205

No one doubts that the words we write or speak are an expression of our inner thoughts and personalities. But beyond the meaningful content of language, a wealth of unique insights into an authors mind are hidden in the style of a text - in such elements as how often certain words and word categories are used, regardless of context. When people try to present themselves a certain way, they tend to select what they think are appropriate nouns and verbs, but they are unlikely to control their use of articles and pronouns. These small words create the style of a text, which is less subject to conscious manipulation. Social psychologist James W. Pennebaker of the University of Texas at Austin Pennebaker developed a computer program that analyzes text, called Linguistic Inquiry and Word Count (LIWC, pronounced Luke). The software has been used to examine other speech characteristics as well, tallying up nouns and verbs in hundreds of categories to expose buried patterns. Most recently, Pennebaker and his colleagues used LIWC to analyze the candidates speeches and interviews during last falls presidential election. The software counts how many times a speaker or author uses words in specic categories, such as emotion or perception, and words that indicate complex cognitive processes. It also tallies up so-called function words such as pronouns, articles, numerals and conjunctions. Within each of these major categories are subsets: Are there more mentions of sad or happy emotions? Does the speaker prefer "I" and "me" to "us" and "we"? LIWC answers these quantitative questions; psychologists must then gure out what the numbers mean. Before LIWC was developed in the mid-1990s, years of psychological research in which people counted words by hand established robust connections between word usage and psychological states or character traits. The political candidates, for example, showed clear dierences in their speaking styles. John McCain tended to speak directly and personally to his constituency, using a vocabulary that was both emotionally loaded and impulsive. Barack Obama, in contrast, made frequent use of causal relationships, which indicated more complex thought processes. He also tended to be vaguer than his Republican rival. Gunakan Petunjuk A dalam menjawab soal nomor 43 sampai nomor 47. 43. An appropriate title for the text is _______. 45. According to the text, which of the following words would be able to be used to tell about our personality?

(A) Linguistic Analysis Program: A New Invention (B) Psychological Analysis in US Presidents Speeches (C) How the Use of Articles and Pronouns Reveals Our Characters. (D) What Your Choice of Words Says About Your Personality (E) Low-level Words That Give Clues to Large-Scale Behavior
44. According to the text, how does LIWC help analyze president candidates speeches?

(A) Provide (B) Examination (C) The

(D) Analysis (E) Test

46. The word robust in paragraph 3 can be best replaced by _______.

(A) denite (B) vigorous (C) frail

(D) strong (E) slight

47. According to the text, which of the following statements about LIWC is FALSE?

(A) It does the quantitative analysis, which was later used by psychologist. (B) It helps the psychologist reveal the candidates hidden motives. (C) It makes analysis of nouns and verbs used by the candidates during their speeches. (D) It counts how many inappropriate words are used by the candidates. (E) It provides psychological research on the connection of word usage and peoples personality.

(A) The software cannot detect unconsious use of words. (B) The software is able to count certain words and analyze the numbers. (C) By using LIWC, we learn about the characteristics of the US presidential candidates. (D) LIWCs result conrms that peoples characters aect their word usage. (E) The software works on the basis of the psychologists analysis design.

c Universitas Indonesia

Downloaded from http://pak-anang.blogspot.com

Halaman 9 dari 11 halaman

Kode Naskah Soal:

205

Dog agility is a dog sport in which a handler directs a dog through an obstacle course in a race for both time and accuracy. Dogs generally run o-leash with no food or toys as incentives. The handler can touch neither dog nor obstacles.____(48)____, the handlers controls are limited to voice, movement, and various body signals, requiring ____(49)____ training of the animal. In its simplest form, an agility course consists of a set of standard obstacles, laid out by an agility judge in a design of his own choosing on a roughly 100 by 100-foot (30 by 30 m) area, with numbers indicating the order in which the dog must complete the obstacles. Courses are ____(50)____ enough that a dog could not complete them correctly without human direction. In ____(51)____ , the handler must assess the course, decide on handling strategies, and direct the dog through the course, with precision and speed equally important. Many strategies exist to ____(52)____ for the inherent dierence in human and dog speeds and the strengths and weaknesses of the various dogs and handlers. Gunakan Petunjuk A dalam menjawab soal nomor 48 sampai nomor 52. 48. ....

(A) In contrast (B) Consequently (C) However


49. ....

(D) Still (E) Besides

(A) exceptional (B) exceptionally (C) excepting (D) exception (E) excepted
50. ....

(A) regular (B) thorny (C) complicated (D) straightforward (E) interesting
51. ....

(A) compete (B) competitor (C) competitive (D) competitively (E) competition
52. ....

(A) compensate (B) estimate (C) appreciate

(D) examine (E) investigate

c Universitas Indonesia

Downloaded from http://pak-anang.blogspot.com

Halaman 10 dari 11 halaman

Kode Naskah Soal:


Gunakan Petunjuk A dalam menjawab soal nomor 53 sampai nomor 60. 53. "May I be of any assistance, madam?" "_______." 57. This old house is a mess. The owner should _______.

205

(A) xing the ceiling (B) have the ceiling xed (C) have xed the ceiling (D) get the ceiling xing (E) be xing the ceiling
58. "I heard that Dona was awarded a scholarship to continue her study." "Oh? Thats great. She _______ very pleased."

(A) Id love to do this (B) Mind your own business (C) Theres nothing you can do (D) Its beyond your knowledge (E) Youre very kind, but Im just looking
54. "The earthquake happening at about 3 oclock yesterday caused many people to panic." "Including me, as I _______ on the third-oor of my oce at that time."

(A) will be (B) could be (C) might be

(D) must be (E) needs to be

(A) worked (B) have worked (C) have been working (D) was working (E) had worked
55. "Wheres the report, Ben? You told me it would be ready by now. I need it for the board meeting this afternoon." "Dont worry, Jim. _______ before the board meeting."

59. _______ as Het achterhuis in 1947, Anne Franks journal was successfully dramatized by Frances Goodrich and Albert Hackett and presented in New York as The Diary of Anne Frank in 1956.

(A) Originally published in Dutch (B) Having originally published in Dutch (C) Having had originally published in Dutch (D) It was originally published in Dutch (E) It was published originally in Dutch
60. Had there been a guidance to write the report on our survey of Dengue Fever in several parts of East Java, there would not have been variant styles of reports. From the above sentence, we may conclude that _______.

(A) I have it nished (B) I have had it nished (C) I will have it nished (D) The report will nish (E) The report has nished
56. "Did you receive our inquiry? When will we receive your conrmation?" "My apology. It seems that _______. Could you possibly resend it?"

(A) we agreed to one style only (B) the guidance had good results (C) there was no xed format for the report (D) the content of our reports was the same (E) there were dierences in the topics of our report

(A) we mislay your letter (B) your letter is mislaid (C) we had mislaid your letter (D) your letter has been mislaid (E) your letter was mislaid

c Universitas Indonesia

Downloaded from http://pak-anang.blogspot.com

Halaman 11 dari 11 halaman

SELEKSI MASUK UNIVERSITAS INDONESIA

SIMAK UI
KEMAMPUAN DASAR
Matematika Dasar Bahasa Indonesia Bahasa Inggris

206
Universitas Indonesia 2010

PETUNJUK UMUM
1. Sebelum mengerjakan ujian, periksalah terlebih dulu, jumlah soal dan nomor halaman yang terdapat pada naskah soal. Naskah soal ini terdiri dari 12 halaman. 2. Tulislah nomor peserta Anda pada lembar jawaban di tempat yang disediakan. 3. Tulislah kode naskah soal ini, pada lembar jawaban di tempat yang disediakan. Kode naskah soal ini: 7. Tulislah jawaban Anda pada lembar jawaban ujian yang disediakan. 8. Untuk keperluan coret-mencoret, harap menggunakan tempat yang kosong pada naskah soal ini dan jangan pernah menggunakan lembar jawaban karena akan mengakibatkan jawaban Anda tidak dapat terbaca. 9. Selama ujian, Anda tidak diperkenankan bertanya atau meminta penjelasan mengenai soal-soal yang diujikan kepada siapapun, termasuk kepada pengawas ujian. 10. Setelah ujian selesai, Anda diharapkan tetap duduk di tempat Anda sampai pengawas ujian datang ke tempat Anda untuk mengumpulkan lembar jawaban. 11. Perhatikan agar lembar jawaban ujian tidak kotor, tidak basah, tidak terlipat, dan tidak sobek.

206

4. Bacalah dengan cermat setiap petunjuk yang menjelaskan cara menjawab soal. 5. Pikirkanlah sebaik-baiknya sebelum menjawab tiap soal, karena setiap jawaban yang salah akan mengakibatkan pengurangan nilai (penilaian: benar +4, kosong 0, salah -1). 6. Jawablah lebih dulu soal-soal yang menurut Anda mudah, kemudian lanjutkan dengan menjawab soal-soal yang lebih sukar sehingga semua soal terjawab.

PETUNJUK KHUSUS
PETUNJUK A:
Pilih satu jawaban yang paling tepat.

PETUNJUK B:
Soal terdiri dari 3 bagian, yaitu PERNYATAAN, kata SEBAB, dan ALASAN yang disusun berurutan. Pilihlah: (A) Jika pernyataan benar, alasan benar, dan keduanya menunjukkan hubungan sebab dan akibat (B) Jika pernyataan benar, alasan benar, tetapi keduanya tidak menunjukkan hubungan sebab dan akibat (C) Jika pernyataan benar dan alasan salah (D) Jika pernyataan salah dan alasan benar (E) Jika pernyataan dan alasan keduanya salah

PETUNJUK C:
Pilihlah: (A) Jika (1), (2), dan (3) yang benar (B) Jika (1) dan (3) yang benar (C) Jika (2) dan (4) yang benar (D) Jika hanya (4) yang benar (E) Jika semuanya benar

Kode Naskah Soal:


MATA UJIAN TANGGAL UJIAN WAKTU JUMLAH SOAL Keterangan : : : : : Matematika Dasar, Bahasa Indonesia, dan Bahasa Inggris 11 APRIL 2010 120 MENIT 60 Mata Ujian MATEMATIKA DASAR nomor 1 sampai nomor 20 Mata Ujian BAHASA INDONESIA nomor 21 sampai nomor 40 Mata Ujian BAHASA INGGRIS nomor 41 sampai nomor 60

206

MATEMATIKA DASAR
Gunakan Petunjuk A dalam menjawab soal nomor 1 sampai nomor 17. 1. lim
x

4. Diketahui matriks A =

1 2 . Nilai k yang 2 5 memenuhi k. det(AT ) = det(A1 ) adalah ....

x2 + 3x x = ...

(A) 1 (B) 0 3 (C) 2 (D) 3 (E)


2. Pada acara peringatan HUT RI di suatu universitas, diadakan acara pembagian hadiah. Saat pendaftaran, setiap peserta diberi sebuah kupon yang nantinya bisa ditukar dengan hadiah apabila nomor yang dimilikinya sesuai dengan nomor yang terambil pada saat pengocokan. Nomor-nomor yang dibagikan terdiri dari suatu huruf yang diikuti oleh dua angka yang berbeda dan angka kedua haruslah bilangan ganjil. Banyaknya nomor undian yang dibuat adalah ....

(A) 81 (B) 9 (C) 1 1 (D) 9 1 (E) 81


5. Himpunan penyelesaian dari pertidaksamaan x2 2x + 1 |x + 1|, adalah ....

(A) {x R| x = 0} (B) {x R| x > 0} (C) {x R| x 0} (D) {x R| x < 0} (E) {x R| x 0}


6. Jika x + y + 2z = k , x + 2y + z = k dan 2x + y + z = k , k = 0, maka x2 + y 2 + z 2 jika dinyatakan dalam k adalah ....

(A) 71 (B) 260 (C) 520

(D) 650 (E) 1.170

3. x1 dan x2 adalah bilangan bulat yang merupakan akar-akar persamaan kuadrat x2 (2p + 4)x + (3p + 4) = 0, di mana p adalah suatu konstanta. Jika x1 , p, x2 merupakan tiga suku pertama dari suatu deret geometri, maka suku ke-12 dari deret geometri tersebut adalah ....

(A) (B) (C) (D) (E)

(A) 1 (B) 1 (C) 6 + 2 5 (D) 6 2 5 (E) 4

k2 16 3k 2 16 4k 2 17 3k 2 8 2k 2 3

c Universitas Indonesia

Downloaded from http://pak-anang.blogspot.com

Halaman 1 dari 12 halaman

Kode Naskah Soal:


7. 10. Nilai dari lim (2 x + 1)
x

206

4x 3 x + 2 = ....

Luas segitiga pada gambar adalah .... cm2 (A) 4(1 3) (B) 4( 3 1) (C) 4( 3 + 1) (D) 2( 3 + 1) (E) 2(1 3) 8. Seorang pengamat melihat kapal laut yang sedang berlabuh dengan kedudukan menghadap menara. Sudut deviasi pengamat terhadap bagian depan kapal adalah 30 dan sudut deviasi terhadap bagian belakang kapal adalah 60 . Tinggi pengamat adalah 1,8 meter; ia berada di puncak menara yang tingginya 22 m, sedangkan menara tersebut dibangun pada ketinggian 21,2 m di atas permukaan laut. Panjang kapal adalah .... (A) 45 3 45 3 (B) 3 (C) 30 3 30 (D) 3 (E) 30 + 2 3 9. Diketahui segitiga ABC dengan panjang AC = 4 + 2 3 dan BAC = 30 . Titik D berada pada AB sedemikian sehingga CD tegak lurus dengan AB dan panjang BD = 1. Nilai sin ABC = .... (A) 3 + 2 3 (B) 2 + 3 1 (C) 1+ 3 2 1 (D) 2+ 6 4 2+ 6 (E)

(A) 3 (B) 4 (C) 1 7 (D) 4 (E)


11. Jika sin x = a, maka 1 2a2 3 + 1 a2 = ....

(A) sin2 x tan2 x (B) sec2 x cos x (C) cos 2x csc x (D) sec 2x tan2 x 1 + cot x (E) sin x cos x
12. Seorang siswa diminta untuk menyelesaikan persamaan x2 + bx + c = 0 , tetapi justru menyelesaikan persamaan x2 + cx + b = 0, b dan c bilangan bulat. Salah satu akar yang diperoleh adalah sama dengan akar dari persamaan semula, namun akar yang lain m kurangnya dari akar kedua persamaan semula. b dan c jika dinyatakan dalam m adalah ....

(A) b = (B) b = (C) b = (D) b = (E) b =

m 1 m1 , c= 2 2 m1 m1 , c= 2 2 m+1 m 1 , c= 2 2 m + 1 m1 , c= 2 2 m1 m+1 , c= 2 2

13. Sebuah akuarium memiliki dasar dan sisi-sisi yang berbentuk persegi panjang dan tidak memiliki tutup. Volume dari akuarium adalah 4 m3 . Lebar dari dasar akuarium adalah 1 m. Untuk pembuatan dasar akuarium diperlukan biaya sebesar Rp10.000,00 per m2 , sedangkan untuk sisi-sisinya diperlukan biaya sebesar Rp5.000,00 per m2 . Biaya minimal yang diperlukan untuk membuat sebuah akuarium adalah ....

(A) Rp20.000,00 (B) Rp40.000,00 (C) Rp50.000,00


c Universitas Indonesia
Downloaded from http://pak-anang.blogspot.com

(D) Rp60.000,00 (E) Rp80.000,00

Halaman 2 dari 12 halaman

Kode Naskah Soal:


14. Grak fungsi f (x) = mx2 + (n + 3)x + m + 2n + 4 memotong sumbu Y di titik A(0, 5) dan memotong sumbu X di titik B dan C . Jika absis titik tengah B dan C adalah 2 maka titik puncak grak tersebut adalah ....

206

Gunakan Petunjuk C dalam menjawab soal nomor 18 sampai nomor 20. 18. Nilai x yang memenuhi 23x log (x2 2x + 2) = 1 3 (2) 1 2 (3) 3 1 (4) 4 (1) 19.
2

log 4 adalah ....

(A) (2,9) (B) (9,2) (C) (2,11)

(D) (11,2) (E) (2,8)

15. Misalkan fungsi f : didenisikan dengan f (2x 3) = 4x2 + 2x 5 dan f adalah turunan pertama dari f . Hasil dari f (2x 3) = ....

(A) 2x 7 (B) 2x 1 (C) 2x + 7

(D) 4x + 1 (E) 8x + 2

16. Jika sistem: x + 2y = 3k 3x ky = 1 dan sistem: kx y = 1 x 2 + y = 1 mempunyai satu penyelesaian yang sama, maka hasil kali semua nilai k yang memenuhi adalah .... 3 2 1 (B) 2 (C) 1

(A)

Daerah yang diarsir adalah gambar himpunan penyelesaian pembatasan suatu soal program linier. Bentuk-bentuk di bawah ini yang mencapai maksimum di A adalah .... (1) 100x + 50y (2) 4x 4y (3) 3x + 3y (4) 8x + 2y 20. Diberikan sepasang persamaan 2x 3y = 13 dan 3x + 2y = b dengan 1 b 100, dan b bilangan bulat. Misalkan n2 = x + y , dengan x dan y adalah solusi dari persamaan di atas, yang berupa bilangan bulat, maka nilai n yang memenuhi adalah .... (1) 4 (2) 3 (3) 1 (4) 2

1 (D) 2 3 (E) 2 2 2 sin 4 17. Diketahui y (x) = 2 cos( x) cos 2 + sin( x) , cos 2 maka periode dari fungsi tersebut adalah ....

(A) 2 (B) (C) 2 (D) 2 (E) 4

c Universitas Indonesia

Downloaded from http://pak-anang.blogspot.com

Halaman 3 dari 12 halaman

Kode Naskah Soal:

206

BAHASA INDONESIA
Gunakan Petunjuk A dalam menjawab soal nomor 21 sampai nomor 40. 21. Kata berimbuhan yang dipakai pada konteks yang tidak tepat ditemukan dalam kalimat ... 23. Anda sering mengenakan earphone saat bekerja, di jalan sambil menunggu bus, atau menjelang tidur? Hati-hati, riset mengatakan bahwa penggunaan earphone bisa berbahaya untuk kesehatan telinga Anda. Berdasarkan sebuah penelitian baru-baru ini di Amerika Serikat, sekitar 5,2 juta anak berusia 619 tahun terganggu pendengarannya gara-gara terlalu sering terpapar musik keras. Salah satu biang keladinya adalah pemakaian earphone selama berjam-jam dengan volume keras. Para ahli kesehatan di sana memperkirakan anak-anak generasi "iPod" ini bakal lebih awal mengalami presbiakusis, yakni ketulian akibat usia lanjut, sekitar usia 40-an. Normalnya, kondisi ini baru muncul sekitar usia 6070 tahun. Pesan kutipan tersebut adalah ...

(A) Burung gereja beterbangan mendengar suara lonceng berdentang. (B) Anak-anak kejar-mengejar di halaman sekolah. (C) Para penonton berlari-larian turun ke tengah lapangan tatkala melihat api berkobar. (D) Amran dan Rio selalu surat-menyurat melalui e-mail. (E) Pemandangan indah terhalang karena lembah berselimutkan kabut.
22. Pemerintah harus menyadari riset bioteknologi tidak akan berjalan tanpa dukungan dana. Untuk meningkatkan hasil pertanian dan mencapai ketersediaan pangan yang cukup, pemerintah perlu memberikan bantuan dana untuk melakukan riset mengenai bioteknologi. Riset bioteknologi ini bukan hal yang murah. Pemerintah juga harus memberikan dana. Kalau dana yang ada terbatas, penelitian yang dilakukan juga jalan di tempat. Selain mendukung dalam masalah dana, pemerintah juga harus mendukung dalam peraturan. Peraturan pemerintah mengenai bioteknologi ini sudah ada, tetapi ada kendala, yaitu komisi yang seharusnya terbentuk dua tahun yang lalu sampai sekarang belum terbentuk. Pertanyaan berikut tidak sesuai dengan isi kutipan di atas, KECUALI ...

(A) Penggunaan earphone dapat mengganggu jam kerja. (B) Penggunaan earphone dapat berbahaya untuk kesehatan telinga. (C) Penggunaan earphone di jalan sambil menunggu bus sangat berbahaya. (D) Penggunaan earphone yang tidak terlalu sering tidak berbahaya. (E) Penggunaan earphone berjam-jam mengakibatkan ketulian di usia 6070 tahun.
24. Cara penulisan kata berikut sesuai dengan EYD, KECUALI ...

(A) Mengapa pemerintah perlu melakukan riset bioteknologi? (B) Apakah peraturan pemerintah mengenai bioteknologi sudah dilaksanakan? (C) Bagaimanakah cara meningkatkan hasil pertanian? (D) Kapankah sebaiknya melakukan riset bioteknologi? (E) Bagaimanakah peranan pemerintah dalam pelaksanaan riset bioteknologi?

(A) khazanah. (B) konkret. (C) karier.

(D) jadwal. (E) sejarahwan.

25. Makna meN-kan pada kalimat "Pemusik Jazz, Benny Likumahuwa, mempromosikan Ambon sebagai kota yang aman dan memiliki pesona" sama dengan makna meN-kan pada ...

(A) Penerbit memublikasikan buku baru Dewi Lestari. (B) Dengan terpaksa, dia membukakan pintu untuk pengamen itu. (C) Nilai rupiah saat ini sangat menggembirakan. (D) Yuanita memilih mematikan lampu. (E) Mereka menegaskan bahwa acara tersebut akan dihentikan.

c Universitas Indonesia

Downloaded from http://pak-anang.blogspot.com

Halaman 4 dari 12 halaman

Kode Naskah Soal:


26. Penulisan gabungan kata yang benar terdapat pada kalimat berikut ini, KECUALI ... 29. Kalimat yang tidak efektif adalah ...

206

(A) Kehebatan Cibaduyut sebagai sentra sepatu kulit terdengar sampai ke mancanegara. (B) Bus antarprovinsi JawaSumatra menjadi salah satu alat transportasi yang diminati penumpang. (C) AC Milan berhasil mencapai semi nal pada pertandingan musim panas yang lalu. (D) Budi daya mutiara sekarang dapat dilakukan di darat. (E) Para peserta karnaval menggunakan busana dan tata rias yang cerah.
27. Sejumlah kalimat berikut efektif, KECUALI ...

(A) Tindak kekejaman, kekerasan, dan menindas orang kecil merupakan perbuatan tidak terpuji. (B) Ketika saya datang, mereka sudah berkumpul di halaman sekolah untuk menanti inspektur upacara. (C) Tata tertib ini tidak boleh diubah sampai ada tata tertib baru yang disahkan oleh pimpinan. (D) Sebagai mahasiswa, Anda diharapkan dapat memberi keteladanan yang baik. (E) Diperlukan orang yang sanggup berpikir kritis dan tidak ekstrem.
30. Kalimat yang baik ditinjau dari segi ejaannya adalah ...

(A) Akan segera saya kirimkan surat ini ke alamat yang kauinginkan. (B) Pengembalian buku yang kamu pinjam tiga bulan yang lalu sudah saya terima. (C) Sudah dua minggu ini saya menunggu-nunggu surat balasan darinya. (D) Uang kuliah yang kamu butuhkan sudah saya transfer. (E) Dia akan membeberkan pesan rahasia dari istri gelapnya di persidangan Kamis depan.
28. Kalimat berikut yang efektif adalah ...

(A) Perkembangan agama Islam pada abad 16 ditandai oleh munculnya faham tasawuf. (B) "Kecermatan memilih kata sangat diperlukan dalam kegiatan mengarang," begitu nasihat Pak Is sewaktu saya belajar di Depok. (C) Karena belum ada yang dirubah, peraturan ini masih berlaku. (D) Kreatitas harus diselaraskan dengan esiensi dan efektitas. (E) Sikap atau pandangan yang terlalu ekstrim tidak semestinya mendapat tempat di sini.
31. Aku bagaikan kejatuhan bulan. Kata kejatuhan dalam kalimat di atas berstruktur sama dengan kata berimbuhan ke-an dalam kalimat berikut, KECUALI ...

(A) Jakarta tempatkan kesehatan sebagai prioritas. (B) Harga sembako ditetapkan pemerintah atau naik perlahan-lahan mendekati harga pasar. (C) Dalam menyampaikan laporan perlu bukti-bukti konkret. (D) Masuknya seluruh kekuatan politik dalam kekuasaan merupakan gejala negatif dalam demokrasi. (E) Dikarenakan persoalan sepele, terjadi pertengkaran yang melibatkan beberapa pemuda warga desa.

(A) Pak Dahlan kebakaran jenggot mendengar namanya disebut-sebut. (B) Tetangga saya tidak mengira kedatangan tamu tak diundang tadi malam. (C) Mata adikku kemasukan debu sehingga memerah. (D) Tiada yang risau akan kematian koruptor yang menyengsarakan negara. (E) Baru saja ia kepergian suaminya.

c Universitas Indonesia

Downloaded from http://pak-anang.blogspot.com

Halaman 5 dari 12 halaman

Kode Naskah Soal:


32. Saat berbicara di depan kader Partai Demokrat di Cikeas, Senin lalu, Presiden juga menyebutkan ada orang yang mencoba mengirim lamaran melalui ibu mertuanya. Inti kalimat dari kalimat di atas adalah ...

206

(A) Presiden berbicara. (B) Saat berbicara, Presiden menyebutkan ada orang. (C) Ada orang yang mencoba mengirim lamaran melalui ibu mertuanya. (D) Kader Partai Demokrat mencoba mengirim lamaran melalui ibu mertuanya. (E) Presiden menyebutkan ada orang.
33. Penelitian ini dilakukan terhadap tiga lm kartun Jepang, yaitu Sailormoon, Dragon Ball, dan Magic Knight Ray Earth, mengungkapkan bahwa 58,4% anak yang telah menonton tayangan tersebut menunjukkan sikap antisosial. Kalimat di atas menjadi efektif jika ...

35. (1) Bayi yang dilahirkan tidak dengan berat badan rendah, pada umumnya mempunyai status gizi saat lahir yang kurang lebih sama dengan status gizi bayi di Amerika. (2) Akan tetapi, seiring dengan bertambahnya umur ditambah faktor-faktor lainnya, sebagian besar bayi tersebut terus mengalami penurunan status gizi. (3) Puncak penurunan terjadi pada umur kurang lebih 1824 bulan. (4) Pada kelompok umur inilah prevalensi balita kurus (wasting) dan balita pendek (stunting) mencapai titik tertinggi (Hadi, 2001). (5) Setelah melewati umur 24 bulan, status gizi balita umumnya mengalami perbaikan meskipun tidak sempurna. Kalimat yang memiliki kesalahan ejaan adalah ...

(A) kalimat 5. (B) kalimat 4. (C) kalimat 3.

(D) kalimat 2. (E) kalimat 1.

(A) Kata ini diganti dengan yang. (B) Kata yang dihilangkan. (C) Kata mengungkapkan dihilangkan. (D) Kata terhadap diganti dengan pada. (E) Kata tersebut diganti dengan itu.
34. Kendati demikian, kebiasaan ini masih kurang efektif dibandingkan dengan cuci tangan menggunakan sabun. Hal ini karena sabun dapat meluruhkan lemak dan kotoran yang mengandung kuman. Dengan penggunaan yang benar, semua sabun memiliki efektivitas yang sama dalam meluruhkan kuman-kuman penyebab penyakit. Kalimat yang dapat mengisi bagian awal paragraf adalah ...

36. Situ Gunung adalah salah satu lokasi wisata tertua di Jawa Barat. Di zaman Belanda, para noni dan sinyo kerap berlibur di lokasi ini. Pada era keemasan kegiatan pecinta alam 1970-an hingga awal 1990, Situ Gunung merupakan salah satu tempat favorit berkemping. Letaknya yang tersembunyi di lembah di antara bukit-bukit itulah yang menarik perhatian orang. Gagasan pokok paragraf tersebut adalah ...

(A) Letak Situ Gunung tersembunyi. (B) Situ Gunung merupakan salah satu tempat favorit berkemping. (C) Para noni dan sinyo kerap berlibur di lokasi ini. (D) Keindahan alam Situ Gunung mengundang orang datang ke sana. (E) Situ Gunung adalah salah satu lokasi wisata tertua di Jawa Barat.

(A) Cuci tangan adalah kebiasaan yang sudah berlaku umum. (B) Kebiasaan cuci tangan sudah ada sejak masa dahulu. (C) Cuci tangan dengan menggunakan air telah menjadi hal yang umum. (D) Cuci tangan harus dibiasakan sejak masa kecil. (E) Kebiasaan cuci tangan berdampak positif bagi kesehatan.

c Universitas Indonesia

Downloaded from http://pak-anang.blogspot.com

Halaman 6 dari 12 halaman

Kode Naskah Soal:


37. Kemarau terjadi di berbagai teritori dengan mulai meningginya temperatur. Kecenderungan pengeringan dalam 30 tahun terakhir terlihat pada hutan-hutan tropis basah Afrika dan Amerika Selatan. Selain itu, pengeringan juga terjadi di wilayah-wilayah yang telah kering sebelumnya, seperti Eropa bagian selatan dan Amerika Utara bagian barat. Di daerah-daerah basah, penguapan terus muncul dalam curahan hujan yang deras. Hal ini meningkatkan risiko banjir. Hujan musim dingin menggantikan salju. Hal ini menimbulkan suatu pertanda buruk bagi ratusan juta orang yang bergantung pada lelehan salju di musim semi untuk pasokan air mereka. Pernyataan yang tidak sesuai dengan bacaan tersebut adalah ...

206

39. (1) Penularan demam berdarah (DBD) terjadi melalui gigitan nyamuk Aedes aegypti/Aedes albopictus betina. (2) Nyamuk ini sebelumnya telah membawa virus dalam tubuhnya dari penderita DBD lain. Nyamuk tersebut berasal dari Brazil dan Ethiopia dan sering menggigit manusia pada waktu pagi dan siang. (4) Orang yang berisiko terkena demam berdarah adalah anak-anak yang berusia di bawah 15 tahun. (5) Orang yang tinggal di lingkungan lembap serta daerah pinggiran kumuh juga berpotensi terkena demam berdarah. (6) Penyakit DBD sering terjadi di daerah tropis dan muncul pada musim penghujan. (7) Virus ini kemungkinan muncul akibat pengaruh musim/alam serta perilaku manusia. Kalimat yang tidak sesuai adalah ...

(A) Temperatur bumi menjadi tinggi, menyebabkan kekeringan di Afrika dan Amerika Selatan. (B) Sebagian Eropa dan Amerika mengalami kekeringan. (C) Daerah-daerah basah mulai menghadapi ancaman banjir. (D) Ancaman kekurangan air mulai membayangi daerah yang mengandalkan salju sebagai sumber air. (E) Salju di belahan bumi bagian selatan mulai menipis.
38. (1) Sesaat sebelum tintir kehabisan cahaya, Tamin masih dapat menikmati bentuk tubuh seorang perawan yang terlentang di atas tikar bawah, tidur pulas oleh kepayahan kerja sehari-hari. (2) Ia ingat kembali, masih kecil gadis itu ketika ia pergi. (3) Ia tidak pernah mengira, adiknya akan semanis itu akhirnya. (4) Alangkah larasnya garis-garis wajah itu dalam cahaya yang bertambah lemah. (5) Ia pernah menjelajahi tanah yang jauh; ia tidak pernah menemui gadis semanis Sumi. Makna kata laras kalimat (4) dalam konteks wacana tersebut adalah ...

(A) kalimat 3. (B) kalimat 4. (C) kalimat 5.

(D) kalimat 6. (E) kalimat 7.

(A) pantas (B) cocok (C) sesuai

(D) sepadan (E) serasi

c Universitas Indonesia

Downloaded from http://pak-anang.blogspot.com

Halaman 7 dari 12 halaman

Kode Naskah Soal:


40. Indonesia tengah menjajaki kemungkinan kerja sama di bidang bioteknologi dengan beberapa negara, di antaranya, Iran, Korea Selatan, Belanda, serta dengan sejumlah negara ASEAN. Kerja sama tersebut meliputi bidang pertanian, energi, dan kesehatan. "Negara-negara tersebut yang paling berminat untuk bekerja sama dengan peneliti kita," kata Ketua Konsorsium Bioteknologi Indonesia (KBI), Dr. Ir. Bambang Prasetyo, M.Sc. di sela Konferensi Internasional Bioteknologi di Bogor, Selasa (5/8). Belanda dan Korea Selatan ingin bekerja sama dalam bidang bioteknologi energi. Sementara, Iran tertarik bekerja sama dalam bidang bioteknologi kelautan. "Untuk bioteknologi bidang nonkelautan mereka sudah menguasai dan sekarang ingin bekerja sama dalam bidang kelautan," ujarnya. Pernyataan berikut tidak sesuai dengan maksud kalimat yang bercetak miring dalam kutipan di atas, KECUALI ...

206

(A) Tidak semua negara mau bekerja sama dalam bidang bioteknologi dengan peneliti Indonesia. (B) Negara Iran, Korea Selatan, Belanda mau bekerja sama dalam bidang pertanian, energi, dan kesehatan. (C) Negara Belanda dan Korea Selatan akan mengadakan kerja sama bioteknologi dalam bidang energi. (D) Negara Iran, Belanda, dan Korea Selatan berminat bekerja sama bioteknologi di bidang kelautan. (E) Negara Iran, Belanda, dan Korea Selatan berminat bekerja sama bioteknologi di bidang kelautan.

c Universitas Indonesia

Downloaded from http://pak-anang.blogspot.com

Halaman 8 dari 12 halaman

Kode Naskah Soal:

206

BAHASA INGGRIS
Insects, such as some moths and fruit ies, react to microbial infection in the same way as mammals; hence, they can be used to test the eciency of new drugs and to reduce the need for animal testing. Neutrophils, a type of white blood cell and part of the mammalian immune system, and haematocytes, cells that carry out a similar function in insects, react in the same way to infecting microbes. Both the insect and mammalian cells produce chemicals with a similar structure which move to the surface of the cells to kill the invading microbe. The immune cells then enclose the microbe and release enzymes to break it down. Insects such as fruit ies, Greater Wax Moths and a type of Hawkmoth can be used to test the ecacy of new antimicrobial drugs or to judge how virulent fungal pathogens are. It is now routine practice to use insect larvae to perform initial testing of new drugs and then to use mice for conrmation tests. Gunakan Petunjuk A dalam menjawab soal nomor 41 sampai nomor 42. 41. The topic of the paragraph is _______.

(A) using insects for drug safety tests (B) using fruit ies to check blood cells (C) the dangers of insect testing (D) the benets of moths (E) reducing animal testing
42. The sentence that best ends the paragraph is _______.

(A) Researchers will continue to explore the similarities between insect and mammalian responses so that insects can be used as models to study dierent disease states in humans (B) Dr. Kevin Kavanagh from the National University of Ireland presented his research ndings at the Society for General Microbiologys meeting at Heriot Watt University, Edinburgh (C) As well as reducing by up to 90% the number of mice required, this method of testing is quicker as tests with insects yield results in 48-72 hours whereas tests with mice usually take 4-6 weeks (D) Immune cells in insects and mammals are structurally and functionally similar despite being separated by over 400 million years of evolution (E) Some scientists, especially those who have never been involved in the experiment, nevertheless, refuse to believe that human reaction can be substituted by moth reaction

c Universitas Indonesia

Downloaded from http://pak-anang.blogspot.com

Halaman 9 dari 12 halaman

Kode Naskah Soal:

206

In a basement lab at Stanford University Medical School, Iret-net Hor-irws mummy lay tightly wrapped in tattered linen as a handful of scientists looked on. Starting with his feet, the scanner rotated around the mummy, snapping X-ray-type images that appeared on nearby computer screens. The pictures, showing well-preserved bone structure, were then mathematically manipulated to generate 3-D images that give a fuller picture of the skeleton. The highly sophisticated scanning technology allows scientists to learn about the 5-foot-4-inch (163-centimeter-tall) mummy in remarkable detail without doing invasive or damaging procedures. The digital images will show features that relate to paleopathology, diseases that may have been suered by the individual, also mummication style and patterns - how they may change through time. It will also be useful for teaching anatomy to everyone from small children through medical school. Iret-net Hor-irw is believed to be an ancient Egyptian priest, and his mummy belongs to the Fine Arts Museums of San Francisco. It was dug up from a cemetery in Akhmim, on the east bank of the Nile. Akhmim, from which the maternal relatives of the famous King Tut come, was an important provincial capital and the site of one of Egypts major temples. Scientists have not been able to pinpoint Iret-net Hor-irws age when he died or his cause of death. The scanning tests may help them get a little closer. For now, they can only date him to around 500 B.C., just before the Persian conquest, when the last native Egyptian dynasty ruled. It is one of the eras which is very poorly understood. After scientists are nished with him, Iret-net Hor-irws mummy will be the centerpiece of an exhibit at the Legion of Honor in San Francisco. The mummy has been out on loan from the Fine Arts Museums of San Francisco since 1944, and the exhibit, "Very Postmortem: Mummies and Medicine," is considered his homecoming. Gunakan Petunjuk A dalam menjawab soal nomor 43 sampai nomor 47. 43. An appropriate title for the text is _______. 46. Which of the following statements about the text is FALSE?

(A) Stanford Computer to Help Paleopathology (B) An Exhibition of an Egyptian Priests Mummy (C) New Technology to Find A Mummys Cause of Death (D) The Homecoming of The Fine Arts Museums Collection (E) Scanning Technology to Help Unwrap the Mystery of the Mummy
44. The word invasive in paragraph 2 can be best replaced by _______.

(A) The mummies bone structure is in ne condition. (B) The mummy can serve educational purposes. (C) The mummy was thought to be one of the Egyptian kings. (D) The mummy belongs to an American museum. (E) The scientists are able to tell the mummies medical history.
47. The text is most probably found in _______.

(A) a leaet of a museum exhibition (B) an article in a medical journal (C) a preface of an exhibition programme book (D) an article in a newspaper (E) a preface of a medical book

(A) consistent (B) surgical (C) serious (D) comprehensive (E) invading
45. What can be learnt from the computer images about the mummy?

(A) How long he had lived. (B) How he died. (C) His hierarchy in Egyptian temples. (D) How he was mummied. (E) How he had served King Tut.

c Universitas Indonesia

Downloaded from http://pak-anang.blogspot.com

Halaman 10 dari 12 halaman

Kode Naskah Soal:

206

A soak in a steamy tub, surrounded by candles sure seems like a great way to relax. But new research suggests that burning certain kinds of candles may ____(48)____ indoor air pollutants. Researchers at South Carolina State University studied petroleum-based and vegetable-source candles to determine their emissions. They let candles from dierent manufacturers burn for up to six hours in a small box, and then ____(49)____ and analyzed substances released into the air. They found that paran-based candles the most popular kind emitted toxic chemicals like toluene and benzene. Soybean and beeswax candles did not, according to the study. Lighting a paran candle once in a while is unlikely to pose a health threat, but frequently lighting many candles in an unventilated space could ____(50)____ to problems, and may aggravate asthma, cause allergy-like symptoms, or irritate the respiratory tract. ____(51)____, candles are not the only one potential source of indoor air pollution. Central heating and cooling systems, as well as gas replaces, can generate indoor air pollution, too. People with respiratory problems such as asthma and chronic ____(52)____ pulmonary disease should make an extra eort to cut down on indoor air pollution by ventilating their home as naturally as possible by opening the windows. Gunakan Petunjuk A dalam menjawab soal nomor 48 sampai nomor 52. 48. ....

(A) reduce (B) generate (C) increase


49. ....

(D) destroy (E) irritate

(A) collected (B) collection (C) collecting


50. ....

(D) collectable (E) collector

(A) refer (B) incline (C) indicate


51. ....

(D) lead (E) reect

(A) Hence (B) Moreover (C) Besides


52. ....

(D) Although (E) However

(A) obstructive (B) obstruct (C) obstructing (D) obstructively (E) obstructed

c Universitas Indonesia

Downloaded from http://pak-anang.blogspot.com

Halaman 11 dari 12 halaman

Kode Naskah Soal:


Gunakan Petunjuk A dalam menjawab soal nomor 53 sampai nomor 60. 53. "Will you please hurry, our school bus leaves in ve minutes." "_______, I wont nish in time."

206

57. Having seized some of the people related to the bombing of the J.W. Marriot and Ritz-Charlton hotels, _______.

(A) Why dont you hurry (B) Its about time (C) You go ahead (D) Youre pulling my leg (E) Just forget it
54. "The earthquake happening at about 3 oclock yesterday caused many people to panic." "Including me, as I _______ on the third-oor of my oce at that time."

(A) police investigation was continued to destroy the network of the terrorists (B) the terrorists network would be destroyed by the police (C) the police were determined to destroy the network of the terrorists (D) a continued investigation was conducted to destroy the terrorists network (E) destroying the tourists network was the polices main objective
58. Tchaikovskys music, _______ with immense technical skill and marked by emotional warmth, lyrical melody, and colorful orchestration, has long had wide appeal for the general public.

(A) worked (B) have worked (C) have been working (D) was working (E) had worked
55. _______ fund raising for helping the victims of earthquake succeeded in collecting large amount of money, many refugees still live in tents.

(A) was written (B) whose written (C) which it was written (D) written (E) it was written
59. "Did you receive our inquiry? When will we receive your conrmation?" "My apology. It seems that _______. Could you possibly resend it?"

(A) Because (B) As (C) Before

(D) Even though (E) Since

(A) we mislay your letter (B) your letter is mislaid (C) we had mislaid your letter (D) your letter has been mislaid (E) your letter was mislaid
60. Sharon would leave the key on the doorknob. What does the sentence mean?

56. "Wheres the report, Ben? You told me it would be ready by now. I need it for the board meeting this afternoon." "Dont worry, Jim. _______ before the board meeting."

(A) I have it nished (B) I have had it nished (C) I will have it nished (D) The report will nish (E) The report has nished

(A) There is a possibility that Sharon will leave the key on the doorknob. (B) Sharon will probably accidentally leave the key on the doorknob. (C) The speaker is certain that Sharon will leave the key on the doorknob. (D) It is typical of Sharon to leave the key on the doorknob. (E) Sharon left the key on the doorknob in the past.

c Universitas Indonesia

Downloaded from http://pak-anang.blogspot.com

Halaman 12 dari 12 halaman

SELEKSI MASUK UNIVERSITAS INDONESIA

SIMAK UI
KEMAMPUAN DASAR
Matematika Dasar Bahasa Indonesia Bahasa Inggris

207
Universitas Indonesia 2010

PETUNJUK UMUM
1. Sebelum mengerjakan ujian, periksalah terlebih dulu, jumlah soal dan nomor halaman yang terdapat pada naskah soal. Naskah soal ini terdiri dari 13 halaman. 2. Tulislah nomor peserta Anda pada lembar jawaban di tempat yang disediakan. 3. Tulislah kode naskah soal ini, pada lembar jawaban di tempat yang disediakan. Kode naskah soal ini: 7. Tulislah jawaban Anda pada lembar jawaban ujian yang disediakan. 8. Untuk keperluan coret-mencoret, harap menggunakan tempat yang kosong pada naskah soal ini dan jangan pernah menggunakan lembar jawaban karena akan mengakibatkan jawaban Anda tidak dapat terbaca. 9. Selama ujian, Anda tidak diperkenankan bertanya atau meminta penjelasan mengenai soal-soal yang diujikan kepada siapapun, termasuk kepada pengawas ujian. 10. Setelah ujian selesai, Anda diharapkan tetap duduk di tempat Anda sampai pengawas ujian datang ke tempat Anda untuk mengumpulkan lembar jawaban. 11. Perhatikan agar lembar jawaban ujian tidak kotor, tidak basah, tidak terlipat, dan tidak sobek.

207

4. Bacalah dengan cermat setiap petunjuk yang menjelaskan cara menjawab soal. 5. Pikirkanlah sebaik-baiknya sebelum menjawab tiap soal, karena setiap jawaban yang salah akan mengakibatkan pengurangan nilai (penilaian: benar +4, kosong 0, salah -1). 6. Jawablah lebih dulu soal-soal yang menurut Anda mudah, kemudian lanjutkan dengan menjawab soal-soal yang lebih sukar sehingga semua soal terjawab.

PETUNJUK KHUSUS
PETUNJUK A:
Pilih satu jawaban yang paling tepat.

PETUNJUK B:
Soal terdiri dari 3 bagian, yaitu PERNYATAAN, kata SEBAB, dan ALASAN yang disusun berurutan. Pilihlah: (A) Jika pernyataan benar, alasan benar, dan keduanya menunjukkan hubungan sebab dan akibat (B) Jika pernyataan benar, alasan benar, tetapi keduanya tidak menunjukkan hubungan sebab dan akibat (C) Jika pernyataan benar dan alasan salah (D) Jika pernyataan salah dan alasan benar (E) Jika pernyataan dan alasan keduanya salah

PETUNJUK C:
Pilihlah: (A) Jika (1), (2), dan (3) yang benar (B) Jika (1) dan (3) yang benar (C) Jika (2) dan (4) yang benar (D) Jika hanya (4) yang benar (E) Jika semuanya benar

Kode Naskah Soal:


MATA UJIAN TANGGAL UJIAN WAKTU JUMLAH SOAL Keterangan : : : : : Matematika Dasar, Bahasa Indonesia, dan Bahasa Inggris 11 APRIL 2010 120 MENIT 60 Mata Ujian MATEMATIKA DASAR nomor 1 sampai nomor 20 Mata Ujian BAHASA INDONESIA nomor 21 sampai nomor 40 Mata Ujian BAHASA INGGRIS nomor 41 sampai nomor 60

207

MATEMATIKA DASAR
Gunakan Petunjuk A dalam menjawab soal nomor 1 sampai nomor 18. 1. Nomor pegawai pada suatu pabrik terdiri atas tiga angka dengan angka pertama bukan nol. Banyaknya nomor pegawai yang ganjil adalah .... 5. Jika x + y + 2z = k , x + 2y + z = k dan 2x + y + z = k , k = 0, maka x2 + y 2 + z 2 jika dinyatakan dalam k adalah ....

(A) (B) (C) (D) (E)

(A) 648 (B) 475 (C) 450

(D) 425 (E) 324

2. A adalah himpunan penyelesaian persamaan 2 cos 3x = 1, dengan 0 x 2 . Banyaknya himpunan bagian A dengan 3 anggota adalah ....

(A) 20 (B) 15 (C) 6

(D) 3 (E) 1

k2 16 3k 2 16 4k 2 17 3k 2 8 2k 2 3

6. Banyaknya penyelesaian dari persamaan sin x sin 1 2 x = 1 cos x dengan 0 x 2 adalah ....

3. x1 dan x2 adalah bilangan bulat yang merupakan akar-akar persamaan kuadrat x2 (2p + 4)x + (3p + 4) = 0, di mana p adalah suatu konstanta. Jika x1 , p, x2 merupakan tiga suku pertama dari suatu deret geometri, maka suku ke-12 dari deret geometri tersebut adalah ....

(A) 0 (B) 1 (C) 2


7. Jika 81ab .45a+b = ....

(D) 3 (E) 4
135, maka nilai dari 7a b =

(A) 1 (B) 1 (C) 6 + 2 5 (D) 6 2 5 (E) 4


4. Banyaknya bilangan bulat negatif yang memenuhi pertidaksamaan (x + 1)(x2 + 2x 7) x2 1 adalah ....

(A) 1/2 (B) 1 (C) 3/2

(D) 2 (E) 3

(A) 0 (B) 1 (C) 2

(D) 3 (E) 4

8. Pada suatu penangkaran terdapat burung pipit dan burung dara. Ketika 5 burung pipit dilepaskan, jumlah burung dara dua kali burung pipit yang tersisa. Kemudian, ketika 25 ekor burung dara dilepaskan, burung pipit yang tersisa adalah 3 kali burung dara yang tersisa. Jumlah burung pipit semula adalah ....

(A) 20 (B) 25 (C) 30

(D) 35 (E) 40

c Universitas Indonesia

Downloaded from http://pak-anang.blogspot.com

Halaman 1 dari 13 halaman

Kode Naskah Soal:


9.

207

12. Jika jumlah 100 suku pertama suatu deret geometri adalah , dan jumlah 200 suku pertamanya adalah 3 , maka jumlah 700 suku pertamanya adalah ....

Luas segitiga pada gambar adalah .... cm2 (A) 4(1 3) (B) 4( 3 1) (C) 4( 3 + 1) (D) 2( 3 + 1) (E) 2(1 3) 10. Jumlah nilai x yang memenuhi persamaan x2 6 det(A) + det(B ) = 0 di mana A = 2 x+3 x2 6x + 8 3 dan B = adalah .... 4 1

13. Jika f (x) = x dan h(x) = 2x + 1, dan (f g h)(x) = 4x2 + 8x + 3 maka g (1) = ....

(A) 8 (B) 10 (C) 15

(D) 63 (E) 127

(A) 1 (B) 0 (C) 1

(D) 2 (E) 3

14. Jika p =3 log 4, maka nilai x yang memenuhi 2 2 persamaan 32x 7x+3 = 4x x6 apabila dinyatakan dalam p adalah ....

(A) (B)

1 + 2p 2p 1+
2 p

(A) 3 (B) 2 (C) 0 (D) 1 1 2 (E) 2 1 2


11. Sebuah perusahaan membuat dua buah produk (X dan Y ) dengan menggunakan dua buah mesin (A dan B ). Setiap unit X memerlukan 50 menit proses pada mesin A dan 30 menit proses pada mesin B . Setiap unit Y memerlukan 24 menit proses pada mesin A dan 33 menit proses pada mesin B . Pada kondisi awal, terdapat 30 unit X dan 90 unit Y di dalam gudang. Mesin A dapat digunakan maksimum 40 jam dan mesin B dapat digunakan 35 jam. Diprediksi akan ada permintaan 75 unit X dan 95 unit Y . Sistem pertidaksamaan linier yang mewakili situasi di atas adalah ....

2p 1 2p (C) 2p 1 + 2p (D) 2+p 1 + 2p (E) 1 2 p 15. Nilai minimum dari kuadrat jarak titik P (0, 3) ke Q yang terletak pada parabola y = 2x2 + 1 adalah ....

(A) 4 (B) (C) (D)


15 16 49 64 3 4

(E) 0
16. Diketahui sebuah barisan mempunyai urutan suku sebagai berikut: 120, 60, 40, 30, .... Suku berikutnya adalah ....

(A) 50X + 24Y 40(60); 30X + 33Y 35(60); x 0; y 5 (B) 50X + 24Y 40(60); 30X + 33Y 35(60); x 45; y 5 (C) 50X + 24Y 40; 30X + 33Y 35(60); x 0; y 5 (D) 50X + 24Y 40; 30X + 33Y 35; x 45; y 5 (E) 50X + 24Y 40(60); 30X + 33Y 35(60); x 0; y 0

(A) 24 (B) 25 (C) 30

(D) 35 (E) 36

c Universitas Indonesia

Downloaded from http://pak-anang.blogspot.com

Halaman 2 dari 13 halaman

Kode Naskah Soal:


1 1 0 17. Jika A = 0 1 0 , maka jumlah dari semua 0 0 1 elemen pada matriks A2010 adalah ....

207

(A) 2010 (B) 2011 (C) 2012

(D) 2013 (E) 2014

18. Dua buah dadu dilempar secara bersamaan. x adalah angka yang keluar dari dadu pertama, y adalah angka yang keluar dari dadu kedua. Jika A = {x, y |x + y < 2y < y + 2x, di mana sisa hasil bagi (x + y ) oleh 2 adalah 0}, maka nilai P (A)= ....

(A) (B) (C) (D) (E)

1 36 1 24 1 18 1 12 1 8

Gunakan Petunjuk C dalam menjawab soal nomor 19 sampai nomor 20. 19. Diberikan sepasang persamaan 2x 3y = 13 dan 3x + 2y = b dengan 1 b 100, dan b bilangan bulat. Misalkan n2 = x + y , dengan x dan y adalah solusi dari persamaan di atas, yang berupa bilangan bulat, maka nilai n yang memenuhi adalah .... (1) 4 (2) 3 (3) 1 (4) 2 ba 0 0 ab a tidak 20. Matriks A = 0 0 a a+b mempunyai invers jika memenuhi .... (1) a = 0, b = 0, dan a = b (2) a = 0, b = 0, dan a = b (3) a = 0 dan b sembarang (4) b = 0 dan a sembarang

c Universitas Indonesia

Downloaded from http://pak-anang.blogspot.com

Halaman 3 dari 13 halaman

Kode Naskah Soal:

207

BAHASA INDONESIA
Gunakan Petunjuk A dalam menjawab soal nomor 21 sampai nomor 40. 21. Michael Phelps, yang hiperaktif, selalu meminta perhatian, tidak bisa fokus, dan bermasalah sejak kecil, menjadi atlet terbesar Olimpiade sepanjang masa. Gagasan utama kalimat tersebut adalah ... 23. Miliaran burung dari belahan bumi utara berpindah dalam kelompok besar ke bagian selatan setiap musim dingin sebagai fenomena alamiah. Perpindahan tersebut dipicu oleh perubahan cuaca yang tajam dan berkurangnya makanan. Burung-burung tersebut mengembara dari tempat asal mereka di Eropa Utara, Asia Utara, bahkan Alaska menuju wilayah yang hangat, daerah tropika atau belahan bumi selatan. Akan tetapi, tidak semua jenis burung melakukan perjalanan panjang dan berbahaya ini. Ada sebagian burung yang tidak melanjutkan migrasinya setelah menempuh jalur yang singkat. Burung-burung tersebut memasuki Indonesia dari dua arah, yaitu utara dan selatan khatulistiwa. Dari bagian utara, burung-burung tersebut masuk melalui Semenanjung Malaya, Sumatera, dan biasanya langsung ke bagian timur serta dari Jepang, Taiwan lalu ke Filipina dan masuk wilayah Indonesia. Ringkasan dari kutipan tersebut adalah ...

(A) Michael Phelps bermasalah, namun menjadi atlet terbesar Olimpiade. (B) Michael Phelps hiperaktif, meminta perhatian, tidak bisa fokus, dan bermasalah. (C) Michael Phelps menjadi atlet terbesar Olimpiade. (D) Michael Phelps selalu meminta perhatian dan tidak bisa fokus. (E) Michael Phelps yang hiperaktif dan bermasalah.
22. Kalimat yang tidak efektif adalah ...

(A) Tindak kekejaman, kekerasan, dan menindas orang kecil merupakan perbuatan tidak terpuji. (B) Ketika saya datang, mereka sudah berkumpul di halaman sekolah untuk menanti inspektur upacara. (C) Tata tertib ini tidak boleh diubah sampai ada tata tertib baru yang disahkan oleh pimpinan. (D) Sebagai mahasiswa, Anda diharapkan dapat memberi keteladanan yang baik. (E) Diperlukan orang yang sanggup berpikir kritis dan tidak ekstrem.

(A) Miliaran burung dari belahan bumi utara berpindah dalam kelompok besar ke bagian selatan setiap musim dingin sebagai fenomena alamiah. Burung yang datang dari arah selatan berasal dari Australia. (B) Perpindahan burung dari belahan bumi utara merupakan sebuah fenomena alamiah. Burung-burung tersebut berpindah untuk mencari wilayah yang hangat. (C) Burung-burung tersebut memasuki Indonesia melalu dua arah, yaitu utara dan selatan. (D) Miliaran burung dari belahan bumi utara berpindah dalam kelompok besar ke bagian selatan setiap musim dingin sebagai fenomena alamiah. Burung-burung tersebut memasuki Indonesia dari dua arah, yaitu utara dan selatan khatulistiwa. (E) Tidak semua burung melakukan migrasi dengan jalur yang panjang dan berbahaya. Ada burung yang menyudahi migrasinya setelah menempuh jalur yang singkat.

c Universitas Indonesia

Downloaded from http://pak-anang.blogspot.com

Halaman 4 dari 13 halaman

Kode Naskah Soal:


24. Kiper nomor satu Qatar dapat menangkap bola dari penyerang Vietnam. Akan tetapi, tangkapannya lemah sehingga bola pun menyusur masuk ke dalam gawang sendiri. Pada penghujung babak pertama, sebenarnya Qatar memiliki peluang untuk menyamakan kedudukan melalui kaki Adel. Namun, tendangannya masih lemah. Tendangan itu sempat mengenai bek Vietnam sehingga hanya menghasilkan tendangan penjuru. Hingga babak pertama berakhir, kedudukan tetap tidak berubah. Usai turun minum, tempo permainan berjalan kian menarik. Penyerang Qatar kembali mengancam gawang Vietnam. Tendangan bebas dari Waleed Abdulla sangat terukur ke arah gawang, namun Duong lebih cepat dalam mengantipasi gerakan bola. Gagasan utama pada bacaan di atas adalah ...

207

26. Kalimat-kalimat berikut tidak efektif, KECUALI ...

(A) Dalam membangun kelompok tertinggal ini, biasanya banyak kendala sehingga perlu dirumuskan model pendekatan yang tepat. (B) Kajian ini masih berupa catatan sehingga masih perlu disempurnakan lagi baik mengenai materi dan cara pendekatannya. (C) Maka orang-orang desa itu jumlahnya tidak dihitung sesuai dengan jumlah mereka tetapi dihitung jumlah tenaganya yang digunakan untuk mengerjakan tanah. (D) Kontrol tenaga kerja pada bidang desa dan pertanian menuntut prasyarat terselenggaranya kontrol atas tanah. (E) Supermal Karawaci beroperasi setiap hari mulai pukul 09.00 sampai dengan pukul 22.00.
27. Kalimat luas berikut yang efektif adalah ...

(A) Penyerang Qatar sering mengancam gawang Vietnam. (B) Tendangan bebas penyerang Qatar mampu diantsipasi Duong. (C) Tendangan bebas Waleed Abdulla sangat terukur. (D) Tempo permainan berjalan kian menarik. (E) Duong lebih cepat mengantisipasi gerakan bola.
25. Ruly, seorang TKW di Taiwan asal Ponorogo, memberikan keterangannya lewat SMS bahwa gaji bersih yang diterimanya setiap bulan sebagai pekerja rumah tangga dengan bekal ijazah SMA adalah 10.000 dolar Taiwan (TWD) atau jika dirupiahkan senilai 2,5 juta rupiah. Gagasan utama kalimat di atas adalah ...

(A) Ruly seorang TKW di Taiwan asal Ponorogo. (B) Seorang TKW di Taiwan asal Ponorogo memberikan keterangan. (C) Ruly memberikan keterangannya. (D) Gaji bersih yang diterimanya setiap bulan. (E) Gaji bersih pekerja rumah tangga adalah 10.000 dolar Taiwan (TWD).

(A) Meskipun pernah tinggal di Indonesia pada tahun 19611963 untuk melakukan riset tentang minoritas Tionghoa, saya belum pernah menginjakkan kaki di kawasan Bangka Belitung, sebuah kawasan yang merupakan pemukiman etnik Tionghoa. (B) Saya pernah tinggal di Indonesia pada tahun 19611963 untuk melakukan riset tentang minoritas Tionghoa, tetapi saya belum pernah menginjakkan kaki di kawasan Bangka Belitung, yaitu kawasan pemukiman etnik Tionghoa. (C) Sebenarnya, saya pernah tinggal di Indonesia pada tahun 19611963 untuk melakukan riset tentang minoritas Tionghoa, tetapi saya belum pernah menginjakkan kaki di kawasan Bangka Belitung, padahal kawasan ini merupakan pemukiman etnik Tionghoa. (D) Meskipun pernah tinggal di Indonesia pada tahun 19611963 untuk melakukan riset tentang minoritas Tionghoa, tetapi saya belum pernah menginjakkan kaki di kawasan Bangka Belitung, kawasan yang merupakan pemukiman etnik Tionghoa. (E) Bahkan, saya pernah tinggal di Indonesia pada tahun 19611963 untuk melakukan riset tentang minoritas Tionghoa, tetapi saya belum pernah menginjakkan kaki di kawasan Bangka Belitung, kawasan yang merupakan pemukiman etnik Tionghoa.

c Universitas Indonesia

Downloaded from http://pak-anang.blogspot.com

Halaman 5 dari 13 halaman

Kode Naskah Soal:


28. Imron patuh kepada orang tuanya senakal-nakalnya ia. Bentuk se-nya pada kalimat berikut mempunyai fungsi yang sama dengan se-nya dalam kalimat ...

207

(A) Jawablah pertanyaan ini sesanggup-sanggupnya kamu. (B) Setinggi-tingginya bangau terbang, surutnya ke kubangan jua. (C) Agar tugas ini segera selesai, bekerja secepat-cepatnya yang engkau mampu. (D) Para siswa belajar serajin-rajinnya menjelang ujian seleksi masuk perguruan tinggi. (E) Jangan bersikap seenak-enaknya di sini.
29. Besarnya jumlah siswa/mahasiswa Indonesia di Singapura dan Malaysia tentu tidak lepas dari berbagai faktor. Pertama, siswa/mahasiswa Indonesia tidak akan mengalami kejutan budaya bila belajar di Singapura atau Malaysia. Dari segi geogras, jarak tempuh dari Indonesia ke Singapura dan Malaysia tidak terlalu jauh. Ketiga, cuaca di Singapura, Malaysia, dan Indonesia sama. Keempat, makanan yang ada di Malaysia, Singapura, dan Indonesia juga tidak jauh berbeda. Kalimat yang tepat untuk bagian awal paragraf adalah ...

30. Iklim sedang berubah lebih cepat daripada yang pernah terjadi sebelumnya. Kegiatan manusia adalah penyebab utamanya. ..., pembakaran bahan bakar fosil-minyak bumi, gas, batu bara-membanjiri atmosfer dengan karbon dioksida yang menjebak panas. Hal ini memicu kenaikan tajam pada temperatur global rata-rata dalam abad terakhir. Dampaknya adalah pada pola penguapan yang berubah, gletser-gletser yang meleleh, badai-badai yang menguat, dan kenaikan permukaan laut. Kata sambung yang diletakkan pada bagian yang kosong adalah ...

(A) akibatnya. (B) oleh karena itu. (C) misalnya. (D) akhirnya. (E) di sisi lain.
31. (1) Reduce berarti mengurangi atau mereduksi sampah yang akan terbentuk. (2) Hal ini dapat dilakukan apabila ibu-ibu rumah tangga kembali ke pola lama, yaitu membawa keranjang belanja ke pasar. (3) Dengan demikian, jumlah kantong plastik yang dibawa ke rumah akan berkurang (tereduksi). (4) Selain itu, bila setiap orang lebih memilih menggunakan saputangan daripada tisu, terjadi penghematan terhadap bahan baku untuk tisu. (5) Jika setiap orang melakukan hal tersebut, beberapa ton sampah yang akan tereduksi per bulan dan beberapa hasil hutan dapat diselamatkan. Kalimat topik pada paragraf tersebut adalah ...

(A) Negeri favorit mahasiswa Indonesia untuk studi adalah Singapura dan Malaysia. (B) Observasi menunjukkan bahwa jumlah siswa/mahasiswa Indonesia yang studi di Singapura dan Malaysia banyak. (C) Banyak mahasiswa Indonesia bercita-cita untuk belajar di Singapura dan Malaysia. (D) Kenyataan membuktikan bahwa Singapura dan Malaysia adalah dua negara yang diminati untuk tujuan studi. (E) Jumlah siswa/mahasiswa Indonesia di Singapura dan Malaysia menarik untuk diamati.

(A) Kalimat 5. (B) Kalimat 4. (C) Kalimat 3.

(D) Kalimat 2. (E) Kalimat 1.

32. Seorang ahli hukum tata negara menyebut hal ini sebagai bentuk "kudeta redaksional". Makna kata kudeta pada kalimat di atas adalah ...

(A) penggalangan kekuasaan. (B) penggulingan kekuasaan. (C) pengambilalihan kekuasaan. (D) perebutan kekuasaan dengan paksa. (E) pengakuan kedaulatan.

c Universitas Indonesia

Downloaded from http://pak-anang.blogspot.com

Halaman 6 dari 13 halaman

Kode Naskah Soal:


33. Penulisan gabungan kata berikut baku, KECUALI ...

207

(A) mitrabestari; matahati; tatabahasa. (B) bilamana; saputangan; darmabakti. (C) peribahasa; sumber daya; syahbandar. (D) halalbihalal; acapkali; citarasa. (E) dukacita; sukaria; tunarungu.
34. Penulisan kata dalam kalimat-kalimat berikut benar, KECUALI ...

37. Duvurger dalam bukunya berhasil memperjelas konik-konik teoretis antara Marxisme dengan Demokrasi Liberal. Ia berhasil menyoroti sesuatu yang selama berabad-abad ini menjadi isu intelektual dan praktis yang paling penting dalam bidang politik. Isu ini masih relevan dengan situasi politik yang ada pada sebagian negara di dunia. Arti ungkapan yang paling sesuai untuk kata relevan dalam paragraf di atas adalah ...

(A) Banyak pengguna komputer sudah memahami copy dan paste. (B) Ibu baru membeli baju bayi di toko My Baby. (C) Saat melakukan drag, teks yang dikopi akan berwarna menyala (highlight). (D) Tiras Harian Kompas paling tinggi. (E) Berikutnya, klik kanan di area yang sudah disorot itu.
35. Kata berimbuhan yang dipakai pada konteks yang tidak tepat ditemukan dalam kalimat ...

(A) sesuai. (B) cocok. (C) tepat.

(D) sepadan. (E) bergayut.

38. Tindakan itu sudah tentu menangguk reaksi keras dari semua kalangan. Kata yang bermakna sama dengan kata menangguk dalam kalimat di atas adalah ...

(A) menuai. (B) mengundang. (C) memanen.

(D) memperoleh. (E) memancing.

(A) Burung gereja beterbangan mendengar suara lonceng berdentang. (B) Anak-anak kejar-mengejar di halaman sekolah. (C) Para penonton berlari-larian turun ke tengah lapangan tatkala melihat api berkobar. (D) Amran dan Rio selalu surat-menyurat melalui e-mail. (E) Pemandangan indah terhalang karena lembah berselimutkan kabut.
36. Pemakaian tanda garis miring (/) berikut ini sesuai dengan EYD, KECUALI ...

39. Cara penulisan kata berikut sesuai dengan EYD, KECUALI ...

(A) khazanah. (B) konkret. (C) karier.

(D) jadwal. (E) sejarahwan.

(A) Tahun akademik 2010/2011 akan segera kita masuki. (B) Acara akan berlangsung dari pukul 13.00 s/d 14.30. (C) Nomor surat keterangan tersebut 21 PFL/XI/2008. (D) Honornya Rp850.000,00/minggu. (E) Jalan Kamboja X/4.

c Universitas Indonesia

Downloaded from http://pak-anang.blogspot.com

Halaman 7 dari 13 halaman

Kode Naskah Soal:


40. Vietnam gagal mencatatkan diri sebagai tim pertama yang lolos ke babak perempat nal setelah pada pertandingan kedua dipaksa bermain imbang 1-1 oleh Qatar di Stadion My Dinh Hanoi. Vietnam yang memiliki nilai sempurna setelah mengalahkan Uni Emirat Arab 2-0 di partai perdana harus tertekan oleh Qatar sejak 10 menit paruh pertama. Pada menit ke-20, penyerang naturalisasi Qatar, Sebastian Andres Quintana, memiliki peluang emas untuk mencetak gol pembuka. Namun sayangnya, tendangan itu masih dapat dibaca dengan baik oleh Duong Hong Son. Delapan menit kemudian, Quintana kembali menjadi momok bagi lini pertahanan Vietnam. Meskipun selalu diserang, justru Vietnam yang mampu membuka keunggulan terlebih dulu. Mereka mencetak gol 10 menit kemudian. Pernyataan yang sesuai dengan bacaan tersebut adalah ...

207

(A) Vietnam dikalahkan Uni Emirat Arab 2-0 sehingga harus mengalahkan Qatar. (B) Pertandingan antara Vietnam dan Qatar berakhir seri. (C) Vietnam terus menyerang Qatar dalam pertandingan tersebut. (D) Qatar berhasil mencetak gol lebih dahulu daripada Vietnam. (E) Qatar lebih banyak menyerang Vietnam.

c Universitas Indonesia

Downloaded from http://pak-anang.blogspot.com

Halaman 8 dari 13 halaman

Kode Naskah Soal:

207

BAHASA INGGRIS
You have invited your friend over for dinner. Your child sees your friend reach for some cookies and says, "Better not take those, or youll get even bigger." Youre embarrassed that your child could speak so rudely. However, you should consider that your child may not know how to use language appropriately in social situations and did not mean harm by the comment. An individual may say words clearly and use long, complex sentences with correct grammar, but still have a communication problem if he or she has not mastered the rules for social language known as pragmatics. Adults may also have diculty with pragmatics, for example, as a result of a brain injury or stroke. An individual with pragmatic problems may say inappropriate or unrelated things during conversations, tell stories in a disorganized way, or have little variety in language use. It is not unusual for children to have pragmatic problems in only a few situations. However, if problems in social language use occur often and seem inappropriate considering the childs age, a pragmatic disorder may exist. Pragmatic disorders often coexist with other language problems such as vocabulary development or grammar. Gunakan Petunjuk A dalam menjawab soal nomor 41 sampai nomor 42. 41. The topic of this paragraph is _______.

(A) pragmatic disorder (B) pragmatic aspect of language (C) pragmatics, logic and semantics (D) children with pragmatics diculties (E) what causes pragmatics problems
42. The paragraph which follows this one most likely discusses _______.

(A) pragmatic language tips (B) how to test social pragmatic skills (C) pragmatic disorder and language inhibition (D) semantic-pragmatic disorder in children (E) pragmatic impairment behavioral problems
and associated

c Universitas Indonesia

Downloaded from http://pak-anang.blogspot.com

Halaman 9 dari 13 halaman

Kode Naskah Soal:

207

The thought of a car or truck that can drive itself is at once both exciting and frightening. Autonomous vehicle navigation, as the technology is known, may make life more convenient if it allows people to kick back and enjoy a good book or movie while their cars guide themselves through rush-hour trac. However, what happens if it starts to rain or if trac suddenly picks up? If the technology is to work at all, it will have to be completely safe on all roads, under all speeds, and in all weather. Therein lies the challenge: if cars and trucks are to drive autonomously, they will need futuristic sensors and advanced computing capabilities to respond to ever-changing road conditions. Perhaps the most extreme example of ever-changing conditions is a war zone, where roads may be reduced to rubble and vehicles are natural targets of attack. Rolling out eets of self-navigating vehicles for the military is an enticing idea because it could keep thousands of troops out of harms way. Nevertheless, will it be possible for these vehicles to operate in war zones? This question was the inspiration for a recent Defense Advanced Research Projects Agency (DARPA) contest aimed at spurring the development of such technologies. Held at a former air force base in Victorville, Calif. in late 2007, the DARPA Urban Challenge oered a $3.5 million purse to competitors who could design the fastest and safest vehicles that could traverse a 60-mile urban course in moving trac in less than six hours. The contestant vehicles were unmanned and had to complete a simulated military supply mission, maneuvering through a mock city environment, avoiding obstacles, merging into moving trac, navigating trac circles, and negotiating intersections - all while conforming to California driving rules. Of the 89 international teams that entered the challenge, only six nished in the allotted time. Wende Zhang of General Motors was part of the team that designed the winning vehicle, which nished with the fastest time - an average speed of approximately 13 miles per hour. The GM team drew upon existing technology already oered in some of their vehicles that can assist in parking or detect lane markers and trigger alarms if the drivers are coming too close to the shoulder of the road. For the DARPA challenge, they developed a more sophisticated package of sensors that included GPS coupled with a camera and a laser-ranging LIDAR system to guide and correct the vehicles route through the city. In Baltimore, Zhang will present GMs patented new methods for detecting lanes and correcting a vehicles route, which helped them win the challenge. Though they won, people should not look for robotic chaueurs immediately. The technology must prove reliable in many dierent road, weather and lighting conditions. Still, says Zhang, a commercially-viable autonomous driving product may be available in the next decade. Gunakan Petunjuk A dalam menjawab soal nomor 43 sampai nomor 47. 43. The best title for this passage is _______. 44. What is stated about autonomous vehicle navigation in the passage?

(A) It cannot function if it runs out of electricity. (B) Bad weather and trac jam are minor problems for autonomous vehicles. (C) Sixty teams managed to design autonomous vehicles in the DARPA competition. (D) Autonomous vehicles will be equipped with a state-of-the art navigating system. (E) Robotic chaueurs will be ready in the near future.
45. It can be inferred from the passage that the DARPA Urban Challenge _______.

(A) Future War Zone Vehicles (B) A Giant Leap in automotive industry (C) Research on the development of unmanned vehicles (D) Defense Advanced Research Projects Agency (E) The Triumph competition
of General Motors in a

(A) meant to search for an advanced manned navigation system (B) was only open for American teams (C) gave a trivial prize to the winning team (D) is to develop self-navigating vehicles for all sorts of road conditions (E) was not very competitive

c Universitas Indonesia

Downloaded from http://pak-anang.blogspot.com

Halaman 10 dari 13 halaman

Kode Naskah Soal:


46. The word "obstacles" in paragraph 4 is closest in meaning to which of the following?

207

(A) Problems (B) Diculties (C) Obstructions

(D) Burdens (E) Nuisances

47. Which of the following statements is FALSE?

(A) Futuristic sensors and advanced computing abilities will be assential feature of autonomous vehicles. (B) Using self-navigating vehicles in the army will reduce the number of injured soldiers in wars. (C) The DARPA Urban Challenge looked for the fastest and safest autonomous vehicle. (D) The GM teams new detecting and navigating methods played an important role in its victory. (E) The U.S. Army is already well-prepared and ready to launch unmanned vehicles in current wars.

c Universitas Indonesia

Downloaded from http://pak-anang.blogspot.com

Halaman 11 dari 13 halaman

Kode Naskah Soal:

207

Doctors have recently started to study the causes of a medical disorder which they have appropriately named SAD, or seasonal aective disorder. People who suer from SAD become very ____(48)____ during the winter months. Their depression appears to be the result of a decrease in the amount of sunlight they are exposed to. Doctors theorize that decreased sunlight aects the ____(49)____ of melatonin, a hormone manufactured in the brain, and serotonin, a chemical that helps transmit nerve impulses. Depression may result from the ensuing imbalance of these two substances in the body. ____(50)____, doctors believe that a decrease in the amount of sunlight the body receives may cause a ____(51)____ in the bodys natural clock, which could, in turn, result in symptoms such as listlessness, oversleeping, weight gain, anxiety, and irritability all symptoms of depression. ____(52)____ absence of light seems to be the cause of this disorder, a daily dose of light appears to be the cure. Doctors advise patients to sit in front of a special light box which simulates natural light for a few hours every day. Gunakan Petunjuk A dalam menjawab soal nomor 48 sampai nomor 52. 48. ....

(A) depressed (B) clear (C) doubtful


49. ....

(D) mad (E) anxious

(A) produce (B) production (C) productive


50. ....

(D) productively (E) produced

(A) Therefore (B) However (C) Despite


51. ....

(D) In addition (E) Thus

(A) disturbance (B) danger (C) result


52. ....

(D) disease (E) decrease

(A) In fact (B) When (C) Besides

(D) Since (E) Moreover

c Universitas Indonesia

Downloaded from http://pak-anang.blogspot.com

Halaman 12 dari 13 halaman

Kode Naskah Soal:


Gunakan Petunjuk A dalam menjawab soal nomor 53 sampai nomor 60. 53. "Id like to nish entering the data today but I have to catch the twelve-oclock train to Bandung." "_______ ."

207

57. "Did you receive our inquiry? When will we receive your conrmation?" "My apology. It seems that _______. Could you possibly resend it?"

(A) we mislay your letter (B) your letter is mislaid (C) we had mislaid your letter (D) your letter has been mislaid (E) your letter was mislaid
58. ______ a new policy related to the establishment of business was released, entrepreneurs had found it dicult to get a license for their new businesses.

(A) You have it done (B) Intan has done it (C) Have you done it? (D) It has to be done (E) Have Intan do it?
54. "The earthquake happening at about 3 oclock yesterday caused many people to panic." "Including me, as I _______ on the third-oor of my oce at that time."

(A) worked (B) have worked (C) have been working (D) was working (E) had worked
55. "Wheres the report, Ben? You told me it would be ready by now. I need it for the board meeting this afternoon." "Dont worry, Jim. _______ before the board meeting."

(A) When (B) After (C) Since

(D) Until (E) While

59. "Excuse me. Could you tell me which bus I should take to get to Plaza Indonesia?" "Sorry, I dont know. You _______ ask the driver."

(A) could have (B) had better (C) ought to have (D) must have (E) would rather
60. By the time Mr. Arin retires, his youngest child will have graduated from university. This means that Mr. Arin _______ when his youngest son graduates.

(A) I have it nished (B) I have had it nished (C) I will have it nished (D) The report will nish (E) The report has nished
56. Last summer, he visited Nasau, a small but vibrant city, with a great range of designer outlets ________ luxury goods at tax-free prices.

(A) will still work (B) will retire (C) doesnt want to retire (D) wants to work (E) doesnt want to work

(A) that sells (B) sold (C) they sold

(D) selling (E) to sell

c Universitas Indonesia

Downloaded from http://pak-anang.blogspot.com

Halaman 13 dari 13 halaman

SELEKSI MASUK UNIVERSITAS INDONESIA

SIMAK UI
KEMAMPUAN DASAR
Matematika Dasar Bahasa Indonesia Bahasa Inggris

208
Universitas Indonesia 2010

PETUNJUK UMUM
1. Sebelum mengerjakan ujian, periksalah terlebih dulu, jumlah soal dan nomor halaman yang terdapat pada naskah soal. Naskah soal ini terdiri dari 12 halaman. 2. Tulislah nomor peserta Anda pada lembar jawaban di tempat yang disediakan. 3. Tulislah kode naskah soal ini, pada lembar jawaban di tempat yang disediakan. Kode naskah soal ini: 7. Tulislah jawaban Anda pada lembar jawaban ujian yang disediakan. 8. Untuk keperluan coret-mencoret, harap menggunakan tempat yang kosong pada naskah soal ini dan jangan pernah menggunakan lembar jawaban karena akan mengakibatkan jawaban Anda tidak dapat terbaca. 9. Selama ujian, Anda tidak diperkenankan bertanya atau meminta penjelasan mengenai soal-soal yang diujikan kepada siapapun, termasuk kepada pengawas ujian. 10. Setelah ujian selesai, Anda diharapkan tetap duduk di tempat Anda sampai pengawas ujian datang ke tempat Anda untuk mengumpulkan lembar jawaban. 11. Perhatikan agar lembar jawaban ujian tidak kotor, tidak basah, tidak terlipat, dan tidak sobek.

208

4. Bacalah dengan cermat setiap petunjuk yang menjelaskan cara menjawab soal. 5. Pikirkanlah sebaik-baiknya sebelum menjawab tiap soal, karena setiap jawaban yang salah akan mengakibatkan pengurangan nilai (penilaian: benar +4, kosong 0, salah -1). 6. Jawablah lebih dulu soal-soal yang menurut Anda mudah, kemudian lanjutkan dengan menjawab soal-soal yang lebih sukar sehingga semua soal terjawab.

PETUNJUK KHUSUS
PETUNJUK A:
Pilih satu jawaban yang paling tepat.

PETUNJUK B:
Soal terdiri dari 3 bagian, yaitu PERNYATAAN, kata SEBAB, dan ALASAN yang disusun berurutan. Pilihlah: (A) Jika pernyataan benar, alasan benar, dan keduanya menunjukkan hubungan sebab dan akibat (B) Jika pernyataan benar, alasan benar, tetapi keduanya tidak menunjukkan hubungan sebab dan akibat (C) Jika pernyataan benar dan alasan salah (D) Jika pernyataan salah dan alasan benar (E) Jika pernyataan dan alasan keduanya salah

PETUNJUK C:
Pilihlah: (A) Jika (1), (2), dan (3) yang benar (B) Jika (1) dan (3) yang benar (C) Jika (2) dan (4) yang benar (D) Jika hanya (4) yang benar (E) Jika semuanya benar

Kode Naskah Soal:


MATA UJIAN TANGGAL UJIAN WAKTU JUMLAH SOAL Keterangan : : : : : Matematika Dasar, Bahasa Indonesia, dan Bahasa Inggris 11 APRIL 2010 120 MENIT 60 Mata Ujian MATEMATIKA DASAR nomor 1 sampai nomor 20 Mata Ujian BAHASA INDONESIA nomor 21 sampai nomor 40 Mata Ujian BAHASA INGGRIS nomor 41 sampai nomor 60

208

MATEMATIKA DASAR
Gunakan Petunjuk A dalam menjawab soal nomor 1 sampai nomor 17. 1. Peluang Kris mendapat nilai A untuk matematika adalah 0,6 dan untuk bahasa Inggris 0,7. Peluang Kris hanya mendapatkan satu nilai A adalah .... 5.

(A) 0,12 (B) 0,18 (C) 0,28

(D) 0,42 (E) 0,46


Luas segitiga pada gambar adalah .... cm2 (A) 4(1 3) (B) 4( 3 1) (C) 4( 3 + 1) (D) 2( 3 + 1) (E) 2(1 3) 6. Diketahui A = 1 2 50 105 , maka det(A3 ) = ....

2. Jika (g f )(x) = 9x2 6x dan g (x) = x2 + 1, maka f (2x + 3)= ....

(A) 6x + 4 (B) 6x + 10 (C) 2x + 4

(D) 2x + 1 (E) 3x + 1

3. x1 dan x2 adalah bilangan bulat yang merupakan akar-akar persamaan kuadrat x2 (2p + 4)x + (3p + 4) = 0, di mana p adalah suatu konstanta. Jika x1 , p, x2 merupakan tiga suku pertama dari suatu deret geometri, maka suku ke-12 dari deret geometri tersebut adalah ....

(A) 125 (B) 25 (C) 5

(D) 25 (E) 125


3x2 7x+8 x2 +1

(A) 1 (B) 1
(C) 6 + 2 5 (D) 6 2 5

7. Himpunan penyelesaian dari 1 < adalah ....

(A) {x |x 2 atau x 3} (B) {x |2 x 3} (C) {x |x 1 atau x 6} (D) {x |1 x 6} (E) {x }

(E) 4
4. A dan B pergi menonton konser musik di suatu stadion yang mempunyai 8 pintu. Mereka masuk dari pintu yang sama, tetapi keluar dari pintu yang berbeda. Banyaknya cara yang dapat mereka lakukan adalah ....

(A) 28 (B) 224 (C) 448

(D) 484 (E) 896

c Universitas Indonesia

Downloaded from http://pak-anang.blogspot.com

Halaman 1 dari 12 halaman

Kode Naskah Soal:


8. Jika x + y + 2z = k , x + 2y + z = k dan 2x + y + z = k , k = 0, maka x2 + y 2 + z 2 jika dinyatakan dalam k adalah ....
2 3

208

12. Jika diketahui a log b + (a log b) + (a log b) + ... = 2, 3 a b maka log b + log a2 = ....

(A) (B) (C) (D) (E)

k2 16 3k 2 16 4k 2 17 3k 2 8 2k 2 3

(A) 1 (B) (C)


3 2 5 3

(D) 2 (E) 3
13. Diketahui (f g )(x) = x2 4x + 2 dan g (x) = x 3. Jika x1 dan x2 adalah nilai-nilai yang memenuhi f (x) = 2, maka nilai x1 + x2 adalah ....

9. Diketahui x1 dan x2 adalah akar-akar persamaan x1 + 1 x2 + 1 2x2 + 6x + a = 0. Jika + < 2, maka x2 x1 nilai a yang memenuhi adalah ....

(A) 4 (B) 2 (C) 2

(D) 4 (E) 10

(A) a < 0 (B) a > 0 (C) a < 3


10. Jika a log 81b = dari
1 (a) 3a b 1 2

(D) a > 3 (E) a > 12


log 27a dengan a, b > 0, maka nilai
1

14. Sebuah kotak obat tanpa tutup alasnya berbentuk persegi dan mempunyai volume 4000 cm3 . Luas permukaan kotak obat minimum adalah ....

(A) 1800 cm2 (B) 1240 cm2 (C) 1200 cm2 (D) 1100 cm2 (E) 1000 cm2
15. Diketahui sebuah barisan 2, 3, 4, 6, 6, 6, 10, 9, 8, 14, 12, 10,.... Jumlah 3n suku pertama, untuk n = 1, 2, 3, 4,...., dari barisan di atas adalah ....

log (b) b

adalah ....

(A) (B) (C) (D) (E)

2a 3b 3a 4b a b 3a 2b 4a 3b

(A) S3n = n 2 (9 9n) (B) S3n = n 2 (9 + 9n) (C) S3n = (D) S3n = (E) S3n =
3n 2 (9 + 9n) n 6 (9 + 9n) n 2 (3 + 3n)

11. Jika a dan b adalah bilangan riil dengan 0 < a < b ab dan a2 + b2 = 8ab, maka = .... a+b (A) 1 3 15 (B) 1 5 15 (C) 1 6 10 (D) 1 5 15 (E) 1 3 15

16. y = sin(sin(sin(sin .... sin(sin(x))....))) Tentukan pada x = 0.

dy dx

(A) (B) 1 (C) 0

(D) 1 (E)

c Universitas Indonesia

Downloaded from http://pak-anang.blogspot.com

Halaman 2 dari 12 halaman

Kode Naskah Soal:


17. Diketahui fungsi kuadrat f (x) = x2 4x + 5. Dua buah garis singgung di titik yang merupakan perpotongan antara f (x) dan garis y = 5 membentuk sebuah segitiga dengan garis y = 5. Maka titik potong kedua garis singgung tersebut adalah ....

208

(A) (3,2) (B) (2,3) (C) (2,3)

(D) (3,2) (E) (3,2)

Gunakan Petunjuk C dalam menjawab soal nomor 18 sampai nomor 20. 18. Pernyataan berikut yang BENAR berkaitan dengan 1 1 + + (2 + .... adalah deret 1 + 21 3 (2 3)2 3)3 (1) deret tersebut merupakan deret geometri dengan rasio 2 + 3 (2) deret tersebut merupakan deret turun (4) jumlah semua sukunya sama dengan (3) suku ke-5 nilainya sama dengan 97 + 56 3
3 2

19. Suatu papan segiempat dibuat dengan spesikasi sebagai berikut: 12 dm keliling < 20 dm dan rasio dari sisi-sisi yang berdekatan (r) adalah 1 < r < 2. Jika ukuran papan diinginkan dalam bilangan bulat, maka luas papan yang mungkin adalah .... (1) 12 (2) 15 (3) 20 (4) 16 20. Diberikan sepasang persamaan 2x 3y = 13 dan 3x + 2y = b dengan 1 b 100, dan b bilangan bulat. Misalkan n2 = x + y , dengan x dan y adalah solusi dari persamaan di atas, yang berupa bilangan bulat, maka nilai n yang memenuhi adalah .... (1) 4 (2) 3 (3) 1 (4) 2

c Universitas Indonesia

Downloaded from http://pak-anang.blogspot.com

Halaman 3 dari 12 halaman

Kode Naskah Soal:

208

BAHASA INDONESIA
Gunakan Petunjuk A dalam menjawab soal nomor 21 sampai nomor 40. 21. Tahukah Anda, Amerika Serikat dapat mengalahkan Jepang dalam Perang Asia Timur Raya bukan karena bom nuklir yang menghanguskan Kota Hiroshima dan Nagasaki, tetapi karena bisa memecahkan kode sandi rahasia pesan-pesan yang digunakan oleh Jepang? Ya, bom nuklir hanya senjata pamungkas untuk perang itu. Namun, titik balik kemenangan justru karena Unit Kombat Intelijen AS dapat memecahkan sandi rahasia pesan yang digunakan oleh tentara Jepang. Sejak itu, segala kekuatan pasukan, gerakan, tipu daya, dan strategi Jepang diketahui persis oleh AS. Paragraf di atas membicarakan ... 23. Sarkofagus adalah sejenis peti kubur dari batu utuh yang terdiri atas wadah dan tutupnya. Jenis makam tradisi megalitik ini bisa digunakan untuk penguburan primer (langsung) atau sekunder (dengan meletakkan tulang-belulang si mati), penguburan tunggal atau ganda (satu kubur dipakai bersama-sama). Sarkofagus banyak ditemukan di Sumatra Utara, mulai dari Parapak sampai ke Dolok Sanggul di tepian Danau Toba. Ciri sarkofagus di daerah ini adalah bentuk dasarnya yang melengkung, yang menyerupai perahu, yang mirip dengan atap rumah di Sumatra Utara. Paragraf di atas menjelaskan ...

(A) AS dapat mengalahkan Jepang dalam Perang Asia Timur Raya. (B) AS menggunakan bom nuklir untuk menghancurkan Kota Hiroshima dan Nagasaki. (C) AS berhasil memecahkan sandi rahasia pesan yang digunakan tentara Jepang dalam Perang Asia Timur Raya. (D) Titik balik kemenangan AS atas Jepang saat Perang Asia Timur Raya. (E) AS mengetahui kekuatan dan strategi Jepang setelah dapat memecahkan sandi rahasia pesan tentara Jepang.
22. 4 Februari 2004, Mark Zuckerberglulusan Harvard dan mantan murid Ardsley High Schoolmeluncurkan Facebook. Situs jejaring sosial itu dengan cepat diminati penduduk dunia. Kesuksesan situs itu menarik masyarakat dan membuat sejumlah pemilik perusahaan teknologi ingin memilikinya. Facebook itu juga yang membuat Mark, pria kelahiran New York 14 Mei 1984 itu, menjadi orang kaya baru di Amerika Serikat. Kutipan di atas membicarakan ...

(A) sarkofagus sebagai makam yang berasal dari tradisi megalitik. (B) fungsi sarkofagus sebagai makam tradisi megalitik. (C) sarkofagus sebagai peninggalan dari tradisi megalitik. (D) keberadaan sarkofagus di Sumatra Utara. (E) ciri sarkofagus yang berada di Sumatra Utara.
24. Kalimat yang tidak efektif adalah ...

(A) Tindak kekejaman, kekerasan, dan menindas orang kecil merupakan perbuatan tidak terpuji. (B) Ketika saya datang, mereka sudah berkumpul di halaman sekolah untuk menanti inspektur upacara. (C) Tata tertib ini tidak boleh diubah sampai ada tata tertib baru yang disahkan oleh pimpinan. (D) Sebagai mahasiswa, Anda diharapkan dapat memberi keteladanan yang baik. (E) Diperlukan orang yang sanggup berpikir kritis dan tidak ekstrem.

(A) Mark Zuckerberg meluncurkan Facebook. (B) Facebook membuat Mark Zuckerberg menjadi orang kaya baru di Amerika Serikat. (C) Mark Zuckerberg sebagai seorang lulusan dari Harvad. (D) Facebook diminati oleh semua orang. (E) Sejumlah pemilik perusahaan Facebook ingin memiliki situs tersebut.

c Universitas Indonesia

Downloaded from http://pak-anang.blogspot.com

Halaman 4 dari 12 halaman

Kode Naskah Soal:


25. Penulisan huruf kapital yang benar terdapat pada kalimat ...

208

28. Penulisan tanda baca yang benar terdapat pada kalimat ...

(A) Tidak ada yang dapat membantah kebenaran kata-kata Bu Lurah sebab memang yang disampaikannya benar. (B) Ya, Tuhan, atas karuniaMu-lah aku dapat bertahan di tempat yang gersang ini. (C) Salah satu hari penting bagi orang Bali yang umumnya beragama Hindu adalah Hari Galungan. (D) Bagi orang Jawa, khususnya yang suka masak, gula Jawa merupakan barang penting seperti halnya bawang putih, bawang merah, dan bumbu-bumbu lainnya. (E) Bagi orang Jawa, R. Ng. Ranggawarsita bukan seorang sastrawan biasa, melainkan Pujangga yang dianggap Waskita.
26. Hal ini disebabkan karena banyak orang memilih kereta rel listrik sebagai sarana transportasinya. Kalimat yang tepat untuk memperbaiki kalimat di atas adalah ...

(A) Saya akan datang, jika ada yang menjemput saya dengan mobil hari ini. (B) Dengan demikian dapat disimpulkan, bahwa penyebab bencana ini terkait dengan ulah manusia yang terlalu rakus, sehingga pohon di hutan habis dibabat semua. (C) Dia masih sempat membungkukkan badannya untuk menghormati kita, meskipun hatinya kesal sekali. (D) Oleh karena itu, saya sempatkan untuk mendatangi dan menyalaminya walaupun badan saya hari ini belum sepenuhnya sehat. (E) Ia tidak berani dan tidak sanggup menatap matamu, karena merasa amat malu, terutama setelah niatnya untuk juga mengawini perempuan lain kamu ketahui.

(A) Hal ini disebabkan banyak orang memilih kereta rel listrik sebagai sarana transportasinya. (B) Hal ini dikarenakan oleh banyaknya orang yang memilih kereta rel listrik sebagai sarana transportasinya. (C) Hal ini disebabkan oleh banyaknya orang memilih kereta rel listrik sebagai sarana transportasinya. (D) Hal ini disebabkan oleh banyaknya orang yang memilih kereta rel listrik sebagai sarana transportasinya. (E) Hal ini dikarenakan banyak orang memilih kereta rel listrik sebagai sarana transportasinya.
27. Kata berimbuhan yang dipakai pada konteks yang tidak tepat ditemukan dalam kalimat ...

(A) Burung gereja beterbangan mendengar suara lonceng berdentang. (B) Anak-anak kejar-mengejar di halaman sekolah. (C) Para penonton berlari-larian turun ke tengah lapangan tatkala melihat api berkobar. (D) Amran dan Rio selalu surat-menyurat melalui e-mail. (E) Pemandangan indah terhalang karena lembah berselimutkan kabut.

c Universitas Indonesia

Downloaded from http://pak-anang.blogspot.com

Halaman 5 dari 12 halaman

Kode Naskah Soal:


29. Temperatur permukaan bumi sedang memanas dengan laju yang menentukan. Berbagai perubahan iklim terkait dengan zaman penting sebelumnya, yang disetel secara aktif oleh berbagai penyebab alamiah, seperti variasi pada orbit bumi yang memengaruhi jumlah sinar matahari. Dalam kasus-kasus itu, siklus pendinginan dan pemanasan berlangsung lambat selama bermilenium-milenium. Saat ini, hal itu berbeda. Iklim sedang berubah lebih cepat daripada yang pernah terjadi sebelumnya. Kegiatan manusia adalah penyebab utamanya. Sebagai contoh, pembakaran bahan bakar fosilminyak bumi, gas, batu baramembanjiri atmosfer dengan karbon dioksida. Hal ini memicu kenaikan tajam pada temperatur global rata-rata dalam abad terakhir. Dampaknya adalah pada pola penguapan yang berubah, gletser-gletser yang meleleh, badai-badai yang menguat, dan kenaikan permukaan laut. Ringkasan yang dapat dibuat dari bacaan tersebut adalah ...

208

31. Melalui konser "Langit Ketujuh", penonton diajak berpetualang menyusuri sebuah pertunjukan musik kolosal. Kata menyusuri bersinonim dengan ...

(A) menjelajahi. (B) mengikuti. (C) mengiringi.

(D) menjalani. (E) menempuh.

32. Kalimat berikut yang efektif adalah ...

(A) Temperatur permukaan bumi menjadi panas dengan laju yang menentukan, sedangkan iklim berubah lebih cepat daripada yang pernah terjadi sebelumnya. (B) Temperatur permukaan bumi sedang memanas. Kegiatan manusia adalah penyebab utama terjadi perubahan iklim. (C) Pembakaran bahan bakar fosil mengakibatkan naiknya temperatur permukaan bumi yang mengakibatkan perubahan iklim. (D) Kenaikan tajam pada temperatur global mengakibatkan perubahan pola penguapan yang terlihat pada melelehnya gletser, menguatnya badai, dan menaiknya permukaan laut. (E) Pada masa lalu, perubahan iklim tidak berlangsung cepat, tetapi secara alamiah.
30. Cara penulisan kata berikut sesuai dengan EYD, KECUALI ...

(A) Riset ini mengkaji tentang ide tentang pemberantasan korupsi. (B) Bukit itu amat tinggi, hijau, dan dengan keindahan yang menakjubkan. (C) Di Indonesia, bukan saja mempunyai kekayaan alam yang melimpah, melainkan juga keanekaragaman budaya. (D) Kepada mahasiswa yang belum menyerahkan makalah dapat menghubungi pengajar. (E) Kegiatannya setiap hari antara lain menata buku, membuat katalog, dan mengatur peminjaman buku.
33. Kalimat yang efektif adalah ...

(A) khazanah. (B) konkret. (C) karier.

(D) jadwal. (E) sejarahwan.

(A) Oleh karena itu, kita wajib menyantuni orang-orang yang menderita akibat krisis moneter. (B) Berdasarkan hasil penelitian kami menunjukkan bahwa tidak ada penduduk di desa ini yang kekurangan karena penghasilan mereka cukup besar. (C) Dengan sistem kuliah jarak jauh akan memungkinkan bertambahnya jumlah mahasiswa. (D) Penulisan biogra pengarang yang ditempatkan di halaman terakhir buku merupakan tanggung jawab seorang editor selain menyunting naskah. (E) Penyuntingan naskah membutuhkan tenaga yang teliti, sabar, tekun, dan harus disediakan waktu dan situasi yang kondusif untuk menyunting naskah.

c Universitas Indonesia

Downloaded from http://pak-anang.blogspot.com

Halaman 6 dari 12 halaman

Kode Naskah Soal:


34. Penetapan Angka Kredit (PAK) tahap I tahun 2009 yang merupakan hasil dari penilaian Daftar Usul Penetapan Angka Kredit (DUPAK) yang diajukan oleh 25 pustakawan dan 4 calon pustakawan sudah diterbitkan. Imbuhan pe-an pada kalimat berikut semakna dengan imbuhan pe-an pada kata bercetak miring dalam kalimat tersebut, KECUALI ...

208

(A) Pembukaan peringatan Hari Krida Pertanian (HKP) yang ke-37 tahun 2009 menurut rencana akan dilaksanakan secara serentak di seluruh Indonesia pada hari Senin, tanggal 22 Juni 2009. (B) Penghargaan diberikan kepada para mahasiswa berprestasi. (C) Penyusunan KTSP (Kurikulum Tingkat Satuan Pendidikan) memerlukan waktu yang tidak sedikit. (D) Berdasarkan Kurikulum 2006, penilaian kognitif, psikomotorik, dan afektif menjadi satu nilai dalam rapor. (E) Sekarang sudah tersedia penyaringan untuk menjernihkan air sehingga kita tidak perlu khawatir lagi.
35. Penulisan gabungan kata yang benar terdapat dalam kalimat ...

36. Dari sisi sosial, puasa melatih orang berdisiplin, cinta keadilan, dan kedamaian. Dari sisi kejiwaan, puasa membuat manusia pandai mengendalikan diri. Dari sisi kesehatan, berpuasa berarti mengistirahatkan saluran pencernaan, enzim, dan hormon yang biasanya bekerja untuk mencerna saluran makanan terus-menerus selama 18 jam. Kalimat yang tepat untuk menjadi awal paragraf di atas adalah ...

(A) Menjalankan ibadah puasa merupakan kegiatan yang menguntungkan. (B) Dari berbagai sisi, puasa merupakan kegiatan yang menguntungkan. (C) Ada beberapa manfaat yang diperoleh dalam kegiatan berpuasa. (D) Ibadah puasa, jika dijalankan dengan kesadaran, memberi keuntungan dari berbagai sisi. (E) Dalam agama Islam, puasa wajib dilakukan karena memberi manfaat pada manusia.
37. (1) Angka partisipasi di jenjang pendidikan tinggi masih menjadi salah satu persoalan besar pendidikan. (2) Sejauh ini, angka partisipasi terbilang rendah, yakni 18 persen. (3) Dari total jumlah penduduk berusia 1924 tahun, yakni sekitar 25 juta orang, baru delapan belas persen menikmati pendidikan tinggi. (4) Indonesia masih jauh tertinggal dibandingkan dengan Malaysia yang angka partisipasi di jenjang pendidikan tingginya sudah 35 persen atau Thailand sekitar 45 persen. (5) Selain persoalan akses, pemerintah juga masih bergulat dengan persoalan mutu dan relevansi pendidikan. Kalimat yang menyimpang dari topik kutipan di atas terdapat pada kalimat ...

(A) Senjata yang digunakan militer memanfaatkan infra merah. (B) Tawuran antarsiswa itu berhasil diselesaikan dengan damai. (C) Perusahaan itu membutuhkan pekerja yang serba bisa. (D) Tujuan daripada seminar itu mencari jalan ke luar dari krisis. (E) Kerjasama siswa saat ujian dilarang.

(A) kesatu. (B) kedua. (C) ketiga.

(D) keempat. (E) kelima.

c Universitas Indonesia

Downloaded from http://pak-anang.blogspot.com

Halaman 7 dari 12 halaman

Kode Naskah Soal:


38. Vivienne Westwood (58) dikenal sebagai perancang yang menggunakan karyanya sebagai cara menyatakan pandangan politik atas isu yang ia anggap penting, terutama yang menyangkut lingkungan dan hak asasi. Dia pernah memprotes kebijakan Presiden George Bush. Sebagai perancang mode, Westwood berhasil menggabungkan pandangan politiknya dalam desain kreatif dan mewujudkannya menjadi produk konkret. Pernyataan yang benar tentang Vivienne Westwood adalah ...

208

40. Solar sel sebagai alat yang dapat digunakan untuk memanfaatkan energi matahari perlu ditingkatkan pemakaiannya untuk mengatasi krisis minyak bumi yang semakin menipis persediaannya. Gagasan utama kalimat tersebut adalah ...

(A) Solar sel alat yang dapat digunakan untuk memanfaatkan energi matahari. (B) Solar sel perlu ditingkatkan pemakaiannya. (C) Pemanfaatan solar sel perlu ditingkatkan untuk mengatasi krisis minyak bumi. (D) Solar sel perlu ditingkatkan pemakaiannya untuk mengatasi krisis minyak bumi. (E) Minyak bumi semakin menipis persediaannya.

(A) Vivienne Westwood adalah perancang mode, pemerhati lingkungan, sekaligus pembela hak asasi. (B) Vivienne Westwood selalu mengkritik aktivitas para tokoh politik yang menyimpang. (C) Vivienne Westwood, selain menjadi perancang mode, juga menjadi pengamat politik. (D) Vivienne Westwood, perancang mode, mewujudkan pandangan politiknya dalam desain yang kreatif dan konkret. (E) Vivienne Westwood, perancang mode, mempunyai pandangan politik yang senantiasa diwujudkan dalam desain kreatif.
39. Jika banyak seniman kontemporer menikmati hidup di tengah kota besar, dia minggir dan tinggal di kampung. Ketika para pelukis gencar mengangkat budaya urban seraya berpameran dari galeri ke galeri, dia berkelana di desa-desa demi memberdayakan rakyat miskin lewat kesenian. Saat sebagian seniman makin makmur dan berjarak dari kehidupan nyata, dia masih setia pada komunitas desanya. Berdasarkan teks di atas, perumusan tentang "dia" yang paling tepat adalah ...

(A) "Dia" adalah seniman yang senang tinggal di kampung. (B) "Dia" adalah pelukis yang mengangkat budaya urban. (C) "Dia" adalah pelukis sekaligus seniman setia yang memperhatikan rakyat. (D) "Dia" adalah seniman setia yang memberdayakan rakyat miskin melalui kesenian. (E) "Dia" adalah seniman pengembara.

c Universitas Indonesia

Downloaded from http://pak-anang.blogspot.com

Halaman 8 dari 12 halaman

Kode Naskah Soal:

208

BAHASA INGGRIS
(1) ____________________________. (2) The rst requirement is to complete a course in a certied climbing school, where you learn about mountain conditions and safe climbing. (3) Another requirement is to wear special clothing; a helmet and climbing boots to complete your outt. (4) You must also carry special equipment: iron spikes to hammer into cracks for support and safety and special climbing ropes. (5) In addition, there are also strict climbing rules for mountaineers. (6) Mountaineers sometimes bring their mountain bikes. (7) You can climb alone or in teams of two or more. (8) When you climb in a team, each person is tied around the waist to the same rope to protect a climber who slips from falling. (9) At all times, you must be aware of falling rocks and when you are tired, you need to nd a ledge to rest on. (10) Most mountain climbers would agree that mountain climbing has many rewards. (11) The higher you climb, the more beautiful the view is of the valley oor. (12) You can see treetops, lakes, and distant mountains. (13) Above you, the sky and moving clouds are a spectacular sight. Gunakan Petunjuk A dalam menjawab soal nomor 41 sampai nomor 42. 41. The paragraph should begin with _______.

(A) Mountain climbing is a dangerous sport with special needs and strict climbing rules (B) Mountain climbing requires not only special clothing but also expensive equipment (C) There are some requirements that you have to meet before you go climbing (D) Mountain climbing is risky; yet, many people do it because of the rewards afterwards (E) Mountain climbing is not a popular sport as it needs rigorous preparation
42. The sentence which is irrelevant to the topic of the paragraph is _______.

(A) sentence 4 (B) sentence 6 (C) sentence 9

(D) sentence 11 (E) sentence 13

c Universitas Indonesia

Downloaded from http://pak-anang.blogspot.com

Halaman 9 dari 12 halaman

Kode Naskah Soal:

208

By the age of 5, most triplets are on par with their peers in mental and emotional development, but those born at the lowest weights may still lag behind, according to a study published in July 2009. Israeli researchers found that among the 126 singletons, twins and triplets they followed from birth to age 5, triplets generally trailed behind their peers in cognitive development over the rst two years of life. By age 5, however, many triplets had bridged the gaps in both IQ and social development, the researchers report in the journal Pediatrics. The exception was triplets whod been particularly growth-restricted in the womb those who, at birth, had weighed more than 15 percent less than the sibling with the highest birth weight. At age 5, these children were still developmentally behind both their siblings and peers. Until now, there had been no well-designed studies following the development of triplets over the rst few years of life. And the catch-up seen among most triplets in this study is "excellent news," lead researcher Dr. Ruth Feldman, of Bar-Ilan University, told Reuters Health. Parents of triplets, she said, should be aware that their children may be slower to reach developmental milestones in infancy, but most are likely to close that gap during the preschool years. Children who were born substantially smaller than their siblings may not catch up, however. At age 5, the study found, these children typically scored at the lower end of the normal range for intellectual, emotional and social development. For instance, Feldman explained, average verbal IQ was about 95, which, while within normal range, would make it dicult for a child to get through standard schooling. Growth restriction was common among the 21 sets of triplets in the study. In 65 percent, one sibling was born weighing more than 15 percent less than the heaviest sibling. The ndings on development point to the importance of giving these children extra attention from infancy onward, according to Feldman. "Knowing that these children respond to parental investment already in the rst months of life tells parents to be especially sensitive and responsive to these children," Feldman explained. In addition, she said, the childrens development during infancy and preschool should be continuously monitored, and parents and children should receive extra help when needed such as interventions to help children regulate their emotions and cultivate social skills, or to improve their attention and concentration abilities. More studies are also needed, the researchers note, to see whether the developmental gaps persist into later childhood and adolescence. Gunakan Petunjuk A dalam menjawab soal nomor 43 sampai nomor 47. 43. An appropriate title for the text is _______. 45. According to the text, when do triplets typically have the biggest gap in development?

(A) During infancy (B) When they reach the age of ve (C) After they reach the age of ve (D) During school years (E) During preschool years
46. The word persist in paragraph 4 can be best replaced by _______.

(A) Intellectual, Emotional and social development of Triplets (B) Triplets, Twins, and Singletons: A Comparison Study (C) Parental Guides to The Development of Babies (D) The Reason for the Slow Growth of Triplets (E) The Correlation between Birth-weight and Intellectual Development
44. According to the text, which triplet-baby would most probably be developmentally behind her peers?

(A) vanish (B) elapse (C) stay

(D) change (E) maintain

47. The text would most probably be found in _______.

(A) Born with less than 1.5 kg in weight (B) Born weighing 20 percent less than her siblings (C) Born with heaviest weight (D) Born weighing 21 percent more than her siblings (E) Born with lowest weight

(A) an advertisement of a children health center (B) an article in a medical journal (C) an article in a popular science magazine (D) an article in a Bar-Ilan University publication (E) a preface of a medical book

c Universitas Indonesia

Downloaded from http://pak-anang.blogspot.com

Halaman 10 dari 12 halaman

Kode Naskah Soal:

208

One morning in August 1888, Bertha Benz and her two sons, Eugen (15), and Richard (13), woke up early. They got dressed very quietly, ____(48)____ as not to waken the head of the family, Karl Benz. They left a note that said, "We are going to visit Grandma." They crept out to Mr. Benzs workshop , opened the door, and pushed out a three-wheeled ____(49)____. It was the fruit of Mr. Benzs long and hard work - the rst engine car. Imagine it: a wooden workshop ____(50)____, more similar to a horse carriage than to the cars we have now: no roof, no hood, two wheels in the back but only one wheel in the front, a kind of handle instead of a steering wheel, leather-covered seats, and, the most important part, a 2.5 horsepower single-cylinder four-stroke engine. The car that Mrs. Benz and her sons rolled out of the workshop had gone through a great deal of development since its registration (no. DRP 37435) two years before, but Mr. Benz felt it needed even more work and ____(51)____. He believed that most people would not trust such a new thing enough to be willing to buy it, and so it seemed to him that the prospects for any success on the market were not good. ____(52)____, his loving and energetic wife believed immensely in his abilities, and was more than certain that the fruit was ripe enough to be picked. Gunakan Petunjuk A dalam menjawab soal nomor 48 sampai nomor 52. 48. ....

(A) so (B) still (C) besides


49. ....

(D) however (E) moreover

(A) carriage (B) bicycle (C) wagon


50. ....

(D) wheelbarrow (E) vehicle

(A) construct (B) constructed (C) constructive (D) construction (E) constructively
51. ....

(A) existence (B) treatment (C) enjoyment


52. ....

(D) impression (E) improvement

(A) Thus (B) Besides (C) In contrast

(D) Therefore (E) Furthermore

c Universitas Indonesia

Downloaded from http://pak-anang.blogspot.com

Halaman 11 dari 12 halaman

Kode Naskah Soal:


Gunakan Petunjuk A dalam menjawab soal nomor 53 sampai nomor 60. 53. We recommend _______ information on the internet or ocial tourism boards before you decide where to go for your holiday.

208

57. The person recently elected headmaster of our school is well-qualied, sociable, and _______ to his job.

(A) his dedication is very high (B) he is highly dedicating (C) is dedicated highly (D) having a high dedication (E) highly dedicated
58. "How do you like your new bedroom?" "Well, it is too dark. I need _______ a brighter colour."

(A) to search (B) search (C) searching (D) that we search (E) you searching
54. "The earthquake happening at about 3 oclock yesterday caused many people to panic." "Including me, as I _______ on the third-oor of my oce at that time."

(A) to have it painted (B) to be painted (C) painting it (D) to have it paint (E) it to be painted
59. _______, alternative therapies are being accepted by more and more people, even the World Health Organization.

(A) worked (B) have worked (C) have been working (D) was working (E) had worked
55. "Did you receive our inquiry? When will we receive your conrmation?" "My apology. It seems that _______. Could you possibly resend it?"

(A) we mislay your letter (B) your letter is mislaid (C) we had mislaid your letter (D) your letter has been mislaid (E) your letter was mislaid
56. "Wheres the report, Ben? You told me it would be ready by now. I need it for the board meeting this afternoon." "Dont worry, Jim. _______ before the board meeting."

(A) Despite intolerant attitudes of the medical establishment (B) As the medical establishment has intolerant attitudes (C) Because of intolerant attitudes of the medical establishment (D) In case of intolerant attitudes of the medical establishment (E) Due to intolerant attitudes of the medical establishment
60. "This cant be the way to Edinburgh." What does the sentence mean?

(A) I have it nished (B) I have had it nished (C) I will have it nished (D) The report will nish (E) The report has nished

(A) The speaker was told that it wasnt the way Edinburgh. (B) The speaker probably knows the other way Edinburgh. (C) The speaker doesnt know the way Edinburgh. (D) The speaker thinks that it isnt the way Edinburgh. (E) The speaker claims that it isnt the way Edinburgh.

to to to to to

c Universitas Indonesia

Downloaded from http://pak-anang.blogspot.com

Halaman 12 dari 12 halaman

SELEKSI MASUK UNIVERSITAS INDONESIA

SIMAK UI
KEMAMPUAN DASAR
Matematika Dasar Bahasa Indonesia Bahasa Inggris

209
Universitas Indonesia 2010

PETUNJUK UMUM
1. Sebelum mengerjakan ujian, periksalah terlebih dulu, jumlah soal dan nomor halaman yang terdapat pada naskah soal. Naskah soal ini terdiri dari 12 halaman. 2. Tulislah nomor peserta Anda pada lembar jawaban di tempat yang disediakan. 3. Tulislah kode naskah soal ini, pada lembar jawaban di tempat yang disediakan. Kode naskah soal ini: 7. Tulislah jawaban Anda pada lembar jawaban ujian yang disediakan. 8. Untuk keperluan coret-mencoret, harap menggunakan tempat yang kosong pada naskah soal ini dan jangan pernah menggunakan lembar jawaban karena akan mengakibatkan jawaban Anda tidak dapat terbaca. 9. Selama ujian, Anda tidak diperkenankan bertanya atau meminta penjelasan mengenai soal-soal yang diujikan kepada siapapun, termasuk kepada pengawas ujian. 10. Setelah ujian selesai, Anda diharapkan tetap duduk di tempat Anda sampai pengawas ujian datang ke tempat Anda untuk mengumpulkan lembar jawaban. 11. Perhatikan agar lembar jawaban ujian tidak kotor, tidak basah, tidak terlipat, dan tidak sobek.

209

4. Bacalah dengan cermat setiap petunjuk yang menjelaskan cara menjawab soal. 5. Pikirkanlah sebaik-baiknya sebelum menjawab tiap soal, karena setiap jawaban yang salah akan mengakibatkan pengurangan nilai (penilaian: benar +4, kosong 0, salah -1). 6. Jawablah lebih dulu soal-soal yang menurut Anda mudah, kemudian lanjutkan dengan menjawab soal-soal yang lebih sukar sehingga semua soal terjawab.

PETUNJUK KHUSUS
PETUNJUK A:
Pilih satu jawaban yang paling tepat.

PETUNJUK B:
Soal terdiri dari 3 bagian, yaitu PERNYATAAN, kata SEBAB, dan ALASAN yang disusun berurutan. Pilihlah: (A) Jika pernyataan benar, alasan benar, dan keduanya menunjukkan hubungan sebab dan akibat (B) Jika pernyataan benar, alasan benar, tetapi keduanya tidak menunjukkan hubungan sebab dan akibat (C) Jika pernyataan benar dan alasan salah (D) Jika pernyataan salah dan alasan benar (E) Jika pernyataan dan alasan keduanya salah

PETUNJUK C:
Pilihlah: (A) Jika (1), (2), dan (3) yang benar (B) Jika (1) dan (3) yang benar (C) Jika (2) dan (4) yang benar (D) Jika hanya (4) yang benar (E) Jika semuanya benar

Kode Naskah Soal:


MATA UJIAN TANGGAL UJIAN WAKTU JUMLAH SOAL Keterangan : : : : : Matematika Dasar, Bahasa Indonesia, dan Bahasa Inggris 11 APRIL 2010 120 MENIT 60 Mata Ujian MATEMATIKA DASAR nomor 1 sampai nomor 20 Mata Ujian BAHASA INDONESIA nomor 21 sampai nomor 40 Mata Ujian BAHASA INGGRIS nomor 41 sampai nomor 60

209

MATEMATIKA DASAR
Gunakan Petunjuk A dalam menjawab soal nomor 1 sampai nomor 19. 1. Sebuah kotak berisi 3 bola merah dan 7 bola hijau. Secara acak, dilakukan pengambilan 2 bola tanpa pengembalian. Peluang terambilnya 2 bola dengan warna berbeda adalah .... 4. Jika x + y + 2z = k , x + 2y + z = k dan 2x + y + z = k , k = 0, maka x2 + y 2 + z 2 jika dinyatakan dalam k adalah ....

(A) (B) (C) (D) (E)

(A) 7/90 (B) 21/100 (C) 21/50

(D) 7/15 (E) 10/21

2. x1 dan x2 adalah bilangan bulat yang merupakan akar-akar persamaan kuadrat x2 (2p + 4)x + (3p + 4) = 0, di mana p adalah suatu konstanta. Jika x1 , p, x2 merupakan tiga suku pertama dari suatu deret geometri, maka suku ke-12 dari deret geometri tersebut adalah ....

k2 16 3k 2 16 4k 2 17 3k 2 8 2k 2 3

(A) 1
(C) 6 + 2 5 (D) 6 2 5

(B) 1

5. Suatu koloni bakteri membelah diri menjadi dua setiap 6 jam dan pada setiap 12 jam seperempat bagian dari koloni itu tidak bertahan hidup. Jika pada awal pengamatan terdapat x bakteri, maka setelah 36 jam, jumlah bakteri dalam koloni tersebut adalah ....

(E) 4
3. Nilai terbesar x dalam 0 < x < 2 sehingga = 3 sin 2x cos 2x 2 2 adalah ....

(A) 6x (B) 24x (C) 27x

(D) 48x (E) 64x

(A) (B) (C) (D) (E)

13 12 7 6 8 6 19 12 11 6

6. Dari huruf-huruf S, I, M, A, dan K akan disusun kata-kata yang tidak selalu bermakna. Peluang huruf vokal untuk selalu berdampingan adalah ....

(A) (B) (C) (D) (E)

1 5 2 5 1 2 3 5 4 5

c Universitas Indonesia

Downloaded from http://pak-anang.blogspot.com

Halaman 1 dari 12 halaman

Kode Naskah Soal:


7. Himpunan penyelesaian dari adalah ....
(2x7)8 (12+xx2 )3 (x2 +x2)6

209

<0

(A) {x |x < 3 atau x > 4} (B) {x | 3 < x < 4} (C) {x | 3 < x < 2 atau 1 < x < 4} (D) (E)
8. x x |1 < x <
7 |2 7 2

atau x > 4

<x<4

10. Seorang ahli gizi sedang merencanakan dua buah menu A dan B . Setiap gram dari menu A mengandung 2 unit lemak, 1 unit karbohidrat, dan 4 unit protein. Setiap gram dari menu B mengandung 3 unit lemak, 3 unit karbohidrat, dan 3 unit protein. Ahli gizi tersebut ingin menyiapkan menu yang mengandung setidaknya 18 unit lemak, 12 unit karbohidrat, dan 24 unit protein dari menu A dan B . Dengan memisalkan X adalah banyaknya gram menu A yang dibuat dan Y adalah banyaknya gram menu B yang dibuat, pertidaksamaan linier yang mewakili situasi di atas adalah ....

Luas segitiga pada gambar adalah .... cm (A) 4(1 3) (B) 4( 3 1) (C) 4( 3 + 1) (D) 2( 3 + 1) (E) 2(1 3) 9. Jika log 3 = a dan log 5 = b , maka 30 log(75 3 10) = ....
2 2

(A) 2X + 3Y 18; x 0; y 0 (B) 2X + 3Y 18; x 0; y > 0 (C) 2X + 3Y 18; x 0; y 0 (D) 2X + 3Y 18; x 0; y 0 (E) 2X + 3Y 18; x 0; y 0
11. Misalkan P = ps qr = .... (A) 1 atau 2 p r

X + 3Y 12; 4X + 3Y 24; X + 3Y 12; 4X + 3Y 24; X + 3Y 12; 4X + 3Y 24; X + 3Y 12; 4X + 3Y 24; X + 3Y 12; 4X + 3Y 24; q s

. Jika P 1 = 2P T , maka

(A) (B) (C) (D) (E)

1+a+7b 3+a+b 1+3a+7b 3+a+b 1+3a+7b 3+3a+3b 1+7a+3b 3+a+b 1+7a+3b 3+3a+3b

1 (B) 1 2 atau 2 1 (C) 1 2 2 atau 2 2 (D) 2 atau 2

(E) 1 atau 1
12. Jika f (x) =
3 log x 12(3 log x) ,

maka f (x) + f

3 x

= ....

(A) 3 (B) 2 (C) 1

(D) 1 (E) 3

13. Persamaan kuadrat x2 px + q = 0 akar-akarnya m dan n. Jika m, n, p, q merupakan barisan aritmetika, m = .... maka n 1 4 1 (B) 2 3 (C) 4 (D) 2

(A)

(E) 4
Downloaded from http://pak-anang.blogspot.com

c Universitas Indonesia

Halaman 2 dari 12 halaman

Kode Naskah Soal:


3 14. Jika cos x, sin x, dan 2 membentuk barisan geometri, maka suku ke-5 dari barisan tersebut adalah .... 1 3 (A) 18

209

19. Sebuah balon berbentuk bola sedang dipompa sehingga volumenya bertambah 100 cm3 per detik. Laju perubahan jari-jari balon ketika diameternya mencapai 50 cm adalah ....

(B) (C) (D) (E)

9 2

1 2 1 3 3

(A) (B)

1 25 1 5

(C) (D) 5 (E) 25

1 ,yz = , maka 15. Jika diketahui x y = 21 3 2+ 3 2 2 2 nilai x + y + z xy yz xz = ....

(A) 5 (B) 10 (C) 15

(D) 20 (E) 25

Gunakan Petunjuk C dalam menjawab soal nomor 20 . 20. Diberikan sepasang persamaan 2x 3y = 13 dan 3x + 2y = b dengan 1 b 100, dan b bilangan bulat. Misalkan n2 = x + y , dengan x dan y adalah solusi dari persamaan di atas, yang berupa bilangan bulat, maka nilai n yang memenuhi adalah .... (1) 4 (2) 3 (3) 1 (4) 2

(x + 1)10 16. Jika diketahui f g h(x) = , (x + 1)10 + 1 x f ( x) = dan h(x) = x + 3, maka g (x + 5) x+1 adalah ....

(A) (x 2)10 (B) (x + 3)10 (C) (x + 5) (D) (x 2)5 (E) (x 3)10


17. Diketahui sebuah barisan mempunyai urutan sebagai berikut: 32,14,16,12,8,10,.... Suku ke-21 dari barisan di atas adalah ....

(A) 6 (B) 2 (C) (D)


1 32 1 2

(E) 2
18. Diketahui fungsi kuadratf (x) = x2 + x + 2. Dua buah garis singgung di titik yang merupakan perpotongan antara f (x) dan y = 2 membentuk sebuah segitiga dengan garis y = 2. Luas dari segitiga yang terbentuk adalah ....

(A) (B) (D) (E)

1 4 1 2

(C) 1
3 2 5 2

c Universitas Indonesia

Downloaded from http://pak-anang.blogspot.com

Halaman 3 dari 12 halaman

Kode Naskah Soal:

209

BAHASA INDONESIA
Gunakan Petunjuk A dalam menjawab soal nomor 21 sampai nomor 40. 21. Taman Wisata Candi Borobudur akan dijadikan sebagai lembaga ... gajah dan untuk memperbanyak ... gajah, tahun ini, Taman Wisata Candi Borobudur akan menambah dua ekor gajah lagi, jantan dan betina. Kata yang tepat untuk mengisi kalimat rumpang tersebut adalah ... 24. Kalimat-kalimat berikut merupakan kalimat efektif, KECUALI ...

(A) preservasi, habitat. (B) rehabilitasi, komunitas. (C) pelestarian, spesies. (D) konservasi, populasi. (E) penelitian, habitat.
22. Dalam perjalanan itu kami mendatangi beberapa tempat wisata. Kata perjalanan pada kalimat di atas mempunyai makna yang sama dengan kata pada kalimat berikut, KECUALI ...

(A) Sebelum menyerahkan tugas, Anda sebaiknya memeriksanya secara cermat terlebih dahulu. (B) Kami akan menyampaikan prakiraan cuaca kota-kota besar yang berlaku besok. (C) Panitia seminar itu belum menentukan di mana rapat itu diselenggarakan. (D) Mobil kami hampir kehabisan bensin ketika sampai di Cirebon. (E) Polisi berhasil menangkap pelaku perusakan bantalan rel kereta api itu.
25. Sebanyak 109 spesies dalam hutan restorasi Harapan Rainforest di perbatasan Jambi-Sumatera Selatan terancam punah. Hal itu disebabkan menurunnya kerapatan pepohonan dalam kawasan hutan tersebut. Penelitian yang dilakukan selama 15 bulan di kawasan restorasi ekosistem Jambi-Sumatera Selatan menunjukkan, keberadaan ratusan jenis hewan dan tumbuhan terancam punah, yaitu 293 spesies burung, 154 mamalia, 27 ambi, 42 reptilia, dan 444 spesies pohon. Kalimat yang tepat untuk melanjutkan paragraf tersebut adalah ...

(A) Perbaikan rumah itu memakan waktu tiga bulan. (B) Beberapa orang warga mengikuti pertandingan bulu tangkis. (C) Para pendaki gunung itu membawa perbekalan yang cukup. (D) Persahabatan mereka sudah berjalan selama sepuluh tahun. (E) Pertemuan kedua sahabat itu sangat mengharukan.
23. Sebelum konser dimulai, di backstage yang terletak di bawah panggung ada acara pemberian penghargaan multiplatinum pada The Black Eyed Peas (BEP) dari Universal Music Indonesia karena album mereka terjual lebih dari 150 ribu kopi. Inti kalimat tersebut adalah ...

(A) Masih banyak lagi jenis makhluk hidup lainnya yang hampir punah. (B) Sudah seharusnya pemerintah melindungi spesies hewan di kawasan itu. (C) Sangat disayangkan jika spesies yang seharusnya dilindungi tersebut sampai punah. (D) Sebaiknya penelitian terhadap hewan dan tumbuhan juga dilakukan di tempat lain. (E) Kepunahan hewan dan tumbuhan itu adalah akibat ulah segelintir orang.
26. Cara penulisan gabungan kata yang benar adalah ...

(A) Album BEP terjual lebih dari 150 ribu kopi. (B) Universal Music Indonesia memberikan penghargaan pada BEP. (C) Penghargaan multiplatinum diberikan di bawah panggung. (D) Ada acara pemberian penghargaan. (E) Penghargaan multiplatinum diberikan sebelum konser dimulai.

(A) kerjasama. (B) karya wisata. (C) tatabahasa. (D) anti komunis. (E) simpang lima.

c Universitas Indonesia

Downloaded from http://pak-anang.blogspot.com

Halaman 4 dari 12 halaman

Kode Naskah Soal:


27. Kata-kata berikut yang semua ditulis secara baku adalah ...

209

30. Kalimat yang tidak efektif dan ditulis dengan ejaan yang tidak benar adalah ...

(A) azasi; akte; ijazah. (B) zaman; khawatir; khusyu. (C) lembap; antre; khaul. (D) khazanah; khawatir; khuldi. (E) trampil; paedagogis; energi.
28. Pada akhirnya, media hanyalah pilihan. Dari cat minyak sampai arang, dari kayu sampai baja antikarat, dari fotogra sampai seni rupa video, dari cukil kayu sampai cetak digital, dan seterusnya. Ketika sebuah karya seni rupa cat air tak membuat kita berpikir bahwa karya ini dapat dibuat lebih baik dengan cat minyak, ketika karya seni gras cetak saring tak mengingatkan sebaiknya karya ini dibuat dengan cukil nilo, dapat dikatakan bahwa seniman memilih media yang pas untuk dirinya. Gagasan pokok paragraf tersebut terletak pada ...

(A) Di sekitar tempat ini ada buah-buahan yang dapat kita makan. (B) Menurut penelitian menunjukkan bahwa wanita tidak suka makan pisang Ambon. (C) Konon laki-laki lebih menyukai buah durian daripada buah kedondong. (D) Kita membutuhkan kamus yang lengkap untuk mendukung aktivitas kita sehari-hari yang lebih banyak berurusan dengan kata. (E) Sudah saatnya kita menggalakkan kegiatan agar produktivitas dan kreativitas kita yang selama ini menurun bisa pulih kembali.
31. Kunyit merupakan salah satu bumbu alami yang bisa membantu mengatasi radang. Dilaporkan oleh majalah Prevention bahwa kunyit mampu membunuh sel-sel kanker, menunda pertumbuhan tumor, dan mendorong efektivitas kemoterapi. Beberapa riset menemukan bahwa kunyit efektif jika dicampur dengan lada hitam dan minyak zaitun. Kunyit yang sudah diolah dalam bentuk bubuk hanya mengandung dua puluh persen dari total kandungan aslinya. Jadi, lebih baik mengonsumsi kunyit yang belum diolah. Pesan berikut tidak sesuai dengan isi kutipan di atas, KECUALI ...

(A) awal paragraf. (B) akhir paragraf. (C) awal dan akhir paragraf. (D) seluruh paragraf. (E) tengah paragraf.
29. Kalimat yang tidak efektif adalah ...

(A) Apabila ia ingin naik kelas, harus rajin belajar. (B) Para tamu dipersilakan duduk. (C) Menurut saya, masih banyak calon mahasiswa yang bingung dalam menentukan pilihan studi mereka. (D) Tidak ada yang sulit dalam hidup ini, bukan? (E) Mahasiswa yang belum lulus Bahasa Inggris diberi kesempatan untuk mengikuti ujian susulan.

(A) Untuk membunuh se-sel kanker di dalam tubuh Anda, konsumsilah kunyit. (B) Jika ingin merasakan kehebatan kunyit, cobalah olahan kunyit setiap saat. (C) Supaya kunyit mampu membunuh sel-sel kanker, campurlah dengan lada hitam. (D) Agar khasiat kunyit lebih baik, gunakanlah kunyit hasil olahan sendiri. (E) Apabila Anda ingin kepraktisan, nikmatilah kunyit buatan sendiri.

c Universitas Indonesia

Downloaded from http://pak-anang.blogspot.com

Halaman 5 dari 12 halaman

Kode Naskah Soal:


32. Cara penulisan kata serapan yang terdapat dalam kalimat berikut salah, KECUALI ...

209

35. Kata berimbuhan yang dipakai pada konteks yang tidak tepat ditemukan dalam kalimat ...

(A) Sejak dini anak-anak perlu dilatih dan distimulir untuk mandiri agar kelak tidak menjadi beban orang lain. (B) Ia terlihat frustasi ketika anak yang dibangga-banggakan selama ini ternyata tidak lulus. (C) Ia memiliki tipe wajah fotojenik sehingga terpilih sebagai gadis sampul. (D) Perundingan prelimineri diselenggarakan sebelum nota kesepahaman ditandatangani. (E) Tidak terlintas dalam benak saya bahwa Kadri akan menjadi anggota kelompok penganut kepercayaan yang ekstrem.
33. Kalimat yang efektif dan ditulis dengan ejaan yang benar adalah ...

(A) Burung gereja beterbangan mendengar suara lonceng berdentang. (B) Anak-anak kejar-mengejar di halaman sekolah. (C) Para penonton berlari-larian turun ke tengah lapangan tatkala melihat api berkobar. (D) Amran dan Rio selalu surat-menyurat melalui e-mail. (E) Pemandangan indah terhalang karena lembah berselimutkan kabut.
36. Cara penulisan kata berikut sesuai dengan EYD, KECUALI ...

(A) khazanah. (B) konkret. (C) karier.

(D) jadwal. (E) sejarahwan.

(A) Anak perlu dididik dengan benar agar tumbuh menjadi anak yang kreatifnya tinggi. (B) Jangan mengandalkan perkembangan intelegensi anak semata-mata hanya pada gurunya saja. (C) Boleh memprotes pembangunan jalan ini, tapi anda harus konsekuensi. (D) Saputangan berdarah ini menjadi tanda bahwa saya masih mencintainya. (E) Dalam makalah singkat ini membahas ciri dan perilaku lima etnik di Indonesia.
34. Kalimat yang tidak efektif adalah ...

37. Para penulis sering memilih dan menggunakan makna kata yang berpretensi ilmiah tanpa memperhatikan untuk siapa buku itu ditulis. Kata berpretensi dalam kalimat tersebut dapat diganti dengan kata ...

(A) berisi. (B) berlagak. (C) berpotensi.

(D) beragam. (E) bermakna.

(A) Tindak kekejaman, kekerasan, dan menindas orang kecil merupakan perbuatan tidak terpuji. (B) Ketika saya datang, mereka sudah berkumpul di halaman sekolah untuk menanti inspektur upacara. (C) Tata tertib ini tidak boleh diubah sampai ada tata tertib baru yang disahkan oleh pimpinan. (D) Sebagai mahasiswa, Anda diharapkan dapat memberi keteladanan yang baik. (E) Diperlukan orang yang sanggup berpikir kritis dan tidak ekstrem.

38. Salju abadi di Kibo Area, Kilimanjaro, terbentuk karena glacier abadi. Namun, kini, luas glacier abadi itu terus menyusut seiring terjadinya pemanasan global. Saat ini lapisan glacier di Kilimanjaro tinggal 2,2 km2 dari total 19 km2 di Afrika. Padahal, pada 1982, luas glacier di Kilimanjaro masih 3,3 km2 . Hal ini berarti ada penyusutan 33% dalam waktu 27 tahun. Oleh karena itu, diperkirakan dalam waktu 12 dekade ke depan, lapisan glacier itu akan lenyap dan Kilimanjaro pun akan terpuruk menjadi seonggok gunung batu berdebu. Mengapa salju abadi di Kibo Area, Kilimanjaro, pada sekitar 12 dekade mendatang hanya akan menjadi gunung batu berdebu saja?

(A) Salju yang ada di tempat itu akan menyusut. (B) Terjadi pemanasan global. (C) Terjadi pelelehan lapisan glacier seiring dengan pemanasan global. (D) Hal ini disebabkan oleh terjadinya penyusutan glacier sebanyak 33%. (E) Terjadi fenomena alam berupa pemanasan global selama 27 tahun di area itu.

c Universitas Indonesia

Downloaded from http://pak-anang.blogspot.com

Halaman 6 dari 12 halaman

Kode Naskah Soal:


39. Apa saja segi buruk permen? Pertama, kandungan vitamin dalam permen sangat sedikit. Begitu pun kandungan seratnya nyaris tidak ada. Permen juga menyebabkan batuk, membuat gemuk, menimbulkan sakit gigi, menghilangkan nafsu makan, hingga membuat anak jadi hiperaktif. Riset terbaru bahkan menyebutkan anak yang terbiasa makan permen sejak kecil, saat dewasa cenderung memiliki perilaku kriminal. Para peneliti dari Universitas Cardi, Wales, Inggris, yang melakukan riset ini mengamati data 17.415 anak yang lahir pada bulan April 1970 di Inggris. Data yang diambil dari British Cohort Study itu secara terperinci menyebutkan informasi kesehatan dan gaya hidup anak-anak tersebut pada periode tertentu, misalnya saat usia 5, 10, dan dewasa. Di usia 34 tahun, para responden diwawancarai apakah mereka pernah melakukan tindakan kriminal. Hasilnya, 69 persen yang pernah melakukan tindakan kekerasan atau kriminal ternyata memiliki riwayat hobi makan permen di usia 10 tahun, dibandingkan dengan 42 persen yang tidak punya catatan kriminal. Jadi, perlukah orang tua mengibarkan "bendera perang" pada permen? Tidak sepenuhnya demikian, kata Simon Moore, dosen senior dalam studi Violence and Society Research Group, Universitas Cardi. Intisari dari kutipan tersebut yang benar adalah ...

209

40. Selama bertahun-tahun, isu kemiskinan menjadi perhatian serius dan fokus kajian para sarjana dan perumus kebijakan publik. Kemiskinan dianalisis dari berbagai sudut pandang dan pendekatan guna mendapatkan pemahaman yang utuh. Kemiskinan bukan gejala sederhana, pun tidak terkait ekonomi semata-mata, tetapi saling terkait dengan masalah lain yang amat kompleks. Para sarjana mencoba menerangkan denisi baru kemiskinan dengan memberi makna melampaui pengertian konvensional yang selama ini dipahami umum. Kesimpulan yang dapat ditarik dari bacaan di atas ...

(A) Kemiskinan menjadi topik pembahasan para ahli. (B) Kemiskinan menjadi keprihatinan bagi para ahli. (C) Kemiskinan bersifat kompleks. (D) Batasan kemiskinan sulit dirumuskan. (E) Denisi kemiskinan perlu dirumuskan kembali sesuai dengan perkembangan zaman.

(A) Orang tua jangan sering memberikan permen kepada anaknya. (B) Hasil riset tentang permen mengagetkan orang tua. (C) Permen mempunyai dampak buruk bagi penikmatnya. (D) Anak yang suka makan permen setelah besar melakukan tindakan kekerasan. (E) Kandungan vitamin pada permen sangat sedikit.

c Universitas Indonesia

Downloaded from http://pak-anang.blogspot.com

Halaman 7 dari 12 halaman

Kode Naskah Soal:

209

BAHASA INGGRIS
(1) ____________________________. (2) However, plastic in the ocean actually decomposes as it is exposed to the rain and sun and other environmental conditions. (3) When plastic decomposes it releases potentially toxic bisphenol A (BPA) and PS oligomer into the water. (4) BPA and PS oligomer are sources of concern because they can disrupt the functioning of hormones in animals and can seriously aect reproductive systems. (5) Polystyrene begins to decompose within one year, releasing components that are detectable in the parts-per-million range. (6) However, the volume of plastics in the ocean is increasing, so that decomposition products remain a potential problem. (7) It is estimated that 10 percent of the worlds plastic waste nds its way into the sea and slowly but surely most of it ends up in the Pacic Ocean. (8) Each year as much as 150,000 tons of plastic debris, most notably Styrofoam, washes up on the shores of Japan alone. (9) Vast expanses of waste, consisting mainly of plastic, oat elsewhere in the oceans. Gunakan Petunjuk A dalam menjawab soal nomor 41 sampai nomor 42. 41. The paragraph should begin with _______.

(A) Plastics in daily use are generally assumed to be quite stable (B) Scientists always believed that plastics in the oceans were hazardous only to sea animals (C) The UN Environmental Program estimates that over a million seabirds, as well as more than 100 thousand marine mammals, die every year from ingesting plastic (D) Some researchers estimate that there are over six kilos of plastic for every kilo of naturally occurring plankton in the Pacic Ocean (E) Plastics decompose with surprising speed and release potentially toxic substances into the water
42. The sentence which is irrelevant to the topic of the paragraph is sentence number _______.

(A) four (B) ve (C) six

(D) seven (E) eight

c Universitas Indonesia

Downloaded from http://pak-anang.blogspot.com

Halaman 8 dari 12 halaman

Kode Naskah Soal:

209

Spider-Man isnt the only person with an interest in spider silk. While Spider-Man uses the threads to zigzag from building to building, or to snare a bad guy, scientists are investigating silk for dierent reasons. Though researchers have learned a lot about silk by investigating spiders, insects such as caterpillars, ants and bees also have been studied for the sticky stu. Scientists are even trying to get silk from animals such as goats. It turns out silk might be good for weaving a lot more than shirts and ties. In the future, the silky ber might be used to make supertough bulletproof vests and light but strong parachute cords. Silk also might work well for delicate tasks inside the body. Researchers are experimenting with using silk to support growing cells, the same way a construction crew builds scaolding around a building to help keep everything in its place during construction. Spider silk is an ideal material, according to a researcher from University of Wyoming, but humans have been gathering silk not from spiders but from silkworms for hundreds of years. But silkworm silk has its problems. A silkworm covers its silk in sticky glue that holds the cocoon together. Sometimes humans have a bad allergic reaction to this glue. Spiders, on the other hand, dont use sticky glue. Most spiders have an abdomen made up of ve dierent sections. The last two sections are where the silk-making happens. These sections of the lower belly are modied into special structures called spinnerets, which are sort of like faucets for silk. The silk is mixed in glands and then secreted out of the spinnerets. Spiders cant shoot silk out for long distances the way Spider-Man does. Instead, they attach the emerging silk to something, like a tree branch, and then move away from the branch. This pulls the silk outward. The main ingredient in spider silk is proteins, and there are many dierent kinds, depending on which spider is spinning and which silk it wants to make. Some of the proteins are very large and complicated, and therefore hard to make a lot of in the lab. So some scientists have put the genes that have the instructions for making silk into other creatures, such as goats. The silk-making genes are turned on only in the goat cells that make milk, so when these goats are milked, there is silk in the milk. However, a liter of milk may have only 15 grams of silk, which means it would take about 600 gallons of milk to make one bulletproof vest. At higher concentrations the milk starts clumping, perhaps because the silk proteins are sticking to milk proteins. Gunakan Petunjuk A dalam menjawab soal nomor 43 sampai nomor 47. 43. An appropriate title for the text is _______. 45. Why did the writer mention building construction?

(A) To illustrate how strong and elastic silk is (B) To describe how researchers extract silk out of spiders (C) To explain about the process of silk making (D) To illustrate how growing cells can be held up by silk (E) To compare between silk from silkworms and silk from spiders
46. The word clumping in paragraph 5 is best replaced by _______.

(A) The Chemical Characteristic of silk (B) Silk Making : from Worms to Spiders (C) Articial Spider Silk Production (D) The Long History of Silk (E) Silks Superpower
44. Which eld might use silk in the future?

(A) Health and automotive (B) Agriculture and military (C) Biochemistry and construction (D) Military and medicine (E) Mining and hospitality

(A) clotting (B) cluttering (C) spreading

(D) dispersing (E) collecting

c Universitas Indonesia

Downloaded from http://pak-anang.blogspot.com

Halaman 9 dari 12 halaman

Kode Naskah Soal:


47. Which of the following statements about the text is FALSE?

209

(A) The use of silk in health is still empirical. (B) Silk making genes can be planted into animals other than goats. (C) Humans have long been collecting spider silk for various purposes. (D) Secreting silk from specially-engineered milk is eective but not ecient. (E) In the future, we will possibly use silk made by bees or ants.

c Universitas Indonesia

Downloaded from http://pak-anang.blogspot.com

Halaman 10 dari 12 halaman

Kode Naskah Soal:

209

The power of music aects all of us and has long appealed to our emotions. In fact, the earliest known music-making is in Stone Age culture, ____(48)____ we have been tapping our toes, humming along, singing and dancing ever since. It is for this reason that UCLA researchers are using music to help children with autism spectrum disorders (ASD), for whom understanding emotions is a very dicult task. This inability enables them of the chance to ____(49)____ eectively and make friends and can often lead to social isolation and loneliness. The researchers have developed a music education program ____(50)____ to help children with ASD better understand emotions and learn to recognize emotions in others. Specically, the children are using a method of music ____(51)____ known as the Or-Schulwerk approach. Developed by 20th -century German composer Carl Or ("schulwerk" is German for schooling), it is a unique approach to music learning that is supported by movement and based on things that kids ____(52)____ like to do, such as sing, chant rhymes, clap, dance and keep a beat or play a rhythm on anything near at hand. Or called this music and movement activity "elemental" - basic, unsophisticated and concerned with the fundamental building blocks of music. Gunakan Petunjuk A dalam menjawab soal nomor 48 sampai nomor 52. 48. ....

(A) if (B) and (C) so


49. ....

(D) but (E) as

(A) communicate (B) communication (C) communicating (D) communicator (E) communicated
50. ....

(A) discovered (B) connected (C) designed


51. ....

(D) related (E) improved

(A) educate (B) educated (C) educating


52. ....

(D) education (E) educative

(A) intuition (B) intuitive (C) intuitiveness (D) intuit (E) intuitively

c Universitas Indonesia

Downloaded from http://pak-anang.blogspot.com

Halaman 11 dari 12 halaman

Kode Naskah Soal:


Gunakan Petunjuk A dalam menjawab soal nomor 53 sampai nomor 60. 53. "My computer wont boot up." "_______"

209

57. Since dictionaries ________ to be read but to be consulted when necessary, the lexicographer must present his material in a concise, convenient, and consistent style.

(A) Mind if I use it? (B) Have you tried it? (C) Why dont you save it? (D) I want to speed it up. (E) Good thing youve backed up your data.
54. "The earthquake happening at about 3 oclock yesterday caused many people to panic." "Including me, as I _______ on the third-oor of my oce at that time."

(A) do not mean (B) are not meaning (C) did not mean (D) were not meant (E) are not meant
58. "Do you have tight working hours?" "No. As long as I ________, I am allowed to have exible working hours."

(A) did my job (B) got my job to do (C) am doing my job (D) have my job done (E) I am done
59. "I just ordered a large pizza for the two of us." "Are you serious? You _____ hungry. Youve just had your big lunch."

(A) worked (B) have worked (C) have been working (D) was working (E) had worked
55. "Wheres the report, Ben? You told me it would be ready by now. I need it for the board meeting this afternoon." "Dont worry, Jim. _______ before the board meeting."

(A) shouldnt be (B) mustnt be (C) cant be

(D) are not (E) may not be

(A) I have it nished (B) I have had it nished (C) I will have it nished (D) The report will nish (E) The report has nished
56. "Did you receive our inquiry? When will we receive your conrmation?" "My apology. It seems that _______. Could you possibly resend it?"

60. If my brother had not gone for a vacation to Singapore last week, he would not have been infected with H1N1 u which has killed some people in Indonesia. From the above sentence we may conclude that _______.

(A) my brother had been to Singapore before he became infected (B) nobody was cured after getting H1N1 (C) my brother caught the H1N1 u some time ago (D) People who went to singapore caught H1N1 (E) my brother died because of the H1N1 u

(A) we mislay your letter (B) your letter is mislaid (C) we had mislaid your letter (D) your letter has been mislaid (E) your letter was mislaid

c Universitas Indonesia

Downloaded from http://pak-anang.blogspot.com

Halaman 12 dari 12 halaman

You might also like